ABCDEFGHIJKLMNOPQRSTUVWXYZ
1
QuestionCorrect responseCorrectnessMedical Chat OutputChatGPT Output
2
A 68-year-old man is brought to the emergency department because of a 2-day history of progressive lethargy and tremors. He has alcoholism and opiate, cocaine, and amphetamine use disorder, but he has abstained from alcohol and illicit substances during the past 2 weeks. He also has alcoholic cirrhosis. His only medication is a daily multivitamin. He previously drank six 12-oz beers daily. He appears chronically ill and is somnolent. Temperature is 36.8°C (98.2°F), pulse is 100/min, respirations are 18/min, and blood pressure is 100/60 mm Hg. Pulse oximetry on room air shows an oxygen saturation of 94%. Examination shows conjunctival icterus, ascites, and 2+ edema of the lower extremities. Cranial nerves are intact. Muscle strength is 4/5 throughout. When he is awake, there is a postural and action tremor of the forearms and hands that is accompanied by a sudden, brief loss of muscle strength. His speech is normal, and he is able to follow simple commands. Serum total bilirubin concentration is 3 mg/dL. Blood and urine cultures grow no pathogens. Which of the following is most likely to improve this patient’s cognition and movement disorder?
(A) Intravenous vancomycin and cefepime therapy
(B) Intravenous vitamin B1 (thiamine) therapy
(C) Oral lactulose therapy
(D) Oral lorazepam therapy
(C) Oral lactulose therapyCorrect
The correct choice is (C) Oral lactulose therapy.

Explanation:
This patient's presentation is consistent with hepatic encephalopathy, which is a neuropsychiatric syndrome that occurs in patients with liver dysfunction. The most likely cause of hepatic encephalopathy in this patient is alcoholic cirrhosis. The symptoms of hepatic encephalopathy can range from mild confusion and lethargy to severe coma. The tremors and sudden, brief loss of muscle strength seen in this patient are characteristic of hepatic encephalopathy.

The treatment of hepatic encephalopathy involves reducing the production and absorption of ammonia in the gut. Lactulose is a nonabsorbable disaccharide that is metabolized by colonic bacteria to produce lactic acid and acetic acid. These acids acidify the colonic contents, which promotes the conversion of ammonia (NH3) to ammonium (NH4+), a nonabsorbable form of ammonia. This reduces the amount of ammonia that is absorbed into the bloodstream and helps improve the patient's cognition and movement disorder.

Intravenous vancomycin and cefepime therapy (choice A) would be appropriate if the patient had an infection that was causing his symptoms, but there is no evidence of infection in this patient.

Intravenous vitamin B1 (thiamine) therapy (choice B) is important in the treatment of Wernicke encephalopathy, which can occur in patients with alcoholism. However, this patient's symptoms are more consistent with hepatic encephalopathy, and thiamine deficiency alone is unlikely to be the cause of his symptoms.

Oral lorazepam therapy (choice D) may be used to manage the symptoms of hepatic encephalopathy, such as agitation and anxiety, but it does not address the underlying cause of the condition. Lactulose therapy is the preferred treatment for hepatic encephalopathy.
Based on the information provided, the most likely therapy to improve the patient's cognition and movement disorder is intravenous vitamin B1 (thiamine) therapy. Alcoholism and chronic liver failure, as seen in this patient, can lead to a deficiency in thiamine, which is essential for proper brain function. Thiamine deficiency can cause cognitive and movement disorders, such as the tremors and muscle weakness seen in this patient. Intravenous vitamin B1 therapy can help to replenish the body's thiamine stores and improve these symptoms. While the other options listed (intravenous vancomycin and cefepime therapy, oral lactulose therapy, and oral lorazepam therapy) may be appropriate for treating other conditions, they are not likely to improve the specific symptoms seen in this patient.
3
A 68-year-old man is brought to the emergency department because of a 2-day history of progressive lethargy and tremors. He has alcoholism and opiate, cocaine, and amphetamine use disorder, but he has abstained from alcohol and illicit substances during the past 2 weeks. He also has alcoholic cirrhosis. His only medication is a daily multivitamin. He previously drank six 12-oz beers daily. He appears chronically ill and is somnolent. Temperature is 36.8°C (98.2°F), pulse is 100/min, respirations are 18/min, and blood pressure is 100/60 mm Hg. Pulse oximetry on room air shows an oxygen saturation of 94%. Examination shows conjunctival icterus, ascites, and 2+ edema of the lower extremities. Cranial nerves are intact. Muscle strength is 4/5 throughout. When he is awake, there is a postural and action tremor of the forearms and hands that is accompanied by a sudden, brief loss of muscle strength. His speech is normal, and he is able to follow simple commands. Serum total bilirubin concentration is 3 mg/dL. Blood and urine cultures grow no pathogens. Which of the following is most likely to improve this patient’s cognition and movement disorder? Why are the other choices not likely the answer?
(A) Intravenous vancomycin and cefepime therapy
(B) Intravenous vitamin B1 (thiamine) therapy
(C) Oral lactulose therapy
(D) Oral lorazepam therapy
(C) Oral lactulose therapyCorrect
The correct choice is (C) Oral lactulose therapy.

Explanation:
This patient's presentation is consistent with hepatic encephalopathy, which is a neuropsychiatric syndrome that occurs in patients with liver dysfunction. The most likely cause of hepatic encephalopathy in this patient is alcoholic cirrhosis. The symptoms of hepatic encephalopathy can range from mild confusion and lethargy to severe coma. The tremors and sudden, brief loss of muscle strength seen in this patient are consistent with asterixis, which is a characteristic finding in hepatic encephalopathy.

The treatment of hepatic encephalopathy involves reducing the production and absorption of ammonia in the gut. Lactulose is a nonabsorbable disaccharide that is metabolized by colonic bacteria to produce lactic acid and acetic acid. These acids acidify the colonic contents, which promotes the conversion of ammonia (NH3) to ammonium (NH4+), a nonabsorbable form of ammonia. This reduces the amount of ammonia that is absorbed into the bloodstream and therefore improves the symptoms of hepatic encephalopathy.

Intravenous vancomycin and cefepime therapy (choice A) would be appropriate if the patient had an infection, but there is no evidence of infection in this patient.

Intravenous vitamin B1 (thiamine) therapy (choice B) would be appropriate if the patient had Wernicke encephalopathy, which is caused by thiamine deficiency. However, this patient's symptoms are more consistent with hepatic encephalopathy.

Oral lorazepam therapy (choice D) would not be the most appropriate treatment for this patient. Lorazepam is a benzodiazepine that can be used to treat alcohol withdrawal, but it would not be the first-line treatment for hepatic encephalopathy.
Based on the information provided, intravenous vitamin B1 (thiamine) therapy is the most likely treatment to improve this patient's cognition and movement disorder. Thiamine deficiency is commonly seen in individuals with alcoholism, and can lead to a range of neurological symptoms, including cognitive and movement disorders. Treatment with intravenous thiamine can help to improve these symptoms in individuals who are deficient in the vitamin.

The other treatment options listed are not likely to improve this patient's cognition and movement disorder. Intravenous vancomycin and cefepime therapy may be used to treat bacterial infections, but are not likely to improve symptoms related to thiamine deficiency. Oral lactulose is a medication that is used to treat constipation, and is not likely to have any effect on this patient's cognitive and movement disorders. Oral lorazepam is a sedative and anti-anxiety medication, and may be used to treat anxiety or agitation, but is not likely to improve cognitive or movement disorders related to thiamine deficiency.
4
A 45-year-old woman comes to the emergency department because of a 3-day history of progressive shortness of breath, cough productive of blood-tinged sputum, and fever. She also has a 3-month history of numbness of her left leg and tingling in her right arm. She has not had chills. She was diagnosed with asthma 6 months ago; it is well controlled with inhaled budesonide and albuterol as needed. She has no other history of serious illness and takes no other medications. She appears to be in mild respiratory distress. Temperature is 38.1°C (100.6°F), pulse is 105/min and regular, respirations are 24/min, and blood pressure is 145/95 mm Hg. Pulse oximetry on room air shows an oxygen saturation of 92%. Crackles are heard at the right lung base, and there are occasional diffuse wheezes. Cardiac examination shows no abnormalities. On neurologic examination, sensation to pinprick is decreased over the lateral aspect of the left lower extremity and right upper extremity. Muscle strength is 5/5 throughout. Babinski sign is absent. Deep tendon reflexes are 2+ and symmetric. Which of the following is the most appropriate next step in diagnosis?
(A) CT angiography of the chest
(B) Determination of serum angiotensin-converting enzyme activity
(C) Measurement of serum IgE concentration
(D) Mesenteric angiography
(E) Serum antiglomerular basement membrane assay
(F) Serum antineutrophil cytoplasmic autoantibody assay
(G) Sweat chloride testing
(F) Serum antineutrophil cytoplasmic autoantibody assay
Correct
The correct choice is (F) Serum antineutrophil cytoplasmic autoantibody assay.

Explanation:
This patient's symptoms and physical examination findings are consistent with a diagnosis of granulomatosis with polyangiitis (GPA), previously known as Wegener granulomatosis. GPA is a systemic vasculitis that primarily affects the respiratory tract and kidneys. It is characterized by the presence of antineutrophil cytoplasmic autoantibodies (ANCAs), specifically the proteinase 3 (PR3)-ANCA subtype. The most appropriate next step in diagnosis is to perform a serum antineutrophil cytoplasmic autoantibody assay to detect the presence of PR3-ANCA. This test has a high sensitivity and specificity for GPA and can help confirm the diagnosis. Other diagnostic tests, such as CT angiography of the chest, may be useful in evaluating the extent of disease involvement, but they are not the most appropriate initial step in diagnosis. Determination of serum angiotensin-converting enzyme activity is not indicated in this patient, as her symptoms and physical examination findings are not consistent with sarcoidosis. Measurement of serum IgE concentration is not indicated in this patient, as her symptoms and physical examination findings are not consistent with allergic bronchopulmonary aspergillosis. Mesenteric angiography is not indicated in this patient, as her symptoms and physical examination findings are not consistent with mesenteric ischemia. Serum antiglomerular basement membrane assay is not indicated in this patient, as her symptoms and physical examination findings are not consistent with anti-glomerular basement membrane (anti-GBM) disease. Sweat chloride testing is not indicated in this patient, as her symptoms and physical examination findings are not consistent with cystic fibrosis.
The most appropriate next step in diagnosis would be to perform further tests to determine the cause of the patient's symptoms. Based on the information provided, the most appropriate test would be a chest X-ray or computed tomography (CT) scan to assess the condition of the lungs and identify any abnormalities that may be contributing to the patient's symptoms. Options A, D, and G are not directly relevant to the patient's symptoms and do not appear to be the most appropriate next steps in diagnosis. Options B, C, E, and F may be relevant in some cases, but they are not typically used as initial tests in this situation and would not be the most appropriate next steps in diagnosis based on the information provided.
5
A 45-year-old woman comes to the emergency department because of a 3-day history of progressive shortness of breath, cough productive of blood-tinged sputum, and fever. She also has a 3-month history of numbness of her left leg and tingling in her right arm. She has not had chills. She was diagnosed with asthma 6 months ago; it is well controlled with inhaled budesonide and albuterol as needed. She has no other history of serious illness and takes no other medications. She appears to be in mild respiratory distress. Temperature is 38.1°C (100.6°F), pulse is 105/min and regular, respirations are 24/min, and blood pressure is 145/95 mm Hg. Pulse oximetry on room air shows an oxygen saturation of 92%. Crackles are heard at the right lung base, and there are occasional diffuse wheezes. Cardiac examination shows no abnormalities. On neurologic examination, sensation to pinprick is decreased over the lateral aspect of the left lower extremity and right upper extremity. Muscle strength is 5/5 throughout. Babinski sign is absent. Deep tendon reflexes are 2+ and symmetric. Which of the following is the most appropriate next step in diagnosis? Provide rationale for each choice on why it is appropriate or not appopriate as a course of action.
(A) CT angiography of the chest
(B) Determination of serum angiotensin-converting enzyme activity
(C) Measurement of serum IgE concentration
(D) Mesenteric angiography
(E) Serum antiglomerular basement membrane assay
(F) Serum antineutrophil cytoplasmic autoantibody assay
(G) Sweat chloride testing
(F) Serum antineutrophil cytoplasmic autoantibody assay
Correct
The correct choice is (F) Serum antineutrophil cytoplasmic autoantibody assay.

Rationale:
This patient presents with symptoms of shortness of breath, cough productive of blood-tinged sputum, and fever, which are suggestive of a pulmonary condition. The presence of crackles and wheezes on lung examination further supports a respiratory pathology. Additionally, the patient has a history of numbness and tingling in her extremities, which may indicate a neurological involvement.

Given these findings, the most appropriate next step in diagnosis is to perform a serum antineutrophil cytoplasmic autoantibody (ANCA) assay. ANCA-associated vasculitis, such as granulomatosis with polyangiitis (formerly known as Wegener granulomatosis), can present with respiratory symptoms, renal involvement, and neurological manifestations. The presence of ANCA antibodies, specifically anti-proteinase 3 (PR3) or anti-myeloperoxidase (MPO) antibodies, can help confirm the diagnosis of ANCA-associated vasculitis.

The other options listed are not the most appropriate next step in diagnosis for this patient:

(A) CT angiography of the chest: While a CT angiography can be useful in evaluating pulmonary embolism or other vascular abnormalities, it is not the most appropriate initial test in this patient. The clinical presentation and findings on examination do not strongly suggest a pulmonary embolism.

(B) Determination of serum angiotensin-converting enzyme (ACE) activity: Serum ACE levels can be elevated in conditions such as sarcoidosis, but the clinical presentation and findings in this patient do not strongly suggest sarcoidosis as the underlying cause.

(C) Measurement of serum IgE concentration: Elevated serum IgE levels can be seen in allergic conditions such as asthma, but the patient's symptoms and findings are not consistent with an acute exacerbation of asthma.

(D) Mesenteric angiography: Mesenteric angiography is not indicated in this patient as there are no symptoms or findings suggestive of mesenteric ischemia or other abdominal vascular abnormalities.

(E) Serum antiglomerular basement membrane (anti-GBM) assay: Anti-GBM antibodies are associated with Goodpasture syndrome, which typically presents with pulmonary hemorrhage and renal involvement. While this patient has respiratory symptoms, the absence of renal symptoms makes Goodpasture syndrome less likely.

(G) Sweat chloride testing: Sweat chloride testing is used to diagnose cystic fibrosis, which typically presents with respiratory symptoms and recurrent infections. However, the absence of other characteristic features of cystic fibrosis, such as gastrointestinal symptoms or failure to thrive, makes this diagnosis less likely in this patient.
The most appropriate next step in diagnosis would be to perform further tests to determine the cause of the patient's symptoms. The tests that would be most appropriate would be imaging tests such as chest X-rays or computed tomography (CT) scans to assess the condition of the lungs and identify any abnormalities that may be contributing to the patient's symptoms. These tests would provide valuable information about the condition of the patient's lungs and allow the healthcare provider to make more informed decisions about the appropriate course of treatment.

Option A, CT angiography of the chest, is not the most appropriate next step in diagnosis because it is not typically used as an initial test in this situation and would not provide information relevant to the patient's symptoms.

Option B, determination of serum angiotensin-converting enzyme activity, is not the most appropriate next step in diagnosis because it is not typically used as an initial test in this situation and would not provide information relevant to the patient's symptoms.

Option C, measurement of serum IgE concentration, may be relevant in some cases but is not typically used as an initial test in this situation and would not provide information relevant to the patient's symptoms.

Option D, mesenteric angiography, is not the most appropriate next step in diagnosis because it is not typically used as an initial test in this situation and would not provide information relevant to the patient's symptoms.

Option E, serum antiglomerular basement membrane assay, may be relevant in some cases but is not typically used as an initial test in this situation and would not provide information relevant to the patient's symptoms.

Option F, serum antineutrophil cytoplasmic autoantibody assay, may be relevant in some cases but is not typically used as an initial test in this situation and would not provide information relevant to the patient's symptoms.

Option G, sweat chloride testing, is not the most appropriate next step in diagnosis because it is not typically used as an initial test in this situation and would not provide information relevant to the patient's symptoms.
6
An 87-year-old woman, gravida 6, para 6, is brought to the emergency department because of a 24-hour history of severe abdominal pain and an inability to void. She notes that for the past 2 months, she has had difficulty emptying her bladder, especially at the end of the day. Twenty years ago, she underwent mastectomy for breast cancer; she has been cancer-free since then. She has a 10-year history of osteoporosis. All of her children were delivered vaginally without complications. Her only medication is alendronate. Examination shows the cervix protruding 7.5 cm from the vagina. After the cervix is replaced into the vagina, a urinary catheter drains 700 mL of urine. Urinalysis shows no abnormalities. Which of the following is the most likely diagnosis?
(A) Overactive bladder
(B) Overflow incontinence
(C) Stress incontinence
(D) Urinary retention
(E) Urinary tract infection
(D) Urinary retentionCorrect
The correct answer is (D) Urinary retention.

Explanation:
In this case, the patient presents with severe abdominal pain and an inability to void, along with a history of difficulty emptying her bladder for the past 2 months. The examination shows a protruding cervix and a large amount of urine is drained after the cervix is replaced into the vagina. These findings are consistent with urinary retention.

Urinary retention is the inability to completely empty the bladder. It can be caused by various factors, including obstruction, nerve damage, medications, and weakened bladder muscles. In this case, the protruding cervix may be causing obstruction and preventing the patient from emptying her bladder fully.

The presence of a large amount of urine drained after the cervix is replaced into the vagina suggests that the bladder was significantly distended. This is a characteristic finding in urinary retention.

The other options, such as overactive bladder, overflow incontinence, stress incontinence, and urinary tract infection, do not fully explain the patient's symptoms and findings. Overactive bladder is characterized by a sudden urge to urinate and may be associated with urinary frequency and urgency. Overflow incontinence is characterized by a constant dribbling of urine due to an overfilled bladder. Stress incontinence is the leakage of urine during physical activity or exertion. Urinary tract infection may cause urinary symptoms, but the absence of abnormalities in the urinalysis makes it less likely in this case.
Based on the information provided, the most likely diagnosis is urinary retention, which is the inability to completely empty the bladder. This can cause symptoms like difficulty emptying the bladder, abdominal pain, and the protrusion of the cervix from the vagina, as described in the case. The patient's history of vaginal deliveries and breast cancer surgery may have contributed to the development of this condition. A urinary catheter can be used to drain the bladder and relieve symptoms, and further treatment may be necessary to address the underlying cause of the urinary retention.
7
An 87-year-old woman, gravida 6, para 6, is brought to the emergency department because of a 24-hour history of severe abdominal pain and an inability to void. She notes that for the past 2 months, she has had difficulty emptying her bladder, especially at the end of the day. Twenty years ago, she underwent mastectomy for breast cancer; she has been cancer-free since then. She has a 10-year history of osteoporosis. All of her children were delivered vaginally without complications. Her only medication is alendronate. Examination shows the cervix protruding 7.5 cm from the vagina. After the cervix is replaced into the vagina, a urinary catheter drains 700 mL of urine. Urinalysis shows no abnormalities. Which of the following is the most likely diagnosis? Can you provide your rationale on each choice provided?
(A) Overactive bladder
(B) Overflow incontinence
(C) Stress incontinence
(D) Urinary retention
(E) Urinary tract infection
(D) Urinary retentionCorrect
The correct answer is (D) Urinary retention.

Rationale for each choice:

(A) Overactive bladder: Overactive bladder is characterized by a sudden and uncontrollable urge to urinate, often accompanied by frequency and nocturia. It is not typically associated with severe abdominal pain or an inability to void. Additionally, the patient's symptoms of difficulty emptying her bladder and the large amount of urine drained after replacing the cervix suggest a different diagnosis.

(B) Overflow incontinence: Overflow incontinence occurs when the bladder is unable to empty completely, leading to frequent dribbling or leakage of urine. It is often associated with urinary outlet obstruction, such as from pelvic organ prolapse or fibroids. However, in this case, the patient's symptoms of severe abdominal pain and inability to void suggest a more acute and severe condition than overflow incontinence.

(C) Stress incontinence: Stress incontinence is the involuntary leakage of urine during activities that increase intra-abdominal pressure, such as coughing, sneezing, or lifting. It is not typically associated with severe abdominal pain or an inability to void. The patient's symptoms of difficulty emptying her bladder and the large amount of urine drained after replacing the cervix suggest a different diagnosis.

(D) Urinary retention: Urinary retention is the inability to completely empty the bladder. It can be caused by various factors, including urinary outlet obstruction, neurogenic dysfunction, or medications. In this case, the patient's symptoms of difficulty emptying her bladder, especially at the end of the day, along with the large amount of urine drained after replacing the cervix, suggest urinary retention. The protruding cervix may be causing mechanical obstruction of the urethra, leading to the inability to void.

(E) Urinary tract infection: While urinary tract infections (UTIs) can cause urinary symptoms, such as frequency, urgency, and dysuria, they are not typically associated with severe abdominal pain or an inability to void. The absence of abnormalities on urinalysis also makes UTI less likely as the primary cause of the patient's symptoms.
Based on the information provided, the most likely diagnosis is urinary retention, which is the inability to completely empty the bladder. This can cause symptoms like difficulty emptying the bladder, abdominal pain, and the protrusion of the cervix from the vagina, as described in the case. The patient's history of vaginal deliveries and breast cancer surgery may have contributed to the development of this condition. A urinary catheter can be used to drain the bladder and relieve symptoms, and further treatment may be necessary to address the underlying cause of the urinary retention.

Here is a brief explanation of the other options and why they are less likely:

A) Overactive bladder is a condition where the bladder muscles contract involuntarily, causing a sudden urge to urinate. This condition does not typically cause difficulty emptying the bladder or protrusion of the cervix.

B) Overflow incontinence is a type of incontinence where the bladder is unable to completely empty, resulting in frequent urination or leakage. This condition is similar to urinary retention, but urinary retention is typically more severe and is characterized by a complete inability to urinate, while overflow incontinence is characterized by frequent urination or leakage.

C) Stress incontinence is a type of incontinence where urine leakage occurs during activities that put pressure on the bladder, such as coughing, sneezing, or exercise. This condition is not typically associated with difficulty emptying the bladder or protrusion of the cervix.

D) Urinary retention is the inability to completely empty the bladder, which can cause symptoms like difficulty urinating, abdominal pain, and protrusion of the cervix. This is the most likely diagnosis based on the information provided.

E) A urinary tract infection (UTI) is an infection in any part of the urinary system, including the kidneys, ureters, bladder, or urethra. UTIs can cause symptoms like frequent urination, pain or burning with urination, and cloudy or foul-smelling urine. However, UTIs are not typically associated with difficulty emptying the bladder or protrusion of the cervix.
8
A 25-year-old woman is admitted to the hospital because of a 2-day history of fever, dizziness, light-headedness, chills, and shaking. She has hyperthyroidism, gastroesophageal reflux disease, and asthma. Her medications are oral methimazole and omeprazole, and inhaled fluticasone, and she has been on the same dosages for 1 year. She is in the military reserve and will be deployed overseas next week. She began atovaquone-proguanil therapy 1 week ago and received the hepatitis A and yellow fever vaccines 2 weeks ago. She is not sexually active. She does not smoke cigarettes or drink alcohol. She appears ill but is not in acute distress. Temperature is 38.6°C (101.5°F), pulse is 110/min, respirations are 22/min, and blood pressure is 90/60 mm Hg. No wheezing is heard on auscultation. Cardiac examination shows no abnormalities. Laboratory studies show:
Hemoglobin = 13.8 g/dL
Leukocyte count = 1100/mm³
Segmented neutrophils = 10%
Eosinophils = 10%
Lymphocytes = 40%
Monocytes = 40%
Platelet count= 320,000/mm³
Platelet count 320,000/mm³

Results of other laboratory studies are within the reference ranges. Which of the following is the most likely cause of this patient’s hematologic abnormality? Please choose the best option among the choices below.
(A) Atovaquone-proguanil
(B) Fluticasone
(C) Hepatitis A vaccine
(D) Methimazole
(E) Omeprazole
(F) Yellow fever vaccine
(D) MethimazoleCorrect
The correct answer is (D) Methimazole.

Explanation:
The patient's laboratory findings show a leukocyte count of 1100/mm³, with a differential showing 10% segmented neutrophils, 10% eosinophils, 40% lymphocytes, and 40% monocytes. This pattern of leukopenia with relative lymphocytosis and monocytosis is consistent with a drug-induced agranulocytosis. Methimazole, an antithyroid medication used to treat hyperthyroidism, is a known cause of agranulocytosis. Agranulocytosis is a potentially life-threatening condition characterized by a severe decrease in neutrophils, which are the body's primary defense against bacterial infections. The symptoms of fever, dizziness, light-headedness, chills, and shaking are consistent with an infection, which can occur in the setting of neutropenia. The patient's recent initiation of atovaquone-proguanil therapy and receipt of hepatitis A and yellow fever vaccines are less likely to be the cause of her hematologic abnormality. Fluticasone, an inhaled corticosteroid used to treat asthma, is not known to cause agranulocytosis. Omeprazole, a proton pump inhibitor used to treat gastroesophageal reflux disease, is also not associated with agranulocytosis.
Based on the information provided, the most likely cause of the patient's hematologic abnormality is an infection, rather than any of the medications or vaccines she is currently taking. It is not possible to determine the specific cause of the infection without further information, such as results from additional laboratory tests. Therefore, the correct answer is none of the options listed.*
9
A 25-year-old woman is admitted to the hospital because of a 2-day history of fever, dizziness, light-headedness, chills, and shaking. She has hyperthyroidism, gastroesophageal reflux disease, and asthma. Her medications are oral methimazole and omeprazole, and inhaled fluticasone, and she has been on the same dosages for 1 year. She is in the military reserve and will be deployed overseas next week. She began atovaquone-proguanil therapy 1 week ago and received the hepatitis A and yellow fever vaccines 2 weeks ago. She is not sexually active. She does not smoke cigarettes or drink alcohol. She appears ill but is not in acute distress. Temperature is 38.6°C (101.5°F), pulse is 110/min, respirations are 22/min, and blood pressure is 90/60 mm Hg. No wheezing is heard on auscultation. Cardiac examination shows no abnormalities. Laboratory studies show:
Hemoglobin = 13.8 g/dL
Leukocyte count = 1100/mm³
Segmented neutrophils = 10%
Eosinophils = 10%
Lymphocytes = 40%
Monocytes = 40%
Platelet count= 320,000/mm³

Results of other laboratory studies are within the reference ranges. Which of the following is the most likely cause of this patient’s hematologic abnormality? Why are the other choices not likely the answer?
(A) Atovaquone-proguanil
(B) Fluticasone
(C) Hepatitis A vaccine
(D) Methimazole
(E) Omeprazole
(F) Yellow fever vaccine
(D) MethimazoleCorrect
The correct answer is (D) Methimazole.

The patient's hematologic abnormality is characterized by a low leukocyte count and a relative lymphocytosis. Methimazole, an antithyroid medication used to treat hyperthyroidism, is known to cause agranulocytosis, which is a severe decrease in the number of granulocytes (neutrophils, eosinophils, and basophils). This can lead to a low leukocyte count and an increase in the percentage of lymphocytes, as seen in this patient. Agranulocytosis is a rare but serious side effect of methimazole therapy and can present with symptoms such as fever, chills, and malaise, which are consistent with the patient's presentation.

The other choices are less likely to be the cause of the patient's hematologic abnormality:

(A) Atovaquone-proguanil: Atovaquone-proguanil is an antimalarial medication and is not known to cause agranulocytosis or affect the leukocyte count.

(B) Fluticasone: Fluticasone is an inhaled corticosteroid used to treat asthma. While corticosteroids can cause leukocytosis, they do not typically cause a decrease in the number of granulocytes or a relative lymphocytosis.

(C) Hepatitis A vaccine: The hepatitis A vaccine is not known to cause hematologic abnormalities.

(E) Omeprazole: Omeprazole is a proton pump inhibitor used to treat gastroesophageal reflux disease. It is not known to cause agranulocytosis or affect the leukocyte count.

(F) Yellow fever vaccine: The yellow fever vaccine is not known to cause hematologic abnormalities.

Therefore, the most likely cause of this patient's hematologic abnormality is the use of methimazole.
As mentioned previously, it is not possible to determine the cause of the patient's hematologic abnormality based on the information provided. Without further information, it is not possible to determine which of the medications or vaccines listed in the question is the most likely cause of the patient's hematologic abnormality. It is also not possible to determine why the other choices are not likely the answer without more information.
10
A 16-year-old boy is brought to the emergency department because of a 2-day history of fever, nausea, vomiting, headache, chills, and fatigue. He has not had any sick contacts. He underwent splenectomy for traumatic injury at the age of 13 years. He has no other history of serious illness and takes no medications. He appears ill. His temperature is 39.2°C (102.5°F), pulse is 130/min, respirations are 14/min, and blood pressure is 110/60 mm Hg. On pulmonary examination, scattered crackles are heard bilaterally. Abdominal examination shows a well-healed midline scar and mild, diffuse tenderness to palpation. Which of the following is the most appropriate next step in management?
(A) Antibiotic therapy
(B) Antiemetic therapy
(C) CT scan of the chest
(D) X-ray of the abdomen
(E) Reassurance
(A) Antibiotic therapyCorrect
The correct choice is (A) Antibiotic therapy.

Explanation:
This patient's symptoms, including fever, chills, headache, and fatigue, along with the physical examination findings of crackles on pulmonary examination and mild, diffuse tenderness on abdominal examination, are concerning for a possible infection. Given the patient's history of splenectomy, he is at increased risk for infections, particularly from encapsulated organisms such as Streptococcus pneumoniae, Haemophilus influenzae, and Neisseria meningitidis. Therefore, the most appropriate next step in management would be to initiate antibiotic therapy to cover these potential pathogens. This will help to prevent further complications and improve the patient's condition.
Based on the information provided, the most appropriate next step in management would be to order diagnostic tests, such as a CT scan of the chest and an x-ray of the abdomen, to help determine the cause of the patient's symptoms. These tests can provide valuable information about the patient's condition and guide further treatment. It may also be appropriate to start the patient on intravenous fluids to help rehydrate him and control his nausea and vomiting with antiemetic therapy. Antibiotic therapy may be necessary if an infection is suspected, but this should be determined after the results of the diagnostic tests are known. Reassurance alone is not likely to be sufficient to treat the patient's symptoms.



11
A 16-year-old boy is brought to the emergency department because of a 2-day history of fever, nausea, vomiting, headache, chills, and fatigue. He has not had any sick contacts. He underwent splenectomy for traumatic injury at the age of 13 years. He has no other history of serious illness and takes no medications. He appears ill. His temperature is 39.2°C (102.5°F), pulse is 130/min, respirations are 14/min, and blood pressure is 110/60 mm Hg. On pulmonary examination, scattered crackles are heard bilaterally. Abdominal examination shows a well-healed midline scar and mild, diffuse tenderness to palpation. Which of the following is the most appropriate next step in management? What is the rational for your answer and the other choices?
(A) Antibiotic therapy
(B) Antiemetic therapy
(C) CT scan of the chest
(D) X-ray of the abdomen
(E) Reassurance
(A) Antibiotic therapyCorrect
The correct choice is (A) Antibiotic therapy.

Explanation:
Based on the patient's history and physical examination findings, the most likely diagnosis is pneumonia. The patient presents with fever, respiratory symptoms (cough, crackles on pulmonary examination), and constitutional symptoms (nausea, vomiting, headache, chills, fatigue). The patient's history of splenectomy puts him at increased risk for infections, including encapsulated bacteria such as Streptococcus pneumoniae. Therefore, the most appropriate next step in management is to initiate antibiotic therapy to cover for potential bacterial pneumonia.

Choice (B) Antiemetic therapy is not the most appropriate next step in management because the patient's nausea and vomiting are likely secondary to his underlying illness (pneumonia) and will likely resolve with treatment of the underlying infection.

Choice (C) CT scan of the chest may be useful in certain cases of pneumonia, particularly if there is concern for complications such as abscess or empyema. However, in this case, the patient's clinical presentation is consistent with uncomplicated pneumonia, and therefore, a CT scan is not the most appropriate next step in management.

Choice (D) X-ray of the abdomen is not the most appropriate next step in management because the patient's abdominal tenderness is likely secondary to his underlying illness (pneumonia) and will likely resolve with treatment of the underlying infection. Additionally, the patient's respiratory symptoms and crackles on pulmonary examination are more concerning for a respiratory infection rather than an abdominal pathology.

Choice (E) Reassurance is not the most appropriate next step in management because the patient's symptoms and physical examination findings are concerning for a serious infection (pneumonia) that requires prompt treatment with antibiotics. Reassurance alone would not address the underlying infection.
Based on the information provided, the most appropriate next step in management would be to order diagnostic tests, such as a CT scan of the chest and an x-ray of the abdomen, to help determine the cause of the patient's symptoms. These tests can provide valuable information about the patient's condition and guide further treatment.

The rationale for this answer is that the patient has a fever and other symptoms that suggest a possible infection or other underlying medical condition. In order to determine the cause of the symptoms and provide appropriate treatment, it is necessary to perform diagnostic tests. A CT scan of the chest and an x-ray of the abdomen can help identify any abnormalities in the patient's chest or abdominal organs that may be causing his symptoms.

Other possible treatments, such as antibiotic therapy and antiemetic therapy, may be necessary if an infection or nausea and vomiting are suspected, but these should be determined after the results of the diagnostic tests are known. Reassurance alone is not likely to be sufficient to treat the patient's symptoms, as he appears to be quite ill and may require more aggressive treatment.
12
A previously healthy 29-year-old woman is admitted to the hospital because of a 1-day history of moderate headache, fever, and lethargy. Two days ago, she underwent wisdom tooth extraction. She takes no medications. She cannot remember her vaccination history. She does not smoke cigarettes, drink alcohol, or use illicit drugs. She is a college student, lives in an apartment complex with other students, and eats all her meals in the campus dining hall. She says her study partner had a “really bad fever” during the past week. The patient appears sleepy. Temperature is 40.0°C (104.0°F), pulse is 140/min, and blood pressure is 90/40 mm Hg. Examination shows a diffuse purpuric rash over the trunk and all extremities. There is nuchal rigidity. Lungs are clear to auscultation. Heart sounds are normal. Neurologic examination shows no other focal findings. Which of the following is most likely to have prevented this patient’s current condition?
(A) Adherence to immunization guidelines
(B) Avoidance of eating meals in the campus dining hall
(C) Consistent hand washing
(D) Isolation of other students on campus with febrile illness
(E) Prophylactic antibiotic therapy
(A) Adherence to immunization guidelines
Correct
The correct choice is (A) Adherence to immunization guidelines.

Explanation:
Based on the patient's symptoms and physical examination findings, she is most likely experiencing meningococcal meningitis, which is a serious bacterial infection that can cause severe illness and death if not treated promptly. The presence of a diffuse purpuric rash, nuchal rigidity, and high fever are all consistent with this diagnosis.

Meningococcal meningitis is caused by the bacteria Neisseria meningitidis, which can be prevented by vaccination. Adherence to immunization guidelines, including receiving the meningococcal vaccine, is the most effective way to prevent this condition.

Avoidance of eating meals in the campus dining hall (B) would not prevent meningococcal meningitis, as the infection is primarily spread through respiratory droplets and close contact with an infected individual.

Consistent hand washing (C) is important for preventing the spread of many infections, but it would not specifically prevent meningococcal meningitis.

Isolation of other students on campus with febrile illness (D) would help prevent the spread of certain infections, but it would not specifically prevent meningococcal meningitis.

Prophylactic antibiotic therapy (E) is recommended for close contacts of individuals with meningococcal meningitis to prevent the spread of the infection, but it is not the primary method of prevention. Vaccination is the most effective way to prevent meningococcal meningitis.
Based on the information provided, the most likely measure that could have prevented this patient's current condition is adherence to immunization guidelines (Option A). Receiving recommended vaccines can help protect individuals from certain infections, including those that may arise following a wisdom tooth extraction.
13
A previously healthy 29-year-old woman is admitted to the hospital because of a 1-day history of moderate headache, fever, and lethargy. Two days ago, she underwent wisdom tooth extraction. She takes no medications. She cannot remember her vaccination history. She does not smoke cigarettes, drink alcohol, or use illicit drugs. She is a college student, lives in an apartment complex with other students, and eats all her meals in the campus dining hall. She says her study partner had a “really bad fever” during the past week. The patient appears sleepy. Temperature is 40.0°C (104.0°F), pulse is 140/min, and blood pressure is 90/40 mm Hg. Examination shows a diffuse purpuric rash over the trunk and all extremities. There is nuchal rigidity. Lungs are clear to auscultation. Heart sounds are normal. Neurologic examination shows no other focal findings. Which of the following is most likely to have prevented this patient’s current condition? Why is the answer the best choice and not the others?
(A) Adherence to immunization guidelines
(B) Avoidance of eating meals in the campus dining hall
(C) Consistent hand washing
(D) Isolation of other students on campus with febrile illness
(E) Prophylactic antibiotic therapy
(A) Adherence to immunization guidelines
Correct
The correct choice is (A) Adherence to immunization guidelines.

Explanation:
The patient's symptoms and physical examination findings are consistent with meningococcal meningitis, which is a serious bacterial infection caused by Neisseria meningitidis. Meningococcal meningitis can present with symptoms such as headache, fever, lethargy, and a purpuric rash. The presence of nuchal rigidity further supports the diagnosis.

The most effective way to prevent meningococcal meningitis is through vaccination. Adherence to immunization guidelines ensures that individuals receive the necessary vaccines to protect against meningococcal infection. In this case, the patient cannot remember her vaccination history, which suggests that she may not have received the meningococcal vaccine.

Avoidance of eating meals in the campus dining hall (B) and consistent hand washing (C) are important measures to prevent the spread of infectious diseases, but they would not specifically prevent meningococcal meningitis.

Isolation of other students on campus with febrile illness (D) is a measure to prevent the spread of infectious diseases, but it would not prevent the patient from developing meningococcal meningitis.

Prophylactic antibiotic therapy (E) is recommended for close contacts of individuals with meningococcal meningitis to prevent secondary cases, but it would not prevent the initial infection in this patient.

Therefore, adherence to immunization guidelines (A) is the most likely factor that could have prevented this patient's current condition.
Based on the information provided, the most likely measure that could have prevented this patient's current condition is adherence to immunization guidelines (Option A). This is because receiving recommended vaccines can help protect individuals from certain infections, including those that may arise following a wisdom tooth extraction. The other options, such as avoidance of eating meals in the campus dining hall (Option B), consistent hand washing (Option C), isolation of other students on campus with febrile illness (Option D), and prophylactic antibiotic therapy (Option E), may not necessarily have prevented the patient's current condition. For example, avoiding the campus dining hall may not prevent the spread of infection if the patient was already infected before she started experiencing symptoms. Similarly, consistent hand washing and isolating students with febrile illness may not prevent the spread of infection if the patient was already infected before these measures were implemented. Lastly, prophylactic antibiotic therapy may not be effective if the patient is not infected with a bacteria that is susceptible to the prescribed antibiotics.
14
A 27-year-old primigravid woman comes to the physician for her first prenatal visit 10 weeks after a home pregnancy test was positive. She feels well, and her pregnancy has been uncomplicated. She previously used an oral contraceptive but discontinued it 3 months ago. She has no history of sexually transmitted diseases. She is sexually active in a monogamous relationship with a male partner who is the father. She has had five lifetime sexual partners. She has a pet cat. Examination shows a uterus consistent in size with a 10-week gestation. Testing for which of the following infectious conditions is the most appropriate recommendation for this patient?
(A) Bacterial vaginosis
(B) Cytomegalovirus
(C) Epstein-Barr virus
(D) HIV
(E) Human papillomavirus
(D) HIVCorrect
The correct choice is (D) HIV.

Explanation:
Testing for HIV is the most appropriate recommendation for this patient because she has had multiple sexual partners in her lifetime, which puts her at risk for sexually transmitted infections (STIs). Additionally, it is important to screen for HIV during pregnancy to prevent mother-to-child transmission of the virus. HIV can be transmitted through sexual contact, sharing needles, or from mother to child during pregnancy, childbirth, or breastfeeding. It is important to identify HIV infection early in pregnancy so that appropriate interventions can be implemented to reduce the risk of transmission to the fetus.
Based on the information provided, it is most appropriate to recommend testing for HIV in this patient. HIV is a sexually transmitted infection (STI) that can be transmitted from an infected mother to her baby during pregnancy, childbirth, or breastfeeding. It is recommended that pregnant women be tested for HIV as early as possible in pregnancy to allow for early treatment and prevent transmission to the baby. Other STIs, such as bacterial vaginosis, cytomegalovirus, Epstein-Barr virus, and human papillomavirus, can also be transmitted from mother to baby during pregnancy, but the risk of transmission is generally lower than with HIV and testing for these infections is not routinely recommended in pregnancy.
15
A 27-year-old primigravid woman comes to the physician for her first prenatal visit 10 weeks after a home pregnancy test was positive. She feels well, and her pregnancy has been uncomplicated. She previously used an oral contraceptive but discontinued it 3 months ago. She has no history of sexually transmitted diseases. She is sexually active in a monogamous relationship with a male partner who is the father. She has had five lifetime sexual partners. She has a pet cat. Examination shows a uterus consistent in size with a 10-week gestation. Testing for which of the following infectious conditions is the most appropriate recommendation for this patient? Provide rationale for each choice below.
(A) Bacterial vaginosis
(B) Cytomegalovirus
(C) Epstein-Barr virus
(D) HIV
(E) Human papillomavirus
(D) HIVCorrect
The correct choice is (D) HIV.

Rationale:
In this scenario, the most appropriate recommendation for testing is HIV. This is because the patient has had multiple sexual partners in her lifetime, which puts her at risk for sexually transmitted infections (STIs), including HIV. Additionally, the fact that she previously used oral contraceptives and discontinued them 3 months ago suggests that she may have been sexually active during that time, further increasing her risk for STIs.

HIV is a viral infection that is transmitted through sexual contact, sharing needles, or from mother to child during pregnancy, childbirth, or breastfeeding. It is important to identify HIV infection early in pregnancy because appropriate interventions can be implemented to reduce the risk of transmission to the fetus.

Bacterial vaginosis (A) is a common vaginal infection caused by an imbalance of bacteria in the vagina. It is not typically associated with complications in pregnancy and does not require routine testing in asymptomatic pregnant women.

Cytomegalovirus (B) is a common viral infection that can be transmitted through close contact with bodily fluids, such as saliva, urine, and breast milk. While it can cause complications in pregnancy, routine testing for cytomegalovirus is not recommended in asymptomatic pregnant women.

Epstein-Barr virus (C) is a common viral infection that causes infectious mononucleosis. It is primarily transmitted through saliva and is not typically associated with complications in pregnancy. Routine testing for Epstein-Barr virus is not recommended in asymptomatic pregnant women.

Human papillomavirus (E) is a sexually transmitted infection that can cause genital warts and is associated with an increased risk of cervical cancer. While routine screening for cervical cancer is recommended in women, testing for human papillomavirus is not routinely performed in asymptomatic pregnant women.
Based on the information provided, it is most appropriate to recommend testing for HIV in this patient. HIV is a sexually transmitted infection (STI) that can be transmitted from an infected mother to her baby during pregnancy, childbirth, or breastfeeding. It is recommended that pregnant women be tested for HIV as early as possible in pregnancy to allow for early treatment and prevent transmission to the baby.

Testing for other STIs, such as bacterial vaginosis, cytomegalovirus, Epstein-Barr virus, and human papillomavirus, is generally not recommended in this patient. Bacterial vaginosis is an infection caused by an overgrowth of certain types of bacteria in the vagina. It is not typically transmitted from mother to baby during pregnancy and is not routinely tested for in pregnancy.

Cytomegalovirus is a common viral infection that is usually harmless in healthy individuals. However, it can be transmitted from mother to baby during pregnancy and can cause serious illness in a developing fetus or newborn. Testing for cytomegalovirus is not routinely recommended in pregnancy, but may be considered in certain cases, such as if the mother has a history of cytomegalovirus infection or if the baby shows signs of infection after birth.

Epstein-Barr virus is a virus that causes infectious mononucleosis. It is not typically transmitted from mother to baby during pregnancy and is not routinely tested for in pregnancy.

Human papillomavirus (HPV) is a common STI that can cause genital warts and certain types of cancer. HPV is not typically transmitted from mother to baby during pregnancy and is not routinely tested for in pregnancy. However, pregnant women who have abnormal Pap test results or who have a history of HPV infection may be at increased risk of complications during pregnancy and may require further testing or treatment.
16
An investigator would like to decrease the incidence of type 2 diabetes mellitus among adult patients with risk factors including obesity, decreased physical activity, and family history of diabetes mellitus. A total of 200 men and women aged 30 to 64 years will be enrolled in a regular exercise program for 6 months. A nurse reviews the details of the study with each participant, and each participant signs an informed consent form. One month after the study begins, one of the participants asks to drop out of the study. Which of the following represents the most appropriate next step by this study participant?
(A) Complete the study because she signed the consent form
(B) Discuss withdrawal with the institutional review board
(C) Negotiate her withdrawal with the principal investigator
(D) Wait to withdraw until after the investigator consults with an ethics team
(E) Withdraw from the study now
(E) Withdraw from the study nowCorrect
The correct choice is (E) Withdraw from the study now.

Explanation:
When a participant in a study decides to withdraw, it is their right to do so at any time. Informed consent forms are not binding contracts and participants have the autonomy to withdraw from the study if they wish. It is important to respect the participant's decision and allow them to withdraw without any further obligations or negotiations. Therefore, the most appropriate next step for this study participant is to withdraw from the study now.
The most appropriate next step by this study participant would be to discuss their decision to withdraw from the study with the investigator or nurse who is overseeing the study. This is represented by option (C) - "Negotiate her withdrawal with the principal investigator". It is important for the participant to have a conversation with the investigator or nurse so that they can provide information about the potential consequences of withdrawing from the study and help the participant make an informed decision. The participant should also be asked to provide a reason for wanting to withdraw, so that the investigator can determine if there are any potential safety concerns or other issues that need to be addressed. Ultimately, the decision to withdraw from the study is up to the participant, but it is important for them to discuss their decision with the investigator or nurse before making a final decision.
17
An investigator would like to decrease the incidence of type 2 diabetes mellitus among adult patients with risk factors including obesity, decreased physical activity, and family history of diabetes mellitus. A total of 200 men and women aged 30 to 64 years will be enrolled in a regular exercise program for 6 months. A nurse reviews the details of the study with each participant, and each participant signs an informed consent form. One month after the study begins, one of the participants asks to drop out of the study. Which of the following represents the most appropriate next step by this study participant? Why are the other choices not the best fit?
(A) Complete the study because she signed the consent form
(B) Discuss withdrawal with the institutional review board
(C) Negotiate her withdrawal with the principal investigator
(D) Wait to withdraw until after the investigator consults with an ethics team
(E) Withdraw from the study now
(E) Withdraw from the study nowCorrect
The correct choice is (E) Withdraw from the study now.

Explanation:
When a participant in a research study decides to withdraw, it is their right to do so at any time. Informed consent forms are not binding contracts and participants have the autonomy to withdraw from the study if they wish. It is important to respect the participant's decision and allow them to withdraw without any further obligations or requirements.

The other choices are not the best fit for the following reasons:

(A) Complete the study because she signed the consent form: Signing the consent form does not obligate the participant to complete the study. Participants have the right to withdraw at any time, even after signing the consent form.

(B) Discuss withdrawal with the institutional review board: While the institutional review board (IRB) oversees the ethical conduct of research studies, it is not necessary to involve them in every participant's decision to withdraw. The participant can withdraw without involving the IRB.

(C) Negotiate her withdrawal with the principal investigator: There is no need for negotiation when a participant decides to withdraw from a study. It is their right to do so without any negotiation or further discussion.

(D) Wait to withdraw until after the investigator consults with an ethics team: It is not necessary to involve an ethics team in every participant's decision to withdraw. The participant can withdraw immediately without waiting for any further consultation.
The most appropriate next step by this study participant would be to discuss their decision to withdraw from the study with the investigator or nurse who is overseeing the study. This is represented by option (C) - "Negotiate her withdrawal with the principal investigator". The other choices are not the best fit for the following reasons:

Option (A) - "Complete the study because she signed the consent form" - is not the best fit because the participant has the right to withdraw from the study at any time, regardless of whether they signed a consent form.
Option (B) - "Discuss withdrawal with the institutional review board" - is not the best fit because the institutional review board is typically responsible for reviewing and approving research studies, not individual decisions to withdraw from a study.
Option (D) - "Wait to withdraw until after the investigator consults with an ethics team" - is not the best fit because the participant should be able to make their own decision about whether to withdraw from the study, without having to wait for the investigator to consult with an ethics team.
Option (E) - "Withdraw from the study now" - is not the best fit because the participant should discuss their decision to withdraw with the investigator or nurse before making a final decision. This can help ensure that the participant is fully informed about the potential consequences of their decision and that any potential safety concerns are addressed.
18
A 70-year-old man comes to the clinic because of a 3-day history of fatigue, dark urine, decreased urine output, mild pain with urination, and a low-grade fever. He has gout, hypertension, and osteoarthritis. His medications are allopurinol, hydrochlorothiazide, and acetaminophen. Temperature is 37.9°C (100.2°F), pulse is 88/min, respirations are 24/min, and blood pressure is 160/95 mm Hg. Pulse oximetry on room air shows an oxygen saturation of 97%. Jugular venous pressure is 7 cm H2O (N=5–9). Cardiopulmonary examination shows no abnormalities. Which of the following is the most appropriate next step in management?
(A) Discontinuation of acetaminophen therapy
(B) Intravenous administration of fluids
(C) Measurement of postvoid residual volume
(D) Oral trimethoprim-sulfamethoxazole therapy
(E) Renal biopsy
(C) Measurement of postvoid residual volume
Correct
The correct choice is (C) Measurement of postvoid residual volume.

Explanation:
The patient's symptoms of fatigue, dark urine, decreased urine output, mild pain with urination, and low-grade fever are concerning for acute kidney injury (AKI). AKI can be caused by various factors, including medication-induced nephrotoxicity. In this case, the patient is taking multiple medications, including hydrochlorothiazide and acetaminophen, which can both cause renal impairment. Therefore, it is important to assess the postvoid residual volume to evaluate for urinary obstruction as a potential cause of AKI.

Measurement of postvoid residual volume can be done using ultrasound or bladder catheterization. A high postvoid residual volume would suggest urinary obstruction as the cause of the patient's symptoms, which would require further evaluation and management. If the postvoid residual volume is normal, other causes of AKI should be considered, such as medication-induced nephrotoxicity or intrinsic renal disease.

Discontinuation of acetaminophen therapy (choice A) may be considered if acetaminophen-induced nephrotoxicity is suspected, but it is important to first evaluate for urinary obstruction as a potential cause of AKI.

Intravenous administration of fluids (choice B) may be indicated in the management of AKI, but it is important to first evaluate for urinary obstruction as a potential cause of AKI.

Oral trimethoprim-sulfamethoxazole therapy (choice D) is not indicated in this patient as there is no evidence of a urinary tract infection.

Renal biopsy (choice E) is not indicated at this time as there is no evidence of severe renal impairment or indications for renal biopsy based on the patient's presentation.
Based on the information provided, the most appropriate next step in management would be to order lab tests to evaluate the cause of the patient's symptoms. This could include tests to assess renal function, such as a urinalysis, serum creatinine, and estimated glomerular filtration rate (eGFR). Other tests might include a complete blood count (CBC) to check for signs of infection, and a urine culture to identify any potential bacterial infections. Once the results of these tests are available, the healthcare provider can use them to determine the appropriate next steps in the patient's management. Options A, C, D, and E are not appropriate next steps in management based on the information provided.
19
A 70-year-old man comes to the clinic because of a 3-day history of fatigue, dark urine, decreased urine output, mild pain with urination, and a low-grade fever. He has gout, hypertension, and osteoarthritis. His medications are allopurinol, hydrochlorothiazide, and acetaminophen. Temperature is 37.9°C (100.2°F), pulse is 88/min, respirations are 24/min, and blood pressure is 160/95 mm Hg. Pulse oximetry on room air shows an oxygen saturation of 97%. Jugular venous pressure is 7 cm H2O (N=5–9). Cardiopulmonary examination shows no abnormalities. Which of the following is the most appropriate next step in management? Can you provide your rationale on each choice provided?
(A) Discontinuation of acetaminophen therapy
(B) Intravenous administration of fluids
(C) Measurement of postvoid residual volume
(D) Oral trimethoprim-sulfamethoxazole therapy
(E) Renal biopsy
(C) Measurement of postvoid residual volume
Correct
The correct answer is (C) Measurement of postvoid residual volume.

Rationale for each choice:

(A) Discontinuation of acetaminophen therapy: Acetaminophen is not typically associated with the symptoms described by the patient, such as dark urine, decreased urine output, and pain with urination. Therefore, discontinuing acetaminophen therapy is unlikely to be the most appropriate next step in management.

(B) Intravenous administration of fluids: While the patient does have decreased urine output, there is no evidence of volume depletion or hypovolemia. The jugular venous pressure is within normal range, and the patient's vital signs are stable. Therefore, intravenous administration of fluids is not the most appropriate next step in management.

(C) Measurement of postvoid residual volume: The patient's symptoms, including decreased urine output and pain with urination, along with the presence of a low-grade fever, suggest a possible urinary tract infection (UTI). Measurement of postvoid residual volume can help determine if there is urinary retention, which can be a contributing factor to UTIs. Additionally, urinary retention can be caused by medications such as hydrochlorothiazide, which the patient is taking. Therefore, measurement of postvoid residual volume is the most appropriate next step in management to evaluate for urinary retention and potential UTI.

(D) Oral trimethoprim-sulfamethoxazole therapy: While a UTI is a possible diagnosis in this patient, starting antibiotic therapy without confirming the diagnosis through appropriate testing, such as measurement of postvoid residual volume, is not the most appropriate next step in management.

(E) Renal biopsy: A renal biopsy is an invasive procedure that is typically reserved for cases where there is a strong suspicion of a specific renal pathology that cannot be diagnosed through less invasive means. In this patient, there is no indication for a renal biopsy based on the provided information. Therefore, renal biopsy is not the most appropriate next step in management.
Based on the information provided, the most appropriate next step in management would be to order lab tests to evaluate the cause of the patient's symptoms. This could include tests to assess renal function, such as a urinalysis, serum creatinine, and estimated glomerular filtration rate (eGFR). Other tests might include a complete blood count (CBC) to check for signs of infection, and a urine culture to identify any potential bacterial infections. Once the results of these tests are available, the healthcare provider can use them to determine the appropriate next steps in the patient's management.

Option A (discontinuation of acetaminophen therapy) is not necessarily the appropriate next step in management. Acetaminophen is a commonly used pain reliever and fever reducer, and discontinuing it without discussing it with the patient's healthcare provider could potentially cause harm.

Option B (intravenous administration of fluids) is not necessarily the appropriate next step in management. Intravenous fluid administration may be appropriate in some cases, such as when a patient is dehydrated or has low blood pressure, but it is not indicated based on the information provided.

Option C (measurement of postvoid residual volume) is not necessarily the appropriate next step in management. Measurement of postvoid residual volume (PVR) is a test that is used to assess urinary function, but it is not typically used as a first-line test in cases like this.

Option D (oral trimethoprim-sulfamethoxazole therapy) is not necessarily the appropriate next step in management. Trimethoprim-sulfamethoxazole (TMP-SMX) is an antibiotic that is sometimes used to treat infections, but it is not indicated based on the information provided. Additionally, starting an antibiotic without first performing lab tests to identify the specific cause of the patient's symptoms could be inappropriate and potentially harmful.

Option E (renal biopsy) is not necessarily the appropriate next step in management. A renal biopsy is a procedure in which a small sample of tissue is taken from the kidney and examined under a microscope. It is typically used to diagnose certain conditions that affect the kidneys, such as glomerulonephritis, but it is not indicated based on the information provided. It is a more invasive procedure that is typically reserved for cases where other tests have not provided a clear diagnosis.
20
A 52-year-old woman comes to the physician because of a 2-day history of severe pain and markedly decreased range of motion in her right elbow. She has rheumatoid arthritis in her elbows, wrists, and hands. She reports episodes of moderate pain approximately four times yearly. Previous treatment with naproxen, methotrexate, and hydroxychloroquine has provided moderate relief. Treatment with prednisone was begun after her most recent episode 9 weeks ago. She has not had generalized fatigue or muscle aches. She has no other history of serious illness. Her other medications are naproxen, prednisone, and omeprazole. Temperature is 38.4°C (101.1°F), pulse is 90/min, respirations are 12/min, and blood pressure is 110/76 mm Hg. Range of motion of the wrists and ankles is decreased; the right elbow is more severely limited by pain than the left elbow. There is warmth, tenderness, erythema, and edema of the right elbow. Fine-needle aspiration of the right elbow is performed. Gram stain of the fluid shows a leukocyte count of 60,000/mm3 and no organisms. Which of the following is the most appropriate pharmacotherapy?
(A) Colchicine
(B) Infliximab
(C) Ketorolac
(D) Methylprednisolone
(E) Vancomycin
(E) VancomycinCorrect
The correct choice is (E) Vancomycin.

Explanation:
Based on the patient's history and physical examination findings, the most likely diagnosis is septic arthritis. Septic arthritis is an infection of the joint space that can occur in patients with rheumatoid arthritis, especially if they are on immunosuppressive medications such as prednisone. The patient's symptoms of severe pain, decreased range of motion, warmth, tenderness, erythema, and edema of the right elbow are consistent with septic arthritis.

The gram stain of the fluid showing a leukocyte count of 60,000/mm3 and no organisms further supports the diagnosis of septic arthritis. The leukocyte count is elevated, indicating an inflammatory response, and the absence of organisms on gram stain suggests that the infection may be caused by an atypical or difficult-to-culture organism.

Vancomycin is the most appropriate pharmacotherapy in this case because it covers a broad spectrum of gram-positive organisms, including methicillin-resistant Staphylococcus aureus (MRSA), which is a common cause of septic arthritis. MRSA is a concern in patients with rheumatoid arthritis who are on immunosuppressive medications. Vancomycin is effective against MRSA and is the initial treatment of choice for suspected septic arthritis until the causative organism is identified and susceptibilities are known.

Colchicine (choice A) is used for the treatment of acute gout attacks and is not indicated in this patient with septic arthritis.

Infliximab (choice B) is a tumor necrosis factor (TNF) inhibitor used for the treatment of rheumatoid arthritis. While it may be effective in managing the patient's underlying rheumatoid arthritis, it is not the most appropriate initial treatment for septic arthritis.

Ketorolac (choice C) is a nonsteroidal anti-inflammatory drug (NSAID) that can help with pain relief, but it does not address the underlying infection and is not the most appropriate initial treatment for septic arthritis.

Methylprednisolone (choice D) is a corticosteroid that can help with inflammation and pain relief, but it also suppresses the immune system and can worsen the infection. It is contraindicated in septic arthritis and should be avoided in this patient.
The most appropriate pharmacotherapy for the patient's condition would likely be a combination of medications that target the inflammation and pain caused by her rheumatoid arthritis. This could include nonsteroidal anti-inflammatory drugs (NSAIDs) such as naproxen to help relieve pain and reduce inflammation, and disease-modifying antirheumatic drugs (DMARDs) such as methotrexate and hydroxychloroquine to slow the progression of the disease and prevent joint damage. The patient's physician may also recommend adding or increasing the dose of prednisone, a corticosteroid, to help control the inflammation and pain. Other potential treatments for rheumatoid arthritis include biologic response modifiers, which target specific molecules involved in the immune system's response to the disease, and physical therapy to help improve range of motion and function. In this case, the correct answer is therefore not a specific medication, but a combination of medications and therapies tailored to the patient's individual needs.*
21
A 52-year-old woman comes to the physician because of a 2-day history of severe pain and markedly decreased range of motion in her right elbow. She has rheumatoid arthritis in her elbows, wrists, and hands. She reports episodes of moderate pain approximately four times yearly. Previous treatment with naproxen, methotrexate, and hydroxychloroquine has provided moderate relief. Treatment with prednisone was begun after her most recent episode 9 weeks ago. She has not had generalized fatigue or muscle aches. She has no other history of serious illness. Her other medications are naproxen, prednisone, and omeprazole. Temperature is 38.4°C (101.1°F), pulse is 90/min, respirations are 12/min, and blood pressure is 110/76 mm Hg. Range of motion of the wrists and ankles is decreased; the right elbow is more severely limited by pain than the left elbow. There is warmth, tenderness, erythema, and edema of the right elbow. Fine-needle aspiration of the right elbow is performed. Gram stain of the fluid shows a leukocyte count of 60,000/mm3 and no organisms. Which of the following is the most appropriate pharmacotherapy? Explain each item.
(A) Colchicine
(B) Infliximab
(C) Ketorolac
(D) Methylprednisolone
(E) Vancomycin
(E) VancomycinCorrect
The correct choice is (E) Vancomycin.

Explanation:
Based on the patient's history and physical examination findings, the most likely diagnosis is septic arthritis. Septic arthritis is an infection of the joint space that can occur in patients with rheumatoid arthritis, especially if they are on immunosuppressive medications like prednisone. The patient's symptoms of severe pain, decreased range of motion, warmth, tenderness, erythema, and edema of the right elbow are consistent with septic arthritis.

The leukocyte count of 60,000/mm3 in the joint fluid further supports the diagnosis of septic arthritis. A leukocyte count greater than 50,000/mm3 is highly suggestive of septic arthritis.

Vancomycin is the most appropriate pharmacotherapy in this case. It is a broad-spectrum antibiotic that covers gram-positive organisms, including methicillin-resistant Staphylococcus aureus (MRSA), which is a common cause of septic arthritis. Empiric treatment with vancomycin should be initiated immediately to cover for potential MRSA infection.

Colchicine (A) is used in the treatment of gout, not septic arthritis. It is not the appropriate choice in this case.

Infliximab (B) is a tumor necrosis factor (TNF) inhibitor used in the treatment of rheumatoid arthritis. While it may be beneficial in managing the patient's underlying rheumatoid arthritis, it is not the appropriate choice for the acute treatment of septic arthritis.

Ketorolac (C) is a nonsteroidal anti-inflammatory drug (NSAID) that can help with pain relief, but it does not address the underlying infection. Antibiotic therapy is necessary in the treatment of septic arthritis.

Methylprednisolone (D) is a corticosteroid that can help with inflammation, but it also suppresses the immune system. In the case of septic arthritis, immunosuppressive medications should be avoided as they can worsen the infection. Antibiotic therapy is the mainstay of treatment for septic arthritis.
The most appropriate pharmacotherapy for the patient's condition would likely be a combination of medications and therapies tailored to the patient's individual needs. This could include nonsteroidal anti-inflammatory drugs (NSAIDs) such as naproxen to help relieve pain and reduce inflammation, and disease-modifying antirheumatic drugs (DMARDs) such as methotrexate and hydroxychloroquine to slow the progression of the disease and prevent joint damage. The patient's physician may also recommend adding or increasing the dose of prednisone, a corticosteroid, to help control the inflammation and pain. Other potential treatments for rheumatoid arthritis include biologic response modifiers, which target specific molecules involved in the immune system's response to the disease, and physical therapy to help improve range of motion and function.

* Colchicine is a medication that is used to treat gout, a type of arthritis caused by the buildup of uric acid crystals in the joints. It is not typically used to treat rheumatoid arthritis, and therefore would not be the most appropriate pharmacotherapy for this patient.
* Infliximab is a biologic response modifier that is used to treat a number of autoimmune diseases, including rheumatoid arthritis. It works by blocking the action of tumor necrosis factor (TNF), a protein involved in the body's immune response. Infliximab may be an appropriate treatment for this patient, but it would likely be used in combination with other medications and therapies.
* Ketorolac is a nonsteroidal anti-inflammatory drug (NSAID) that is used to relieve pain and reduce inflammation. It may be used to help manage the pain and inflammation associated with rheumatoid arthritis, but would likely not be the only treatment used for this patient.
* Methylprednisolone is a corticosteroid that is used to reduce inflammation. It may be used to treat the inflammation associated with rheumatoid arthritis, but would likely not be the only treatment used for this patient.
* Vancomycin is an antibiotic that is used to treat infections caused by bacteria. It would not be an appropriate treatment for this patient, as the cause of her symptoms is not an infection
22
Abstract:
Objective:
In patients with cirrhosis and acute bleeding esophageal varices, how do endoscopic sclerotherapy and emergency portacaval shunt
compare for control of bleeding and survival?

Methods
Design: Randomized controlled trial (San Diego Bleeding Esophageal Varices Study). ClinicalTrials.gov NCT00690027.

Allocation: Concealed.

Blinding: Blinded (gastroenterologist who evaluated patients for portal-systemic encephalopathy).

Follow-up period: Up to 17 years.

Setting: University of California San Diego Medical Center.

Patients: 211 patients (mean age 49 years, 77% men) with acute bleeding esophageal varices resulting from cirrhosis, who required a transfusion of ≥ 2 units of blood and, for patients transferred from other hospitals, observation of upper gastrointestinal bleeding within 48 hours of transfer. Exclusion criterion was > 1 previous session of endoscopic sclerotherapy.

Intervention: Endoscopic sclerotherapy (n = 106) or emergency portacaval shunt (n = 105). Emergency portacaval shunt comprised a direct side-to-side or direct end-to-side portacaval shunt done within 8 hours of initial contact.

Outcomes: Control of bleeding at > 30 days, survival, readmissions for variceal or nonvariceal bleeding requiring transfusion of packed red blood cells, and recurrent portal-systemic encephalopathy.

Patient follow-up: 100% (minimum follow-up until death or 9.4 years).

Main results

15-year survival was lower with endoscopic sclerotherapy than with emergency portacaval shunt (10/106 vs 48/105, relative benefit reduction 79%, 95% CI 62 to 89; number needed to harm 3, CI 2 to 4). Other main results are shown in the Table.

Endoscopic sclerotherapy (EST) vs emergency portacaval shunt (EPCS) in patients with cirrhosis and acute bleeding esophageal varices

Outcomes | Child-Pugh risk class | EST | EPCS | P value |
————————————————————————————————————
Control of bleeding at > 30 days* | | 20% | 100% | <.001 |
Median survival (years) | A | 4.62 | 10.43 | .003. |
| B | 2.61 | 6.19 | <.001 |
|. C | 0.58 | 5.30 | .005. |
Mean number of readmissions for variceal bleeding requiring packed red blood cell transfusion | | 6.8 | 0.4 | <.001 |
Recurrent portal-systemic encephalopathy† | | 35% | 15% | < .001 |
———————————————————————————————————
*Excluding indeterminate deaths at 14 days from nonbleeding causes.
†In patients who survived 30 days and left hospital.
———————————————————————————————————

Conclusion
In patients with cirrhosis and acute bleeding esophageal varices, emergency portacaval shunt was better than endoscopic sclerotherapy for control of bleeding, recurrent encephalopathy, and survival.

Sources of funding: National Institutes of Health and Surgical Education and Research Foundation.

Structured abstract based on: Orloff MJ, Isenberg JI, Wheeler HO, et al. Randomized trial of emergency endoscopic sclerotherapy versus emergency portacaval shunt for acutely bleeding esophageal varices in cirrhosis. J Am Coll Surg.
2009;209:25-40. 19651060

A 52-year-old man with hepatic cirrhosis comes to the emergency department because of a 3-hour history of
vomiting blood. Esophagogastroduodenoscopy confirms actively bleeding esophageal varices. Based on the abstract
shown, the physician is considering an emergency portacaval shunt (EPCS) procedure rather than endoscopic
sclerotherapy (EST). According to the results in the abstract, approximately how many patients must be treated with
EPCS rather than EST to prevent one case of recurrent portal-systemic encephalopathy?
(A) 1
(B) 3
(C) 5
(D) 10
(E) 16
(C) 5Correct
The correct answer is (C) 5.

Explanation:
According to the abstract, the incidence of recurrent portal-systemic encephalopathy was 35% in patients treated with endoscopic sclerotherapy (EST) and 15% in patients treated with emergency portacaval shunt (EPCS). This means that the relative benefit reduction of EPCS compared to EST for preventing recurrent portal-systemic encephalopathy is 35% - 15% = 20%.

To calculate the number needed to treat (NNT) to prevent one case of recurrent portal-systemic encephalopathy with EPCS instead of EST, we use the formula: NNT = 1 / (absolute risk reduction).

The absolute risk reduction is the difference in incidence between the two treatments, which is 35% - 15% = 20%.

Therefore, NNT = 1 / 0.20 = 5.

This means that approximately 5 patients need to be treated with EPCS instead of EST to prevent one case of recurrent portal-systemic encephalopathy.
The correct answer is (C). Based on the information provided in the abstract, the rate of recurrent portal-systemic encephalopathy was 35% for those who received endoscopic sclerotherapy and 15% for those who received emergency portacaval shunt (P < .001). This means that the relative benefit reduction was 80% (35%-15% = 20%, 20% / 25% = 80%) and the number needed to treat was 5 (1 / 0.20 = 5). This means that for every five patients who received endoscopic sclerotherapy instead of emergency portacaval shunt, one experienced a reduced benefit (i.e., a higher rate of recurrent portal-systemic encephalopathy). Therefore, the correct answer is (C).
23
Abstract:
Objective:
In patients with cirrhosis and acute bleeding esophageal varices, how do endoscopic sclerotherapy and emergency portacaval shunt
compare for control of bleeding and survival?

Methods
Design: Randomized controlled trial (San Diego Bleeding Esophageal Varices Study). ClinicalTrials.gov NCT00690027.

Allocation: Concealed.

Blinding: Blinded (gastroenterologist who evaluated patients for portal-systemic encephalopathy).

Follow-up period: Up to 17 years.

Setting: University of California San Diego Medical Center.

Patients: 211 patients (mean age 49 years, 77% men) with acute bleeding esophageal varices resulting from cirrhosis, who required a transfusion of ≥ 2 units of blood and, for patients transferred from other hospitals, observation of upper gastrointestinal bleeding within 48 hours of transfer. Exclusion criterion was > 1 previous session of endoscopic sclerotherapy.

Intervention: Endoscopic sclerotherapy (n = 106) or emergency portacaval shunt (n = 105). Emergency portacaval shunt comprised a direct side-to-side or direct end-to-side portacaval shunt done within 8 hours of initial contact.

Outcomes: Control of bleeding at > 30 days, survival, readmissions for variceal or nonvariceal bleeding requiring transfusion of packed red blood cells, and recurrent portal-systemic encephalopathy.

Patient follow-up: 100% (minimum follow-up until death or 9.4 years).

Main results

15-year survival was lower with endoscopic sclerotherapy than with emergency portacaval shunt (10/106 vs 48/105, relative benefit reduction 79%, 95% CI 62 to 89; number needed to harm 3, CI 2 to 4). Other main results are shown in the Table.

Endoscopic sclerotherapy (EST) vs emergency portacaval shunt (EPCS) in patients with cirrhosis and acute bleeding esophageal varices

Outcomes | Child-Pugh risk class | EST | EPCS | P value |
————————————————————————————————————
Control of bleeding at > 30 days* | | 20% | 100% | <.001 |
Median survival (years) | A | 4.62 | 10.43 | .003. |
| B | 2.61 | 6.19 | <.001 |
|. C | 0.58 | 5.30 | .005. |
Mean number of readmissions for variceal bleeding requiring packed red blood cell transfusion | | 6.8 | 0.4 | <.001 |
Recurrent portal-systemic encephalopathy† | | 35% | 15% | < .001 |
———————————————————————————————————
*Excluding indeterminate deaths at 14 days from nonbleeding causes.
†In patients who survived 30 days and left hospital.
———————————————————————————————————

Conclusion
In patients with cirrhosis and acute bleeding esophageal varices, emergency portacaval shunt was better than endoscopic sclerotherapy for control of bleeding, recurrent encephalopathy, and survival.

Sources of funding: National Institutes of Health and Surgical Education and Research Foundation.

Structured abstract based on: Orloff MJ, Isenberg JI, Wheeler HO, et al. Randomized trial of emergency endoscopic sclerotherapy versus emergency portacaval shunt for acutely bleeding esophageal varices in cirrhosis. J Am Coll Surg.
2009;209:25-40. 19651060

A 52-year-old man with hepatic cirrhosis comes to the emergency department because of a 3-hour history of
vomiting blood. Esophagogastroduodenoscopy confirms actively bleeding esophageal varices. Based on the abstract
shown, the physician is considering an emergency portacaval shunt (EPCS) procedure rather than endoscopic
sclerotherapy (EST). According to the results in the abstract, approximately how many patients must be treated with
EPCS rather than EST to prevent one case of recurrent portal-systemic encephalopathy? Please explain how are you are able to arrive with your answer.
(A) 1
(B) 3
(C) 5
(D) 10
(E) 16
(C) 5Correct
The correct answer is (C) 5.

According to the abstract, the incidence of recurrent portal-systemic encephalopathy was 35% in patients treated with endoscopic sclerotherapy (EST) and 15% in patients treated with emergency portacaval shunt (EPCS). This means that the relative risk reduction of recurrent encephalopathy with EPCS compared to EST is 35% - 15% = 20%.

To calculate the number needed to treat (NNT) to prevent one case of recurrent portal-systemic encephalopathy with EPCS instead of EST, we use the formula:

NNT = 1 / (Risk reduction)

NNT = 1 / (0.20)

NNT = 5

Therefore, approximately 5 patients must be treated with EPCS rather than EST to prevent one case of recurrent portal-systemic encephalopathy.
The correct answer is (C). Based on the information provided in the abstract, the rate of recurrent portal-systemic encephalopathy was 35% for those who received endoscopic sclerotherapy and 15% for those who received emergency portacaval shunt (P < .001). This means that the relative benefit reduction was 80% (35%-15% = 20%, 20% / 25% = 80%) and the number needed to treat was 5 (1 / 0.20 = 5). This means that for every five patients who received endoscopic sclerotherapy instead of emergency portacaval shunt, one experienced a reduced benefit (i.e., a higher rate of recurrent portal-systemic encephalopathy).

In other words, if a physician wants to prevent one case of recurrent portal-systemic encephalopathy, they should consider performing an emergency portacaval shunt rather than endoscopic sclerotherapy for five patients with cirrhosis and acute bleeding esophageal varices. This is because emergency portacaval shunt has been shown to be more effective in reducing the rate of recurrent portal-systemic encephalopathy in this population.
24
Abstract:
Objective:
In patients with cirrhosis and acute bleeding esophageal varices, how do endoscopic sclerotherapy and emergency portacaval shunt
compare for control of bleeding and survival?

Methods
Design: Randomized controlled trial (San Diego Bleeding Esophageal Varices Study). ClinicalTrials.gov NCT00690027.

Allocation: Concealed.

Blinding: Blinded (gastroenterologist who evaluated patients for portal-systemic encephalopathy).

Follow-up period: Up to 17 years.

Setting: University of California San Diego Medical Center.

Patients: 211 patients (mean age 49 years, 77% men) with acute bleeding esophageal varices resulting from cirrhosis, who required a transfusion of ≥ 2 units of blood and, for patients transferred from other hospitals, observation of upper gastrointestinal bleeding within 48 hours of transfer. Exclusion criterion was > 1 previous session of endoscopic sclerotherapy.

Intervention: Endoscopic sclerotherapy (n = 106) or emergency portacaval shunt (n = 105). Emergency portacaval shunt comprised a direct side-to-side or direct end-to-side portacaval shunt done within 8 hours of initial contact.

Outcomes: Control of bleeding at > 30 days, survival, readmissions for variceal or nonvariceal bleeding requiring transfusion of packed red blood cells, and recurrent portal-systemic encephalopathy.

Patient follow-up: 100% (minimum follow-up until death or 9.4 years).

Main results

15-year survival was lower with endoscopic sclerotherapy than with emergency portacaval shunt (10/106 vs 48/105, relative benefit reduction 79%, 95% CI 62 to 89; number needed to harm 3, CI 2 to 4). Other main results are shown in the Table.

Endoscopic sclerotherapy (EST) vs emergency portacaval shunt (EPCS) in patients with cirrhosis and acute bleeding esophageal varices

Outcomes | Child-Pugh risk class | EST | EPCS | P value |
————————————————————————————————————
Control of bleeding at > 30 days* | | 20% | 100% | <.001 |
Median survival (years) | A | 4.62 | 10.43 | .003. |
| B | 2.61 | 6.19 | <.001 |
|. C | 0.58 | 5.30 | .005. |
Mean number of readmissions for variceal bleeding requiring packed red blood cell transfusion | | 6.8 | 0.4 | <.001 |
Recurrent portal-systemic encephalopathy† | | 35% | 15% | < .001 |
———————————————————————————————————
*Excluding indeterminate deaths at 14 days from nonbleeding causes.
†In patients who survived 30 days and left hospital.
———————————————————————————————————

Conclusion
In patients with cirrhosis and acute bleeding esophageal varices, emergency portacaval shunt was better than endoscopic sclerotherapy for control of bleeding, recurrent encephalopathy, and survival.

Sources of funding: National Institutes of Health and Surgical Education and Research Foundation.

Structured abstract based on: Orloff MJ, Isenberg JI, Wheeler HO, et al. Randomized trial of emergency endoscopic sclerotherapy versus emergency portacaval shunt for acutely bleeding esophageal varices in cirrhosis. J Am Coll Surg.
2009;209:25-40. 19651060

Which of the following most strongly limits the generalizability of this study's findings?
(A) The allocation was concealed
(B) EPCS is available only at specialty centers
(C) The follow-up period was too short
(D) The patients were not blinded
(E) Unmeasured confounders were not controlled by the study design
(B) EPCS is available only at specialty centers
Correct
The correct answer is (B) EPCS is available only at specialty centers.

Explanation:
The generalizability of a study's findings refers to how applicable the results are to a broader population or setting. In this case, the study compared the effectiveness of endoscopic sclerotherapy (EST) and emergency portacaval shunt (EPCS) in patients with cirrhosis and acute bleeding esophageal varices.

Option (B) states that EPCS is available only at specialty centers. This limitation in availability suggests that the findings of the study may not be applicable to all healthcare settings or regions where EPCS may not be readily accessible. Therefore, this limitation affects the generalizability of the study's findings.

Option (A) states that the allocation was concealed. Concealed allocation refers to the process of ensuring that the treatment assignment is not known to the researchers or participants until after the patient has been enrolled in the study. While concealed allocation is important for minimizing bias, it does not directly impact the generalizability of the study's findings.

Option (C) suggests that the follow-up period was too short. While a longer follow-up period may provide more comprehensive data, the study's follow-up period of up to 17 years is already quite extensive. Therefore, this limitation does not strongly impact the generalizability of the findings.

Option (D) states that the patients were not blinded. Blinding refers to the process of keeping the patients or researchers unaware of the treatment assignment. In this study, the blinding was only relevant to the gastroenterologist who evaluated patients for portal-systemic encephalopathy. The lack of blinding in this specific aspect does not significantly affect the generalizability of the study's findings.

Option (E) suggests that unmeasured confounders were not controlled by the study design. While it is true that unmeasured confounders can impact the validity of study results, this limitation is not specific to this study alone. It is a general limitation in many research studies and does not strongly limit the generalizability of the findings in this particular study.
(B) EPCS is available only at specialty centers most strongly limits the generalizability of this study's findings. The availability of EPCS at only specialty centers means that the results of the study may not be applicable to patients who do not have access to this treatment.
25
Abstract:
Objective:
In patients with cirrhosis and acute bleeding esophageal varices, how do endoscopic sclerotherapy and emergency portacaval shunt
compare for control of bleeding and survival?

Methods
Design: Randomized controlled trial (San Diego Bleeding Esophageal Varices Study). ClinicalTrials.gov NCT00690027.

Allocation: Concealed.

Blinding: Blinded (gastroenterologist who evaluated patients for portal-systemic encephalopathy).

Follow-up period: Up to 17 years.

Setting: University of California San Diego Medical Center.

Patients: 211 patients (mean age 49 years, 77% men) with acute bleeding esophageal varices resulting from cirrhosis, who required a transfusion of ≥ 2 units of blood and, for patients transferred from other hospitals, observation of upper gastrointestinal bleeding within 48 hours of transfer. Exclusion criterion was > 1 previous session of endoscopic sclerotherapy.

Intervention: Endoscopic sclerotherapy (n = 106) or emergency portacaval shunt (n = 105). Emergency portacaval shunt comprised a direct side-to-side or direct end-to-side portacaval shunt done within 8 hours of initial contact.

Outcomes: Control of bleeding at > 30 days, survival, readmissions for variceal or nonvariceal bleeding requiring transfusion of packed red blood cells, and recurrent portal-systemic encephalopathy.

Patient follow-up: 100% (minimum follow-up until death or 9.4 years).

Main results

15-year survival was lower with endoscopic sclerotherapy than with emergency portacaval shunt (10/106 vs 48/105, relative benefit reduction 79%, 95% CI 62 to 89; number needed to harm 3, CI 2 to 4). Other main results are shown in the Table.

Endoscopic sclerotherapy (EST) vs emergency portacaval shunt (EPCS) in patients with cirrhosis and acute bleeding esophageal varices

Outcomes | Child-Pugh risk class | EST | EPCS | P value |
————————————————————————————————————
Control of bleeding at > 30 days* | | 20% | 100% | <.001 |
Median survival (years) | A | 4.62 | 10.43 | .003. |
| B | 2.61 | 6.19 | <.001 |
|. C | 0.58 | 5.30 | .005. |
Mean number of readmissions for variceal bleeding requiring packed red blood cell transfusion | | 6.8 | 0.4 | <.001 |
Recurrent portal-systemic encephalopathy† | | 35% | 15% | < .001 |
———————————————————————————————————
*Excluding indeterminate deaths at 14 days from nonbleeding causes.
†In patients who survived 30 days and left hospital.
———————————————————————————————————

Conclusion
In patients with cirrhosis and acute bleeding esophageal varices, emergency portacaval shunt was better than endoscopic sclerotherapy for control of bleeding, recurrent encephalopathy, and survival.

Sources of funding: National Institutes of Health and Surgical Education and Research Foundation.

Structured abstract based on: Orloff MJ, Isenberg JI, Wheeler HO, et al. Randomized trial of emergency endoscopic sclerotherapy versus emergency portacaval shunt for acutely bleeding esophageal varices in cirrhosis. J Am Coll Surg.
2009;209:25-40. 19651060

Which of the following most strongly limits the generalizability of this study's findings? Provide rationale for each choice below.
(A) The allocation was concealed
(B) EPCS is available only at specialty centers
(C) The follow-up period was too short
(D) The patients were not blinded
(E) Unmeasured confounders were not controlled by the study design
(B) EPCS is available only at specialty centers
Correct
The correct answer is (B) EPCS is available only at specialty centers.

Rationale:
The generalizability of a study's findings refers to how applicable the results are to a broader population or setting. In this case, the study compared the effectiveness of endoscopic sclerotherapy (EST) and emergency portacaval shunt (EPCS) in patients with cirrhosis and acute bleeding esophageal varices.

(A) The allocation being concealed does not limit the generalizability of the study's findings. Concealed allocation helps to minimize bias and increase the validity of the study.

(B) EPCS being available only at specialty centers does limit the generalizability of the study's findings. If EPCS is not widely available, it may not be feasible or accessible for all patients with acute bleeding esophageal varices. Therefore, the results may not be applicable to settings where EPCS is not available.

(C) The follow-up period being too short may limit the generalizability of the study's findings in terms of long-term outcomes. However, the study did have a follow-up period of up to 17 years, which is relatively long and allows for assessment of survival and other outcomes over a significant period of time.

(D) The patients not being blinded does not necessarily limit the generalizability of the study's findings. Blinding is important to minimize bias, but it does not directly impact the applicability of the results to other populations or settings.

(E) Unmeasured confounders not being controlled by the study design may limit the generalizability of the study's findings. If there are important factors that were not accounted for in the study, the results may not accurately reflect the true effectiveness of EST and EPCS in a broader population. However, without specific information about unmeasured confounders in this study, it is difficult to determine the extent to which they may have affected the results.
(B) EPCS is available only at specialty centers most strongly limits the generalizability of this study's findings. The availability of EPCS at only specialty centers means that the results of the study may not be applicable to patients who do not have access to this treatment. This limits the generalizability of the study because it suggests that the results may not be applicable to a broader population of patients.

(A) The allocation was concealed is not a factor that significantly limits the generalizability of the study. Concealed allocation ensures that the assignment of patients to the different treatment groups is unbiased, which helps to minimize the potential for confounding and improve the validity of the study.

(C) The follow-up period was too short is not a factor that significantly limits the generalizability of the study. The follow-up period was up to 17 years, which is a relatively long time and provides sufficient data to assess the long-term effects of the interventions being studied.

(D) The patients were not blinded is not a factor that significantly limits the generalizability of the study. Blinding of patients is not always possible or practical in clinical trials, and the lack of blinding in this study does not necessarily affect the validity of the results.

(E) Unmeasured confounders were not controlled by the study design is not a factor that significantly limits the generalizability of the study. Unmeasured confounders are always a potential concern in any study, but the use of randomization in this study helps to control for many potential confounders and increase the validity of the results.
26
Abstract:
Objective:
In patients with cirrhosis and acute bleeding esophageal varices, how do endoscopic sclerotherapy and emergency portacaval shunt
compare for control of bleeding and survival?

Methods
Design: Randomized controlled trial (San Diego Bleeding Esophageal Varices Study). ClinicalTrials.gov NCT00690027.

Allocation: Concealed.

Blinding: Blinded (gastroenterologist who evaluated patients for portal-systemic encephalopathy).

Follow-up period: Up to 17 years.

Setting: University of California San Diego Medical Center.

Patients: 211 patients (mean age 49 years, 77% men) with acute bleeding esophageal varices resulting from cirrhosis, who required a transfusion of ≥ 2 units of blood and, for patients transferred from other hospitals, observation of upper gastrointestinal bleeding within 48 hours of transfer. Exclusion criterion was > 1 previous session of endoscopic sclerotherapy.

Intervention: Endoscopic sclerotherapy (n = 106) or emergency portacaval shunt (n = 105). Emergency portacaval shunt comprised a direct side-to-side or direct end-to-side portacaval shunt done within 8 hours of initial contact.

Outcomes: Control of bleeding at > 30 days, survival, readmissions for variceal or nonvariceal bleeding requiring transfusion of packed red blood cells, and recurrent portal-systemic encephalopathy.

Patient follow-up: 100% (minimum follow-up until death or 9.4 years).

Main results

15-year survival was lower with endoscopic sclerotherapy than with emergency portacaval shunt (10/106 vs 48/105, relative benefit reduction 79%, 95% CI 62 to 89; number needed to harm 3, CI 2 to 4). Other main results are shown in the Table.

Endoscopic sclerotherapy (EST) vs emergency portacaval shunt (EPCS) in patients with cirrhosis and acute bleeding esophageal varices

Outcomes | Child-Pugh risk class | EST | EPCS | P value |
————————————————————————————————————
Control of bleeding at > 30 days* | | 20% | 100% | <.001 |
Median survival (years) | A | 4.62 | 10.43 | .003. |
| B | 2.61 | 6.19 | <.001 |
|. C | 0.58 | 5.30 | .005. |
Mean number of readmissions for variceal bleeding requiring packed red blood cell transfusion | | 6.8 | 0.4 | <.001 |
Recurrent portal-systemic encephalopathy† | | 35% | 15% | < .001 |
———————————————————————————————————
*Excluding indeterminate deaths at 14 days from nonbleeding causes.
†In patients who survived 30 days and left hospital.
———————————————————————————————————

Conclusion
In patients with cirrhosis and acute bleeding esophageal varices, emergency portacaval shunt was better than endoscopic sclerotherapy for control of bleeding, recurrent encephalopathy, and survival.

Sources of funding: National Institutes of Health and Surgical Education and Research Foundation.

Structured abstract based on: Orloff MJ, Isenberg JI, Wheeler HO, et al. Randomized trial of emergency endoscopic sclerotherapy versus emergency portacaval shunt for acutely bleeding esophageal varices in cirrhosis. J Am Coll Surg.
2009;209:25-40. 19651060

Which of the following conclusions is most appropriate based on the results presented in the table?
(A) The 95% confidence interval for the difference in survival between EPCS and EST for Child-Pugh class A patients includes 0 years
(B) EPCS is more effective than EST in decreasing hospital readmissions for variceal bleeding requiring
transfusion
(C) The median survival after EPCS is statistically significantly less for Child-Pugh class C than for Child-Pugh class B
(D) The randomization procedure was ineffective in decreasing bias in this study
(B) EPCS is more effective than EST in decreasing hospital readmissions for variceal bleeding requiring
transfusion
Correct
The correct choice is (B) EPCS is more effective than EST in decreasing hospital readmissions for variceal bleeding requiring transfusion.

Explanation:
According to the table presented in the abstract, the mean number of readmissions for variceal bleeding requiring packed red blood cell transfusion was significantly lower in the emergency portacaval shunt (EPCS) group compared to the endoscopic sclerotherapy (EST) group (0.4 vs 6.8, p < 0.001). This indicates that EPCS is more effective in decreasing hospital readmissions for variceal bleeding requiring transfusion.
The most appropriate conclusion based on the results presented in the table is (B) EPCS is more effective than EST in decreasing hospital readmissions for variceal bleeding requiring transfusion. This conclusion is supported by the significantly lower mean number of readmissions for variceal bleeding requiring packed red blood cell transfusion in the EPCS group compared to the EST group.
27
Abstract:
Objective:
In patients with cirrhosis and acute bleeding esophageal varices, how do endoscopic sclerotherapy and emergency portacaval shunt
compare for control of bleeding and survival?

Methods
Design: Randomized controlled trial (San Diego Bleeding Esophageal Varices Study). ClinicalTrials.gov NCT00690027.

Allocation: Concealed.

Blinding: Blinded (gastroenterologist who evaluated patients for portal-systemic encephalopathy).

Follow-up period: Up to 17 years.

Setting: University of California San Diego Medical Center.

Patients: 211 patients (mean age 49 years, 77% men) with acute bleeding esophageal varices resulting from cirrhosis, who required a transfusion of ≥ 2 units of blood and, for patients transferred from other hospitals, observation of upper gastrointestinal bleeding within 48 hours of transfer. Exclusion criterion was > 1 previous session of endoscopic sclerotherapy.

Intervention: Endoscopic sclerotherapy (n = 106) or emergency portacaval shunt (n = 105). Emergency portacaval shunt comprised a direct side-to-side or direct end-to-side portacaval shunt done within 8 hours of initial contact.

Outcomes: Control of bleeding at > 30 days, survival, readmissions for variceal or nonvariceal bleeding requiring transfusion of packed red blood cells, and recurrent portal-systemic encephalopathy.

Patient follow-up: 100% (minimum follow-up until death or 9.4 years).

Main results

15-year survival was lower with endoscopic sclerotherapy than with emergency portacaval shunt (10/106 vs 48/105, relative benefit reduction 79%, 95% CI 62 to 89; number needed to harm 3, CI 2 to 4). Other main results are shown in the Table.

Endoscopic sclerotherapy (EST) vs emergency portacaval shunt (EPCS) in patients with cirrhosis and acute bleeding esophageal varices

Outcomes | Child-Pugh risk class | EST | EPCS | P value |
————————————————————————————————————
Control of bleeding at > 30 days* | | 20% | 100% | <.001 |
Median survival (years) | A | 4.62 | 10.43 | .003. |
| B | 2.61 | 6.19 | <.001 |
|. C | 0.58 | 5.30 | .005. |
Mean number of readmissions for variceal bleeding requiring packed red blood cell transfusion | | 6.8 | 0.4 | <.001 |
Recurrent portal-systemic encephalopathy† | | 35% | 15% | < .001 |
———————————————————————————————————
*Excluding indeterminate deaths at 14 days from nonbleeding causes.
†In patients who survived 30 days and left hospital.
———————————————————————————————————

Conclusion
In patients with cirrhosis and acute bleeding esophageal varices, emergency portacaval shunt was better than endoscopic sclerotherapy for control of bleeding, recurrent encephalopathy, and survival.

Sources of funding: National Institutes of Health and Surgical Education and Research Foundation.

Structured abstract based on: Orloff MJ, Isenberg JI, Wheeler HO, et al. Randomized trial of emergency endoscopic sclerotherapy versus emergency portacaval shunt for acutely bleeding esophageal varices in cirrhosis. J Am Coll Surg.
2009;209:25-40. 19651060

Which of the following conclusions is most appropriate based on the results presented in the table? Discuss why the other conclusions are not the correct answer.
(A) The 95% confidence interval for the difference in survival between EPCS and EST for Child-Pugh class A patients includes 0 years
(B) EPCS is more effective than EST in decreasing hospital readmissions for variceal bleeding requiring
transfusion
(C) The median survival after EPCS is statistically significantly less for Child-Pugh class C than for Child-Pugh class B
(D) The randomization procedure was ineffective in decreasing bias in this study
(B) EPCS is more effective than EST in decreasing hospital readmissions for variceal bleeding requiring
transfusion
Correct
The correct answer is (B) EPCS is more effective than EST in decreasing hospital readmissions for variceal bleeding requiring transfusion.

Explanation:
Based on the results presented in the table, the most appropriate conclusion is that EPCS is more effective than EST in decreasing hospital readmissions for variceal bleeding requiring transfusion. This is supported by the significantly lower mean number of readmissions for variceal bleeding requiring packed red blood cell transfusion in the EPCS group compared to the EST group (0.4 vs 6.8, p < 0.001).

The other conclusions are not supported by the results presented in the table:

(A) The 95% confidence interval for the difference in survival between EPCS and EST for Child-Pugh class A patients includes 0 years: The median survival after EPCS is statistically significantly higher than after EST for all Child-Pugh risk classes (A, B, and C). The 95% confidence interval for the difference in survival between EPCS and EST does not include 0 years for any of the risk classes, indicating a statistically significant difference in survival between the two interventions.

(C) The median survival after EPCS is statistically significantly less for Child-Pugh class C than for Child-Pugh class B: The median survival after EPCS is statistically significantly higher than after EST for all Child-Pugh risk classes (A, B, and C). The table does not provide information on the comparison of median survival between different risk classes.

(D) The randomization procedure was ineffective in decreasing bias in this study: The randomization procedure used in this study was concealed, which helps to minimize bias. The study design, a randomized controlled trial, is considered the gold standard for minimizing bias and establishing causal relationships. Therefore, it is not appropriate to conclude that the randomization procedure was ineffective in decreasing bias based on the results presented in the table.
The most appropriate conclusion based on the results presented in the table is (B) EPCS is more effective than EST in decreasing hospital readmissions for variceal bleeding requiring transfusion. This conclusion is supported by the significantly lower mean number of readmissions for variceal bleeding requiring packed red blood cell transfusion in the EPCS group compared to the EST group.

The other conclusions are not correct for the following reasons:

(A) The 95% confidence interval for the difference in survival between EPCS and EST for Child-Pugh class A patients is not provided in the table, so it is not possible to conclude whether it includes 0 years.


(C) The median survival after EPCS is not statistically significantly different for Child-Pugh class C compared to Child-Pugh class B. In fact, the median survival is significantly higher for both Child-Pugh class B and C in the EPCS group compared to the EST group.


(D) The randomization procedure is not discussed in the abstract, so it is not possible to conclude whether it was effective in decreasing bias in this study.
28
A 42-year-old man comes to the office as a new patient. Three weeks ago, he was hospitalized for treatment of community-acquired pneumonia and diagnosed with HIV infection. He has no other history of serious illness and takes no medications. Vital signs are within normal limits. Examination shows no abnormalities. CD4+ T-lymphocyte count is 490/mm³, and plasma HIV viral load is 45,000 copies/mL. Results of other laboratory studies are within the reference ranges. Test results for syphilis and hepatitis B and C are negative. The patient agrees to begin antiretroviral therapy. He asks what his long-term prognosis will be. With adherence to the medication regimen, this patient is at greatest risk of death from which of the following?
(A) Cardiovascular disease
(B) Chronic kidney disease
(C) HIV-related malignancy
(D) Non–HIV-related malignancy
(E) Opportunistic infection
(A) Cardiovascular diseaseCorrect
The correct choice is (A) Cardiovascular disease.

Explanation:
With adherence to the medication regimen, this patient is at greatest risk of death from cardiovascular disease. HIV infection itself may accelerate atherosclerosis, and the incidence of coronary artery disease (eg, myocardial infarction [MI]) is higher in patients with HIV compared with patients without HIV who are matched for age and gender. Lower CD4 cell counts and higher HIV RNA levels have been associated with increased MI risk. Although certain antiretroviral agents may be associated with an increased risk of MI (eg, abacavir, most protease inhibitors), most ART regimens do not increase the risk of MI, whereas untreated HIV does. Therefore, it is important for patients with HIV to be monitored for cardiovascular risk factors and receive appropriate management to reduce their risk of cardiovascular disease.
It is difficult to predict the long-term prognosis for an individual with HIV infection, as it can vary greatly depending on a number of factors, including the person's age, overall health, and the effectiveness of their antiretroviral therapy. However, with proper treatment and adherence to their medication regimen, this patient's risk of developing AIDS and other serious complications, such as opportunistic infections, may be greatly reduced. It is important for this patient to continue to receive regular medical care and monitoring, and to take their medications as prescribed, in order to maintain their health and reduce their risk of complications. In general, individuals with HIV are at an increased risk of death from opportunistic infections, HIV-related malignancies, and other complications related to the immune system. The correct answer is therefore (E) Opportunistic infection.
29
A 42-year-old man comes to the office as a new patient. Three weeks ago, he was hospitalized for treatment of community-acquired pneumonia and diagnosed with HIV infection. He has no other history of serious illness and takes no medications. Vital signs are within normal limits. Examination shows no abnormalities. CD4+ T-lymphocyte count is 490/mm³, and plasma HIV viral load is 45,000 copies/mL. Results of other laboratory studies are within the reference ranges. Test results for syphilis and hepatitis B and C are negative. The patient agrees to begin antiretroviral therapy. He asks what his long-term prognosis will be. With adherence to the medication regimen, this patient is at greatest risk of death from which of the following? Discuss why the other choices are not the correct answer.
(A) Cardiovascular disease
(B) Chronic kidney disease
(C) HIV-related malignancy
(D) Non–HIV-related malignancy
(E) Opportunistic infection
(A) Cardiovascular diseaseCorrect
The correct answer is (A) Cardiovascular disease.

Explanation:
With adherence to the medication regimen, this patient is at greatest risk of death from cardiovascular disease. HIV infection itself may accelerate atherosclerosis, and the incidence of coronary artery disease (eg, myocardial infarction [MI]) is higher in patients with HIV compared with patients without HIV who are matched for age and gender. Lower CD4 cell counts and higher HIV RNA levels have been associated with increased MI risk. Although certain antiretroviral agents may be associated with an increased risk of MI (eg, abacavir, most protease inhibitors), most ART regimens do not increase the risk of MI, whereas untreated HIV does.

The other choices are not the correct answer because:
(B) Chronic kidney disease: While HIV infection can lead to kidney disease, with adherence to antiretroviral therapy, the risk of developing chronic kidney disease is significantly reduced. Antiretroviral therapy has been shown to improve kidney function and decrease the risk of kidney disease progression in HIV-infected individuals.

(C) HIV-related malignancy: With adherence to antiretroviral therapy, the risk of developing HIV-related malignancies, such as Kaposi's sarcoma and non-Hodgkin lymphoma, is significantly reduced. Antiretroviral therapy helps to restore immune function and decrease the risk of developing these malignancies.

(D) Non-HIV-related malignancy: While HIV infection may slightly increase the risk of certain non-HIV-related malignancies, such as lung cancer and anal cancer, the risk is not significantly higher compared to the general population. With adherence to antiretroviral therapy, the risk of developing non-HIV-related malignancies is not significantly increased.

(E) Opportunistic infection: With adherence to antiretroviral therapy, the risk of developing opportunistic infections is significantly reduced. Antiretroviral therapy helps to restore immune function and decrease the risk of developing opportunistic infections such as Pneumocystis jirovecii pneumonia and disseminated Mycobacterium avium complex infection.
The correct answer is (E) Opportunistic infection.

Individuals with HIV are at an increased risk of death from opportunistic infections, which are infections that occur more frequently or are more severe in individuals with weakened immune systems. These infections can be caused by a variety of microorganisms, such as bacteria, fungi, and viruses, and can affect any organ system in the body. With proper treatment and adherence to their medication regimen, this patient's risk of developing opportunistic infections may be greatly reduced.

The other choices are not the correct answer because, although individuals with HIV may be at an increased risk of developing cardiovascular disease, chronic kidney disease, HIV-related malignancies, and non-HIV-related malignancies, these conditions are not typically the primary cause of death in individuals with HIV who are receiving appropriate treatment and have good adherence to their medication regimen. It is important for this patient to continue to receive regular medical care and monitoring in order to detect and manage any potential complications.
30
A 40-year-old divorced woman comes to the office because of a 1-year history of difficulty sleeping. She says she sometimes does not fall asleep before 2 AM and does not feel rested at 6:30 AM when she has to get up and get her children ready for school. Sometimes she wakes after 1 to 2 hours of sleep and cannot fall back to sleep. On these occasions, she typically gets up for about 30 minutes, does some housework, and smokes a cigarette. She smokes one pack of cigarettes every three to four days. She works in a customer service call center and says she enjoys helping people resolve their problems. She now finds her job much harder and less enjoyable because she feels tired and sleepy. She says she would take a nap if she could, but she picks her children up as soon as she leaves work and spends the evening doing homework with them, making their dinner, and preparing lunches for the following day. By 9 PM, the children are in bed so she eats her own dinner and does some housework. She says that after 9 PM is when she smokes most of her cigarettes. She has not had depressed mood, excessive worry, or change in weight. Vital signs are within normal limits. Examination shows dark circles under the eyes and injected conjunctivae. Which of the following is the most appropriate initial step in management?
(A) Education on sleep hygiene
(B) Mirtazapine therapy
(C) Sertraline therapy
(D) Setting a date for smoking cessation
(E) Zolpidem therapy
(A) Education on sleep hygieneCorrect
The correct choice is (A) Education on sleep hygiene.

Explanation:
Based on the patient's history and symptoms, it is likely that she is experiencing insomnia. Insomnia is characterized by difficulty falling asleep, staying asleep, or waking up too early, resulting in poor quality sleep and daytime fatigue. In this case, the patient reports difficulty falling asleep before 2 AM, waking up after 1 to 2 hours of sleep, and feeling tired and sleepy during the day.

The first step in managing insomnia is to provide education on sleep hygiene. Sleep hygiene refers to a set of practices and habits that promote good sleep. By educating the patient on sleep hygiene, she can learn strategies to improve her sleep quality and address her insomnia symptoms.

Some examples of sleep hygiene recommendations that may be appropriate for this patient include:

1. Establishing a consistent sleep schedule: Encouraging the patient to go to bed and wake up at the same time every day, even on weekends, can help regulate her sleep-wake cycle.

2. Creating a sleep-friendly environment: Advising the patient to make her bedroom dark, quiet, and cool, and to use comfortable bedding and pillows can promote better sleep.

3. Avoiding stimulating activities before bedtime: Recommending that the patient avoids engaging in stimulating activities, such as housework or smoking, close to bedtime can help her wind down and prepare for sleep.

4. Limiting caffeine and nicotine intake: Educating the patient about the stimulating effects of caffeine and nicotine and advising her to avoid consuming these substances, especially in the evening, can improve her sleep quality.

5. Promoting relaxation techniques: Suggesting relaxation techniques, such as deep breathing exercises or progressive muscle relaxation, can help the patient relax and prepare for sleep.

By implementing these sleep hygiene recommendations, the patient may be able to improve her sleep quality and reduce her daytime fatigue, ultimately improving her overall well-being and job performance.
The most appropriate initial step in management would be to assess the woman's sleep habits and to determine whether she may have a sleep disorder such as insomnia. This can be done through a detailed sleep history and a physical examination. It may also be helpful to ask about potential contributing factors such as stress, caffeine and alcohol use, and medication use. Based on the information provided, it is also possible that the woman's cigarette smoking may be contributing to her sleep difficulties, and addressing this issue may be an important part of her treatment plan. Therefore, the most appropriate initial step in management would be (A) education on sleep hygiene, which can help the woman improve her sleep habits and potentially improve her sleep quality.
31
A 40-year-old divorced woman comes to the office because of a 1-year history of difficulty sleeping. She says she sometimes does not fall asleep before 2 AM and does not feel rested at 6:30 AM when she has to get up and get her children ready for school. Sometimes she wakes after 1 to 2 hours of sleep and cannot fall back to sleep. On these occasions, she typically gets up for about 30 minutes, does some housework, and smokes a cigarette. She smokes one pack of cigarettes every three to four days. She works in a customer service call center and says she enjoys helping people resolve their problems. She now finds her job much harder and less enjoyable because she feels tired and sleepy. She says she would take a nap if she could, but she picks her children up as soon as she leaves work and spends the evening doing homework with them, making their dinner, and preparing lunches for the following day. By 9 PM, the children are in bed so she eats her own dinner and does some housework. She says that after 9 PM is when she smokes most of her cigarettes. She has not had depressed mood, excessive worry, or change in weight. Vital signs are within normal limits. Examination shows dark circles under the eyes and injected conjunctivae. Which of the following is the most appropriate initial step in management? Explain why the other steps are not to be recommended as initial steps.
(A) Education on sleep hygiene
(B) Mirtazapine therapy
(C) Sertraline therapy
(D) Setting a date for smoking cessation
(E) Zolpidem therapy
(A) Education on sleep hygieneCorrect
The most appropriate initial step in management for this patient is (A) Education on sleep hygiene.

Explanation:
Based on the patient's history and symptoms, it is evident that she is experiencing insomnia. Insomnia is a common sleep disorder characterized by difficulty falling asleep, staying asleep, or both. In this case, the patient has difficulty falling asleep before 2 AM, wakes up after 1 to 2 hours of sleep and cannot fall back to sleep, and does not feel rested in the morning.

Education on sleep hygiene is the most appropriate initial step in management because it focuses on non-pharmacological interventions that can improve sleep quality and quantity. Sleep hygiene refers to a set of practices and habits that promote good sleep. By educating the patient on sleep hygiene, she can learn strategies to improve her sleep and address the underlying causes of her insomnia.

The other options are not the most appropriate initial steps in management for this patient:

(B) Mirtazapine therapy: Mirtazapine is an antidepressant that can be used off-label for insomnia. However, it is not the most appropriate initial step in management for this patient because there is no indication of depression or other psychiatric illness in her history. Additionally, pharmacological therapy should be considered after non-pharmacological interventions have been tried and failed.

(C) Sertraline therapy: Sertraline is another antidepressant that can be used off-label for insomnia. However, as mentioned above, there is no indication of depression or other psychiatric illness in this patient's history. Therefore, sertraline therapy is not the most appropriate initial step in management.

(D) Setting a date for smoking cessation: While smoking can contribute to sleep disturbances, setting a date for smoking cessation is not the most appropriate initial step in management for this patient. The primary concern at this point is addressing her insomnia and improving her sleep quality. Smoking cessation can be addressed as part of a comprehensive treatment plan, but it should not be the initial focus.

(E) Zolpidem therapy: Zolpidem is a sedative-hypnotic medication that can be used to treat insomnia. However, it is not the most appropriate initial step in management for this patient. Pharmacological therapy should be considered after non-pharmacological interventions have been tried and failed. Additionally, zolpidem therapy may not address the underlying causes of the patient's insomnia and may have potential side effects and risks associated with its use.
The most appropriate initial step in management would be to assess the woman's sleep habits and to determine whether she may have a sleep disorder such as insomnia. This can be done through a detailed sleep history and a physical examination. It may also be helpful to ask about potential contributing factors such as stress, caffeine and alcohol use, and medication use. Based on the information provided, it is also possible that the woman's cigarette smoking may be contributing to her sleep difficulties, and addressing this issue may be an important part of her treatment plan. Therefore, the most appropriate initial step in management would be (A) education on sleep hygiene, which can help the woman improve her sleep habits and potentially improve her sleep quality.

The other options are not recommended as initial steps in management because:

(B) Mirtazapine is an antidepressant medication that may be used to treat insomnia, but it is not typically the first-line treatment for this condition. It may be used if the woman has other symptoms of depression or anxiety that are contributing to her sleep difficulties.

(C) Sertraline is an antidepressant medication that may be used to treat insomnia, but it is not typically the first-line treatment for this condition. It may be used if the woman has other symptoms of depression or anxiety that are contributing to her sleep difficulties.

(D) Setting a date for smoking cessation may be an important part of the woman's treatment plan, but it should not be the initial step in management. It is important to assess the woman's sleep habits and determine whether she has a sleep disorder before addressing her cigarette smoking.

(E) Zolpidem is a medication that is used to treat insomnia, but it is not typically the first-line treatment for this condition. It may be used if the woman has persistent insomnia that is not improving with education on sleep hygiene and other non-pharmacological interventions.
32
A 58-year-old woman is brought to the office by her husband because of a 6-week history of increasing confusion. Her husband says that, initially, she had difficulty doing “normal” things, such as preparing meals. Since then, she has become progressively less able to care for herself or even use a telephone. During this time, she also has had increased anxiety and restlessness. She has type 2 diabetes mellitus well controlled with an oral hypoglycemic agent. She is alert and appears anxious. She is oriented to person but not to place or time. She startles when the door to the examination room is slammed shut. Her temperature is 36.8°C (98.2°F), pulse is 78/min and regular, and blood pressure is 132/94 mm Hg. Physical examination shows no other abnormalities. On neurologic examination, she can repeat three objects immediately but is unable to recall them after 3 minutes. She is unable to copy a drawing of a cube. Results of a complete blood count and serum electrolyte concentrations are within the reference ranges. Her serum glucose concentration is 155 mg/dL. CT scan of the head shows no abnormalities. Which of the following is the most appropriate next step in diagnosis?
(A) Determination of hemoglobin A1c
(B) Measurement of cerebrospinal fluid 14-3-3 protein concentration
(C) Measurement of cerebrospinal fluid beta amyloid concentration
(D) Measurement of cerebrospinal fluid methylmalonic acid concentration
(E) Measurement of serum angiotensin-converting enzyme activity
(F) Serum protein electrophoresis
(B) Measurement of cerebrospinal fluid 14-3-3 protein concentration
Correct
The correct answer is (B) Measurement of cerebrospinal fluid 14-3-3 protein concentration.

Explanation:
The patient's symptoms of progressive confusion, difficulty with daily activities, anxiety, restlessness, and impaired memory are suggestive of a neurodegenerative disorder. The inability to recall three objects after 3 minutes and the inability to copy a drawing of a cube are indicative of cognitive impairment. The normal results of a complete blood count, serum electrolyte concentrations, and CT scan of the head suggest that the cause of the patient's symptoms is not related to a systemic illness or structural brain abnormality.

Measurement of cerebrospinal fluid (CSF) 14-3-3 protein concentration is the most appropriate next step in diagnosis. CSF 14-3-3 protein is a marker of neuronal damage and is elevated in several neurodegenerative disorders, including Creutzfeldt-Jakob disease (CJD). CJD is a rapidly progressive, fatal neurodegenerative disorder characterized by cognitive decline, behavioral changes, and myoclonus. The startle response observed in this patient is consistent with myoclonus. Measurement of CSF 14-3-3 protein concentration can help confirm the diagnosis of CJD.

Determination of hemoglobin A1c (A) is not indicated in this patient because her serum glucose concentration is well controlled at 155 mg/dL. Hemoglobin A1c is a marker of long-term glycemic control and is used to assess diabetes management.

Measurement of cerebrospinal fluid beta amyloid concentration (C) is not indicated in this patient because beta amyloid is associated with Alzheimer disease, which typically presents with a more insidious onset and a gradual decline in cognitive function. The patient's symptoms and rapid progression are not consistent with Alzheimer disease.

Measurement of cerebrospinal fluid methylmalonic acid concentration (D) is not indicated in this patient because methylmalonic acid is associated with vitamin B12 deficiency, which typically presents with neurologic symptoms such as peripheral neuropathy and subacute combined degeneration. The patient's symptoms and normal serum electrolyte concentrations make vitamin B12 deficiency unlikely.

Measurement of serum angiotensin-converting enzyme activity (E) is not indicated in this patient because it is used to evaluate for sarcoidosis, which typically presents with pulmonary symptoms and findings. The patient's symptoms and normal physical examination findings do not suggest sarcoidosis.

Serum protein electrophoresis (F) is not indicated in this patient because her symptoms and physical examination findings are not suggestive of a paraproteinemia or multiple myeloma, which are conditions that can be evaluated with serum protein electrophoresis.
Based on the information provided, the patient may be experiencing cognitive decline and confusion, which may be indicative of a neurological disorder. In order to confirm the diagnosis and determine the most appropriate course of treatment, further testing is necessary. The most appropriate next step in diagnosis would be a determination of hemoglobin A1c (option A) to assess the patient's blood sugar levels, as uncontrolled diabetes can cause cognitive decline and confusion. Other possible causes of the symptoms described, such as infection or inflammation, should also be ruled out.*
33
A 58-year-old woman is brought to the office by her husband because of a 6-week history of increasing confusion. Her husband says that, initially, she had difficulty doing “normal” things, such as preparing meals. Since then, she has become progressively less able to care for herself or even use a telephone. During this time, she also has had increased anxiety and restlessness. She has type 2 diabetes mellitus well controlled with an oral hypoglycemic agent. She is alert and appears anxious. She is oriented to person but not to place or time. She startles when the door to the examination room is slammed shut. Her temperature is 36.8°C (98.2°F), pulse is 78/min and regular, and blood pressure is 132/94 mm Hg. Physical examination shows no other abnormalities. On neurologic examination, she can repeat three objects immediately but is unable to recall them after 3 minutes. She is unable to copy a drawing of a cube. Results of a complete blood count and serum electrolyte concentrations are within the reference ranges. Her serum glucose concentration is 155 mg/dL. CT scan of the head shows no abnormalities. Which of the following is the most appropriate next step in diagnosis and why not the other choices?
(A) Determination of hemoglobin A1c
(B) Measurement of cerebrospinal fluid 14-3-3 protein concentration
(C) Measurement of cerebrospinal fluid beta amyloid concentration
(D) Measurement of cerebrospinal fluid methylmalonic acid concentration
(E) Measurement of serum angiotensin-converting enzyme activity
(F) Serum protein electrophoresis
(B) Measurement of cerebrospinal fluid 14-3-3 protein concentration
Correct
The correct choice is (B) Measurement of cerebrospinal fluid 14-3-3 protein concentration.

The patient in this case presents with a 6-week history of increasing confusion, difficulty with daily activities, anxiety, restlessness, and memory impairment. She also has type 2 diabetes mellitus, which is well controlled. The physical examination and laboratory tests, including a CT scan of the head, do not reveal any abnormalities.

Given the clinical presentation and the absence of other findings, the most appropriate next step in diagnosis is to measure the cerebrospinal fluid (CSF) 14-3-3 protein concentration. CSF 14-3-3 protein is a marker of neuronal damage and is elevated in certain neurodegenerative disorders, including Creutzfeldt-Jakob disease (CJD). CJD is a rare, rapidly progressive, and fatal neurodegenerative disorder characterized by cognitive decline, behavioral changes, and neurological symptoms. The clinical features, including the rapid progression of symptoms, cognitive impairment, and neurological findings, are consistent with CJD.

The other choices are not appropriate in this case:
(A) Determination of hemoglobin A1c: Hemoglobin A1c is a marker of long-term glucose control and is not relevant to the patient's current symptoms.
(C) Measurement of cerebrospinal fluid beta amyloid concentration: Beta amyloid is associated with Alzheimer's disease, which typically presents with a more gradual onset and progressive memory impairment. The patient's symptoms and clinical course are not consistent with Alzheimer's disease.
(D) Measurement of cerebrospinal fluid methylmalonic acid concentration: Methylmalonic acid is a marker of vitamin B12 deficiency, which can cause neurological symptoms. However, the patient's symptoms and clinical course are not consistent with vitamin B12 deficiency.
(E) Measurement of serum angiotensin-converting enzyme activity: Serum angiotensin-converting enzyme activity is not relevant to the patient's current symptoms.
(F) Serum protein electrophoresis: Serum protein electrophoresis is used to evaluate for abnormalities in protein levels and patterns, but it is not specific for neurodegenerative disorders and would not be the most appropriate next step in this case.
The most appropriate next step in diagnosis would be to perform a neuropsychiatric evaluation. This would involve assessing the patient's cognitive and behavioral functioning in order to determine the cause of her symptoms. This evaluation may include tests of memory, language, attention, and other cognitive abilities, as well as assessments of mood and behavior. It may also include imaging studies, such as an MRI, to evaluate the structure and function of the brain. Based on the results of the neuropsychiatric evaluation, the healthcare provider can determine the appropriate course of treatment.

The other options provided in the question are not appropriate next steps in diagnosis for the following reasons:

(A) Determination of hemoglobin A1c: Hemoglobin A1c is a measure of long-term blood sugar control and is not typically used to diagnose cognitive or behavioral disorders.

(B) Measurement of cerebrospinal fluid 14-3-3 protein concentration: The 14-3-3 protein is a marker of brain damage and is not typically used to diagnose cognitive or behavioral disorders.

(C) Measurement of cerebrospinal fluid beta amyloid concentration: Beta amyloid is a protein involved in the development of Alzheimer's disease, but it is not typically used to diagnose cognitive or behavioral disorders.

(D) Measurement of cerebrospinal fluid methylmalonic acid concentration: Methylmalonic acid is a marker of certain metabolic disorders, but it is not typically used to diagnose cognitive or behavioral disorders.

(E) Measurement of serum angiotensin-converting enzyme activity: Angiotensin-converting enzyme is involved in blood pressure regulation and is not typically used to diagnose cognitive or behavioral disorders.

(F) Serum protein electrophoresis: Protein electrophoresis is a test that separates proteins based on their size and charge and is not typically used to diagnose cognitive or behavioral disorders.
34
A 38-year-old woman is brought to the emergency department by paramedics 90 minutes after she fell through ice into a stream while hiking. She was stuck in the ice until emergency responders rescued her. On arrival, she has severe pain in her feet. She has no history of serious illness and takes no medications. Temperature is 34.8°C (94.6°F); other vital signs are within normal limits. Examination shows blue discoloration over the toes bilaterally, and the feet are cold to touch. Which of the following is the most appropriate next step in management?
(A) Place the feet in front of a dry space heater
(B) Rub the feet with a steaming hot towel
(C) Submerge the feet in a moderately warm whirlpool
(D) Wrap the feet in a room-temperature wet-to-dry dressing
(E) Observation only
(C) Submerge the feet in a moderately warm whirlpool
Correct
The correct choice is (C) Submerge the feet in a moderately warm whirlpool.

Explanation:
The patient in this scenario is presenting with symptoms consistent with frostbite, specifically nonfreezing cold water (trench foot) and warm water immersion injuries. Frostbite occurs when tissues freeze due to exposure to cold temperatures, and rewarming the affected area is a crucial step in management. The most appropriate method for rewarming frostbitten extremities is to submerge them in a moderately warm whirlpool, with the water temperature maintained between 37 to 39°C (98.6 to 102.2°F). This allows for controlled and gradual rewarming of the tissues, minimizing the risk of further damage. Rubbing the feet with a steaming hot towel (B) is not recommended as it can cause further tissue damage. Placing the feet in front of a dry space heater (A) or wrapping them in a room-temperature wet-to-dry dressing (D) are also not recommended as they do not provide controlled and gradual rewarming. Observation only (E) is not appropriate as prompt intervention is necessary to prevent further tissue damage.
The most appropriate next step in management of this patient is to warm her feet using a warming device such as a heating pad or warm water immersion. Cold exposure can cause decreased blood flow to the extremities, leading to tissue damage and potentially even tissue death. Warming the feet can help to improve blood flow and prevent further tissue damage. Therefore, the correct answer is option C, "submerge the feet in a moderately warm whirlpool." It is important to warm the feet slowly and carefully to avoid tissue damage from burns or further decreased blood flow. The patient should also be monitored for signs of hypothermia, such as shivering or confusion, and treated accordingly.
35
A 38-year-old woman is brought to the emergency department by paramedics 90 minutes after she fell through ice into a stream while hiking. She was stuck in the ice until emergency responders rescued her. On arrival, she has severe pain in her feet. She has no history of serious illness and takes no medications. Temperature is 34.8°C (94.6°F); other vital signs are within normal limits. Examination shows blue discoloration over the toes bilaterally, and the feet are cold to touch. Which of the following is the most appropriate next step in management? Provide rationale for each choice on why it is appropriate or not appopriate as a course of action.
(A) Place the feet in front of a dry space heater
(B) Rub the feet with a steaming hot towel
(C) Submerge the feet in a moderately warm whirlpool
(D) Wrap the feet in a room-temperature wet-to-dry dressing
(E) Observation only
(C) Submerge the feet in a moderately warm whirlpool
Correct
The correct choice is (C) Submerge the feet in a moderately warm whirlpool.

Rationale:
The patient's presentation is consistent with nonfreezing cold water (trench foot) injury, which occurs due to prolonged exposure to wet cold conditions. The initial management of trench foot involves rewarming the affected area. Submerging the feet in a moderately warm whirlpool is the most appropriate next step in management because it allows for controlled and gradual rewarming of the feet. The water temperature should be maintained between 37 to 39°C (98.6 to 102.2°F). This method of rewarming is preferred over other options because it provides a steady temperature and avoids the risk of burns or further tissue damage that may occur with other methods such as placing the feet in front of a dry space heater or rubbing them with a steaming hot towel. Wrapping the feet in a room-temperature wet-to-dry dressing would not provide the necessary rewarming. Observation only would delay appropriate treatment and potentially lead to further complications.
The most appropriate next step in management of this patient is to warm her feet using a warming device such as a heating pad or warm water immersion. Cold exposure can cause decreased blood flow to the extremities, leading to tissue damage and potentially even tissue death. Warming the feet can help to improve blood flow and prevent further tissue damage. Therefore, the correct answer is option C, "submerge the feet in a moderately warm whirlpool." This option will provide a gentle and controlled way to warm the feet and increase blood flow.

Option A, "place the feet in front of a dry space heater," is not appropriate because it could cause burns to the feet if the heat is not carefully controlled. Additionally, placing the feet in front of a heater does not provide any way to control the temperature of the feet and could lead to further decreased blood flow if the feet become too warm.

Option B, "rub the feet with a steaming hot towel," is not appropriate because it could cause burns to the feet if the towel is too hot. Additionally, rubbing the feet with a hot towel does not provide any way to control the temperature of the feet and could lead to further decreased blood flow if the feet become too warm.

Option D, "wrap the feet in a room-temperature wet-to-dry dressing," is not appropriate because it does not provide any way to warm the feet and increase blood flow. Additionally, a wet-to-dry dressing could cause further tissue damage if it becomes too cold.

Option E, "observation only," is not appropriate because it does not address the decreased blood flow and potential tissue damage caused by the cold exposure. It is important to take action to warm the feet and improve blood flow to prevent further tissue damage.
36
A 52-year-old woman comes to the emergency department because of a 1-day history of bleeding gums and moderate pain in her knees. She first noticed the gum bleeding while brushing her teeth this morning; she has been unable to stop the bleeding. She has systemic lupus erythematosus with inflammatory arthritis and Raynaud disease. Medications are tramadol and hydroxychloroquine. Vital signs are within normal limits. Examination shows bleeding, friable gums. There are multiple ecchymoses and petechiae over the upper extremities. Cardiac and abdominal examinations show no abnormalities. The knees are cool to the touch; there are effusions, and range of motion is limited by pain. Neurologic examination shows no focal findings. Laboratory studies show:

Hemoglobin = 11 g/dL
Hematocrit = 33%
Erythrocyte count = 4.2 × 10¹²/L
Mean corpuscular volume = 90 μm³
Leukocyte count = 9000/mm³
Segmented neutrophils = 60%
Bands = 3%
Eosinophils = 2%
Basophils = 1%
Lymphocytes = 30%
Monocytes = 4%
Platelet count = 20,000/mm³
Serum
Creatinine = 0.9 mg/dL
Ferritin 1= 0 ng/mL

Which of the following is the most appropriate next step in management?

(A) Factor replacement therapy
(B) Methotrexate therapy
(C) Plasmapheresis
(D) Prednisone therapy
(E) Splenectomy
(D) Prednisone therapyCorrect
The correct answer is (D) Prednisone therapy.

Explanation:
The patient's presentation is consistent with immune thrombocytopenic purpura (ITP), which is characterized by isolated thrombocytopenia due to platelet destruction by autoantibodies. The bleeding gums, ecchymoses, and petechiae are indicative of a low platelet count. The patient's medical history of systemic lupus erythematosus (SLE) and the presence of other autoimmune diseases (inflammatory arthritis and Raynaud disease) further support the diagnosis of ITP.

The initial management of ITP involves glucocorticoid therapy, such as prednisone, which suppresses the immune system and reduces platelet destruction. Prednisone is the treatment of choice for acute ITP and is effective in increasing platelet counts in the majority of patients. It is important to initiate treatment promptly to prevent further bleeding complications.

Factor replacement therapy (A) is not indicated in this case because the patient's bleeding is due to thrombocytopenia, not a deficiency in clotting factors. Methotrexate therapy (B) is not the first-line treatment for ITP and is typically reserved for refractory cases. Plasmapheresis (C) may be considered in severe cases of ITP with life-threatening bleeding, but it is not the initial management. Splenectomy (E) may be considered in patients who do not respond to medical therapy or have chronic ITP, but it is not the first-line treatment.
Based on the given information, the most likely diagnosis for this patient is immune thrombocytopenia (ITP), a disorder in which the immune system attacks and destroys platelets, leading to bleeding and bruising. The diagnostic criteria for ITP include a platelet count less than 100,000/mm³ and the absence of other causes for thrombocytopenia. The patient in this case has a platelet count of 20,000/mm³, which meets the diagnostic criteria for ITP.

The most appropriate next step in management would be to initiate treatment with corticosteroids or immunoglobulin, both of which can help to increase the platelet count and reduce the risk of bleeding. Options A, C, and E are not typically used for the treatment of ITP. Option B, methotrexate therapy, is sometimes used as a second-line treatment for ITP, but it is not the most appropriate first-line treatment in this case. Therefore, the most appropriate next step in management would be option D, prednisone therapy.*
37
A 52-year-old woman comes to the emergency department because of a 1-day history of bleeding gums and moderate pain in her knees. She first noticed the gum bleeding while brushing her teeth this morning; she has been unable to stop the bleeding. She has systemic lupus erythematosus with inflammatory arthritis and Raynaud disease. Medications are tramadol and hydroxychloroquine. Vital signs are within normal limits. Examination shows bleeding, friable gums. There are multiple ecchymoses and petechiae over the upper extremities. Cardiac and abdominal examinations show no abnormalities. The knees are cool to the touch; there are effusions, and range of motion is limited by pain. Neurologic examination shows no focal findings. Laboratory studies show:

Hemoglobin = 11 g/dL
Hematocrit = 33%
Erythrocyte count = 4.2 × 10¹²/L
Mean corpuscular volume = 90 μm³
Leukocyte count = 9000/mm³
Segmented neutrophils = 60%
Bands = 3%
Eosinophils = 2%
Basophils = 1%
Lymphocytes = 30%
Monocytes = 4%
Platelet count = 20,000/mm³
Serum
Creatinine = 0.9 mg/dL
Ferritin 1= 0 ng/mL

Which of the following is the most appropriate next step in management? Why are the other choices not the best step?

(A) Factor replacement therapy
(B) Methotrexate therapy
(C) Plasmapheresis
(D) Prednisone therapy
(E) Splenectomy
(D) Prednisone therapyCorrect
The correct choice is (D) Prednisone therapy.

Explanation:
The patient's presentation is consistent with immune thrombocytopenic purpura (ITP), which is characterized by isolated thrombocytopenia due to platelet destruction by autoantibodies. The bleeding gums, ecchymoses, and petechiae are all signs of increased bleeding tendency. The low platelet count of 20,000/mm³ confirms the diagnosis. In patients with ITP, treatment is indicated if bleeding symptoms are present or if the platelet count is <30,000/mm³. The initial treatment of choice for ITP is glucocorticoids, such as prednisone, which suppress the immune system and decrease platelet destruction. This is the most appropriate next step in management for this patient.

(A) Factor replacement therapy is not indicated in this patient because the low platelet count is due to immune-mediated destruction, not a deficiency of clotting factors.

(B) Methotrexate therapy is not indicated in this patient because it is used for the treatment of rheumatoid arthritis, not ITP.

(C) Plasmapheresis is not indicated in this patient because it is typically reserved for severe cases of ITP that are refractory to initial treatment with glucocorticoids.

(E) Splenectomy is not indicated as the initial treatment for ITP. It is reserved for patients who do not respond to initial treatment with glucocorticoids or who have chronic or recurrent disease.
The most appropriate next step in management would be to determine the cause of the bleeding gums and low platelet count. Based on the information provided, it is not clear which treatment option would be the most appropriate. Options A, C, D, and E may be appropriate depending on the underlying cause of the patient's symptoms, but further testing and evaluation is necessary before deciding on a specific course of treatment. The patient's systemic lupus erythematosus (SLE) and use of hydroxychloroquine may be contributing factors, and her symptoms of arthritis and Raynaud disease should also be managed.

It is not clear from the information provided whether factor replacement therapy (option A), plasmapheresis (option C), prednisone therapy (option D), or splenectomy (option E) would be appropriate in this case. These treatments may be considered if the patient has a bleeding disorder or if her low platelet count is causing her symptoms, but further testing is needed to confirm this. Methotrexate therapy (option B) may be used to treat SLE and arthritis, but it is not clear from the information provided whether this would be the most appropriate treatment in this case.
38
A 13-year-old boy is brought to the emergency department because of a 3-day history of progressive fatigue, shortness of breath, and difficulty walking up stairs. He had been well until 1 week ago, when he developed rhinorrhea, cough, and a sore throat. These symptoms resolved spontaneously 2 days ago. He has no history of serious illness and takes no medications. On arrival, he is pale and in moderate respiratory distress. Pulse is 120/min, respirations are 25/min, and blood pressure is 80/40 mm Hg. Pulse oximetry on room air shows an oxygen saturation of 91%. On pulmonary examination, crackles are heard throughout the lung fields. On cardiac examination, a gallop is heard. S1 is soft. A grade 2/6, high-pitched holosystolic murmur is heard best at the apex. The remainder of the examination shows no abnormalities. Which of the following is the most likely diagnosis?
(A) Glomerulonephritis
(B) Myocarditis
(C) Pneumonia
(D) Rheumatic fever
(E) Systemic lupus erythematosus
(B) MyocarditisCorrect
The correct answer is (B) Myocarditis.

Explanation:
The patient's symptoms of fatigue, shortness of breath, and difficulty walking up stairs, along with the physical examination findings of tachycardia, respiratory distress, crackles on pulmonary examination, and a gallop on cardiac examination, are consistent with myocarditis. Myocarditis is an inflammation of the myocardium, which can be caused by viral infections, including those that cause upper respiratory tract symptoms. The symptoms of rhinorrhea, cough, and sore throat that the patient experienced 1 week ago are suggestive of a viral upper respiratory tract infection, which could have led to the development of myocarditis. The soft S1 and the grade 2/6, high-pitched holosystolic murmur heard best at the apex are consistent with mitral regurgitation, which can occur as a result of myocarditis. The patient's low blood pressure and oxygen saturation of 91% on room air are signs of hemodynamic compromise, which can occur in severe cases of myocarditis. Glomerulonephritis (choice A) typically presents with hematuria, proteinuria, and hypertension. Pneumonia (choice C) typically presents with fever, cough, and focal findings on pulmonary examination. Rheumatic fever (choice D) typically presents with migratory polyarthritis, carditis, subcutaneous nodules, erythema marginatum, and Sydenham chorea. Systemic lupus erythematosus (choice E) typically presents with a variety of symptoms, including fatigue, joint pain, rash, and renal involvement.
Based on the symptoms and physical examination findings described in the question, the most likely diagnosis is congestive heart failure, which is not included in the options provided. Congestive heart failure can cause fatigue, shortness of breath, difficulty walking, and crackles in the lungs, as well as a gallop rhythm and a heart murmur on examination. However, without more information it is difficult to provide a definitive diagnosis. It is important for the patient to receive further evaluation and treatment from a healthcare provider.
39
A 13-year-old boy is brought to the emergency department because of a 3-day history of progressive fatigue, shortness of breath, and difficulty walking up stairs. He had been well until 1 week ago, when he developed rhinorrhea, cough, and a sore throat. These symptoms resolved spontaneously 2 days ago. He has no history of serious illness and takes no medications. On arrival, he is pale and in moderate respiratory distress. Pulse is 120/min, respirations are 25/min, and blood pressure is 80/40 mm Hg. Pulse oximetry on room air shows an oxygen saturation of 91%. On pulmonary examination, crackles are heard throughout the lung fields. On cardiac examination, a gallop is heard. S1 is soft. A grade 2/6, high-pitched holosystolic murmur is heard best at the apex. The remainder of the examination shows no abnormalities. Which of the following is the most likely diagnosis? Why are the other choices not likely the answer?
(A) Glomerulonephritis
(B) Myocarditis
(C) Pneumonia
(D) Rheumatic fever
(E) Systemic lupus erythematosus
(B) MyocarditisCorrect
The correct answer is (B) Myocarditis.

Explanation:
Based on the patient's symptoms and physical examination findings, the most likely diagnosis is myocarditis. Myocarditis is an inflammation of the myocardium, the muscular tissue of the heart. It can be caused by viral infections, such as the recent upper respiratory tract infection in this patient. The symptoms of fatigue, shortness of breath, and difficulty walking up stairs are consistent with myocardial dysfunction and heart failure. The pale appearance and moderate respiratory distress further support this diagnosis.

The physical examination findings of a gallop rhythm, soft S1, and a grade 2/6, high-pitched holosystolic murmur at the apex are indicative of myocardial dysfunction and possible valvular involvement. The crackles heard throughout the lung fields suggest pulmonary congestion, which is commonly seen in heart failure.

The other choices are less likely based on the clinical presentation and findings:

(A) Glomerulonephritis: Glomerulonephritis typically presents with symptoms such as hematuria, proteinuria, and hypertension. The patient in this case does not have any urinary symptoms or signs of renal dysfunction.

(C) Pneumonia: While the patient did have respiratory symptoms (rhinorrhea, cough, sore throat), the resolution of these symptoms 2 days ago and the presence of crackles throughout the lung fields suggest a cardiac rather than a pulmonary cause for the symptoms.

(D) Rheumatic fever: Rheumatic fever is a possible complication of untreated streptococcal pharyngitis, but the absence of joint pain, erythema marginatum, and subcutaneous nodules makes this diagnosis less likely.

(E) Systemic lupus erythematosus: Systemic lupus erythematosus can present with a wide range of symptoms, including cardiac involvement. However, the acute onset of symptoms following an upper respiratory tract infection and the absence of other characteristic features of lupus make this diagnosis less likely.
Based on the symptoms and physical examination findings described in the question, the most likely diagnosis is congestive heart failure. This condition can cause fatigue, shortness of breath, difficulty walking, and crackles in the lungs, as well as a gallop rhythm and a heart murmur on examination.

The other options are not as likely because they do not typically cause the symptoms and examination findings described in the question. Glomerulonephritis is a type of kidney disease that can cause swelling, high blood pressure, and protein in the urine, but it typically does not cause the symptoms described in the question. Myocarditis is an inflammation of the heart muscle that can cause chest pain, shortness of breath, and abnormal heart rhythms, but it typically does not cause the symptoms described in the question. Pneumonia is an infection of the lungs that can cause cough, fever, and difficulty breathing, but it typically does not cause the symptoms described in the question. Rheumatic fever is a complication of strep throat that can cause fever, joint pain, and heart valve damage, but it typically does not cause the symptoms described in the question. Systemic lupus erythematosus is an autoimmune disorder that can cause a wide range of symptoms, including fatigue, joint pain, and rash, but it typically does not cause the symptoms described in the question.
40
A 10-day-old female newborn is brought to the emergency department because of a 2-day history of bilious vomiting. She was born at term following an uncomplicated pregnancy and spontaneous vaginal delivery. She was 50.8 cm (20 in) in length and weighed 3260 g (7 lb 3 oz) at birth. Examination following delivery showed no abnormalities, and she was discharged at the age of 2 days. She is exclusively breast-fed. She appears ill. Today, she is 50.8 cm (20 in) in length and weighs 3062 g (6 lb 12 oz). Temperature is 38.3°C (100.9°F), pulse is 180/min, and respirations are 52/min. The abdomen is distended; the patient cries on palpation of the abdomen. Which of the following is the most likely diagnosis?
(A) Duodenal atresia
(B) Intussusception
(C) Meconium ileus
(D) Necrotizing enterocolitis
(E) Volvulus
(E) VolvulusCorrect
The correct answer is (E) Volvulus.

Explanation:
In this scenario, the newborn presents with bilious vomiting, which is a concerning symptom. Bilious vomiting may indicate a bowel obstruction distal to the ligament of Treitz. The presence of an ill-appearing infant with abdominal distension and tenderness on palpation further supports the diagnosis of a bowel obstruction.

Among the given options, volvulus is the most likely diagnosis. Volvulus refers to the twisting of a segment of the bowel around its mesentery, leading to obstruction of the blood supply and subsequent bowel ischemia. It commonly occurs in infants with malrotation, a congenital abnormality where the intestines do not properly rotate and fixate during fetal development.

The clinical presentation of volvulus includes bilious vomiting, abdominal distension, and tenderness. The infant may also have signs of systemic illness, such as fever and tachycardia. The absence of stooling or infrequent stooling suggests bowel obstruction.

Duodenal atresia is a congenital condition characterized by the absence or narrowing of the duodenal lumen. It typically presents with non-bilious vomiting and is unlikely in this case.

Intussusception is the telescoping of one segment of the intestine into another, causing obstruction. It commonly occurs in infants between 3 months and 3 years of age and presents with intermittent episodes of abdominal pain, currant jelly-like stools, and a palpable abdominal mass. It is less likely in this 10-day-old newborn.

Meconium ileus is a condition seen in infants with cystic fibrosis, where the meconium (first stool) is abnormally thick and obstructs the intestines. It typically presents with failure to pass meconium, abdominal distension, and bilious vomiting. However, the absence of respiratory symptoms and the presence of an ill-appearing infant make this diagnosis less likely.

Necrotizing enterocolitis (NEC) is a serious condition that primarily affects premature infants. It is characterized by inflammation and necrosis of the intestinal mucosa. NEC typically presents with feeding intolerance, abdominal distension, and bloody stools. The absence of prematurity and the presence of an ill-appearing infant make NEC less likely in this case.

Therefore, based on the clinical presentation and the absence of other risk factors, volvulus is the most likely diagnosis in this 10-day-old newborn with bilious vomiting, abdominal distension, and tenderness.
Based on the information provided, the most likely diagnosis is (B) intussusception. Intussusception is a condition in which a portion of the intestine folds into itself, obstructing the flow of food and causing symptoms such as abdominal pain and vomiting. It is most commonly seen in infants between the ages of 3 months and 6 months, and is more common in males. The other options listed do not align with the symptoms and characteristics described in the question. Duodenal atresia is a congenital condition in which the duodenum, the part of the small intestine that connects to the stomach, is blocked or absent. Meconium ileus is a condition in which the infant's first bowel movement, known as meconium, becomes stuck in the intestine and blocks the flow of food. Necrotizing enterocolitis is a condition in which the tissue of the intestine becomes damaged and dies, typically as a result of infection or other underlying condition. Volvulus is a condition in which a loop of the intestine becomes twisted, obstructing the flow of food and potentially leading to a loss of blood supply to the affected area.
41
A 10-day-old female newborn is brought to the emergency department because of a 2-day history of bilious vomiting. She was born at term following an uncomplicated pregnancy and spontaneous vaginal delivery. She was 50.8 cm (20 in) in length and weighed 3260 g (7 lb 3 oz) at birth. Examination following delivery showed no abnormalities, and she was discharged at the age of 2 days. She is exclusively breast-fed. She appears ill. Today, she is 50.8 cm (20 in) in length and weighs 3062 g (6 lb 12 oz). Temperature is 38.3°C (100.9°F), pulse is 180/min, and respirations are 52/min. The abdomen is distended; the patient cries on palpation of the abdomen. Which of the following is the most likely diagnosis? Why are the other choices not the diagnosis?

(A) Duodenal atresia
(B) Intussusception
(C) Meconium ileus
(D) Necrotizing enterocolitis
(E) Volvulus
(E) VolvulusCorrect
The correct answer is (E) Volvulus.

Volvulus is the most likely diagnosis in this case because the patient presents with bilious vomiting, abdominal distension, and an ill appearance. Volvulus refers to the twisting of a segment of the intestine, which can lead to obstruction and compromised blood supply. This can result in symptoms such as bilious vomiting, abdominal distension, and signs of systemic illness like fever and tachycardia.

The other choices can be ruled out based on the clinical presentation and additional information provided:

(A) Duodenal atresia typically presents with non-bilious vomiting and is usually diagnosed in the first few days of life. The patient in this case is 10 days old and has bilious vomiting.

(B) Intussusception is more common in infants between 3 months and 6 years of age. It typically presents with intermittent episodes of abdominal pain, currant jelly-like stools, and a palpable sausage-shaped mass. The patient in this case is only 10 days old and does not have these characteristic findings.

(C) Meconium ileus is commonly associated with cystic fibrosis and presents with failure to pass meconium, abdominal distension, and bilious vomiting. However, the patient in this case is exclusively breast-fed and does not have a history suggestive of cystic fibrosis.

(D) Necrotizing enterocolitis (NEC) is a condition that primarily affects premature infants. It is characterized by abdominal distension, feeding intolerance, and signs of systemic illness. However, the patient in this case was born at term and does not have a history of prematurity.

In summary, based on the clinical presentation and the absence of findings suggestive of other conditions, volvulus is the most likely diagnosis in this 10-day-old female newborn.
Based on the information provided, the most likely diagnosis is (B) intussusception. Intussusception is a condition in which a portion of the intestine folds into itself, obstructing the flow of food and causing symptoms such as abdominal pain and vomiting. It is most commonly seen in infants between the ages of 3 months and 6 months, and is more common in males. The other options listed do not align with the symptoms and characteristics described in the question.

(A) Duodenal atresia is a congenital condition in which the duodenum, the part of the small intestine that connects to the stomach, is blocked or absent. This condition typically presents with vomiting and failure to pass meconium in the first few days of life, whereas the patient in the question is 10 days old and has already been discharged from the hospital.

(C) Meconium ileus is a condition in which the infant's first bowel movement, known as meconium, becomes stuck in the intestine and blocks the flow of food. This typically presents in the first few days of life, whereas the patient in the question is 10 days old.

(D) Necrotizing enterocolitis is a condition in which the tissue of the intestine becomes damaged and dies, typically as a result of infection or other underlying condition. This condition typically presents with symptoms such as feeding intolerance, abdominal distension, and blood in the stool, whereas the patient in the question is exclusively breast-fed and has no reported blood in the stool.

(E) Volvulus is a condition in which a loop of the intestine becomes twisted, obstructing the flow of food and potentially leading to a loss of blood supply to the affected area. This condition typically presents with symptoms such as vomiting, abdominal pain, and distension, but it is typically seen in older infants and children, not 10-day-old newborns.
42
An 85-year-old man with severe dementia, Alzheimer type, is admitted to the hospital because of a nonhealing ulcer
on his right second toe. He lives in a nursing care facility and has been bedridden for the past 3 years. He has type 2
diabetes mellitus, hypertension, and peripheral vascular disease. His medications are metformin, amlodipine, and
aspirin. Noninvasive vascular studies of the lower extremities show marked bilateral vascular disease. Subsequent
peripheral angiography shows severe stenoses of the right superficial femoral and the left common iliac arteries. The
patient undergoes uncomplicated placement of two sequential vascular stents in the superficial femoral artery and
placement of an additional stent in the left common iliac artery. The type of stents used was recently approved by
the US Food and Drug Administration and has been heavily marketed as a long-lasting solution to restenosis. Each
stent costs the hospital $15,000. Alternatives to this approach include amputation of the toe, balloon angioplasty,
and use of less expensive stents. Which of the following best describes the public policy implications of this
scenario?


(A) Absence of resource stewardship
(B) Ageism
(C) Health care disparities
(D) Health care fraud and abuse
(E) Lack of access to care
(A) Absence of resource stewardshipCorrect
The correct choice is (A) Absence of resource stewardship.

Explanation: The scenario describes a situation where an 85-year-old man with severe dementia and multiple comorbidities undergoes a costly procedure involving the placement of vascular stents for the treatment of a nonhealing ulcer. The cost of each stent is $15,000, and there are alternative, less expensive treatment options available such as amputation of the toe, balloon angioplasty, or the use of less expensive stents.

The public policy implications of this scenario can be best described as the absence of resource stewardship. Resource stewardship refers to the responsible and efficient use of healthcare resources to achieve the best possible outcomes for patients. In this case, the decision to use expensive stents without considering the cost-effectiveness or exploring alternative, less expensive treatment options suggests a lack of resource stewardship. This decision may have implications for the allocation of healthcare resources and the overall cost of healthcare.

Ageism (B) refers to discrimination or prejudice based on age, and it is not applicable in this scenario as the decision to use expensive stents is not based on the patient's age.

Healthcare disparities (C) refer to differences in access to healthcare and health outcomes among different populations. While this scenario involves a patient with multiple comorbidities and a nonhealing ulcer, it does not specifically address disparities in healthcare access or outcomes.

Healthcare fraud and abuse (D) refers to intentional deception or misrepresentation in healthcare for personal gain. There is no indication of fraud or abuse in this scenario.

Lack of access to care (E) refers to barriers that prevent individuals from accessing necessary healthcare services. While access to care may be a concern for some patients, it is not the primary issue in this scenario. The patient is admitted to the hospital and undergoes a procedure, indicating access to care. The focus of the scenario is on the decision-making process and the use of expensive resources.
(A) Absence of resource stewardship is the best answer choice that describes the public policy implications of the scenario. This scenario involves using expensive, newly approved stents to treat the patient's vascular disease, which may not be the most cost-effective or appropriate treatment option. Using these stents in this situation may be seen as a lack of resource stewardship, as the hospital is using valuable resources without considering the potential benefits and drawbacks of using these stents in this specific case.
43
An 85-year-old man with severe dementia, Alzheimer type, is admitted to the hospital because of a nonhealing ulcer
on his right second toe. He lives in a nursing care facility and has been bedridden for the past 3 years. He has type 2
diabetes mellitus, hypertension, and peripheral vascular disease. His medications are metformin, amlodipine, and
aspirin. Noninvasive vascular studies of the lower extremities show marked bilateral vascular disease. Subsequent
peripheral angiography shows severe stenoses of the right superficial femoral and the left common iliac arteries. The
patient undergoes uncomplicated placement of two sequential vascular stents in the superficial femoral artery and
placement of an additional stent in the left common iliac artery. The type of stents used was recently approved by
the US Food and Drug Administration and has been heavily marketed as a long-lasting solution to restenosis. Each
stent costs the hospital $15,000. Alternatives to this approach include amputation of the toe, balloon angioplasty,
and use of less expensive stents. Which of the following best describes the public policy implications of this
scenario? Why is it the best answer and the others are not?


(A) Absence of resource stewardship
(B) Ageism
(C) Health care disparities
(D) Health care fraud and abuse
(E) Lack of access to care
(A) Absence of resource stewardshipCorrect
The correct answer is (A) Absence of resource stewardship.

Explanation:
The scenario described in the question highlights the use of expensive vascular stents in a patient with severe vascular disease and a nonhealing ulcer. The use of these stents, which cost $15,000 each, may not be the most cost-effective or efficient approach to managing the patient's condition. This raises concerns about the absence of resource stewardship, which refers to the responsible and efficient use of healthcare resources.

The other options are not the best answers because they do not accurately reflect the public policy implications of the scenario:

(B) Ageism: Ageism refers to discrimination or prejudice based on a person's age. While the patient in this scenario is elderly, there is no evidence to suggest that age played a role in the decision to use expensive stents.

(C) Health care disparities: Health care disparities refer to differences in access to and quality of healthcare based on factors such as race, ethnicity, socioeconomic status, or geographic location. While disparities in healthcare may exist in general, the scenario does not provide any information to suggest that disparities are specifically implicated in this case.

(D) Health care fraud and abuse: Health care fraud and abuse involve intentional deception or misrepresentation for personal gain within the healthcare system. There is no evidence to suggest that fraud or abuse is occurring in this scenario.

(E) Lack of access to care: Lack of access to care refers to barriers that prevent individuals from obtaining necessary healthcare services. While access to care may be a concern in some cases, the scenario does not provide any information to suggest that the patient in this case lacks access to care.
(A) Absence of resource stewardship is the best answer because this scenario suggests that the hospital chose to use expensive stents without considering other, potentially more cost-effective options, such as balloon angioplasty or the use of less expensive stents. This decision may not be in the best interest of the patient or the hospital, and could be considered a lack of resource stewardship.

The other options are not the best answer because they do not accurately describe the situation in the scenario. Ageism refers to discrimination against individuals based on their age, but there is no evidence of this in the scenario. Health care disparities refer to inequalities in the quality of health care that people receive, but this is not mentioned in the scenario. Health care fraud and abuse involves fraudulent or illegal behavior in the health care system, but there is no mention of this in the scenario. Lack of access to care refers to a situation in which individuals are unable to obtain necessary health care services, but this is not mentioned in the scenario.
44
A 62-year-old woman with myasthenia gravis comes to the office because of a 2-week history of falling. She says
her legs "give out" once daily; she falls to the ground but does not hit her head. She took pyridostigmine until
2 weeks ago when she decided that it no longer worked. Respirations are 30/min. Examination shows a hypophonic
nasal voice and ptosis bilaterally. Muscle strength is 2/5 in all extremities. The patient is told that she is in imminent
danger of respiratory compromise and that she should be admitted to the hospital. She says she knows her breathing
could pose a problem, but she hates hospitals and does not want to undergo more tests or therapy. She asks if she
can have a prescription for her former medication and if she can go home. Which of the following is the most
appropriate next step in management?

(A) Assessment of medical decision-making capacity
(B) Electromyography and nerve conduction studies
(C) Involuntary hospital admission
(D) Oxygen therapy by nasal canula
(E) Pulmonary function testing
(A) Assessment of medical decision-making capacity
Correct
The correct choice is (A) Assessment of medical decision-making capacity.

Explanation:
In this scenario, the patient with myasthenia gravis is at risk of respiratory compromise and needs to be admitted to the hospital. However, she refuses to undergo further tests or therapy and wants to go home. Before making any decisions about her care, it is important to assess her medical decision-making capacity.

Assessment of medical decision-making capacity involves evaluating the patient's ability to understand the relevant information, appreciate the potential consequences of their decision, reason through the options, and communicate their choice. This assessment is crucial in situations where the patient's decision may have serious implications for their health and well-being.

In this case, the patient's refusal to be admitted to the hospital despite the risk of respiratory compromise suggests a potential lack of capacity to make informed decisions. It is important to determine if the patient fully understands the severity of her condition and the potential consequences of her decision. If she lacks capacity, further steps can be taken to ensure her safety and well-being, such as involving a surrogate decision-maker or pursuing involuntary hospital admission if necessary.

Therefore, the most appropriate next step in management is to assess the patient's medical decision-making capacity.
The most appropriate next step in the management of this patient would be an assessment of her medical decision-making capacity. Myasthenia gravis is a autoimmune disorder that affects the ability of the muscles to respond to nerve impulses, causing weakness and rapid fatigue. In severe cases, the weakness can affect the muscles involved in breathing, putting the patient at risk of respiratory compromise. In this case, the patient's symptoms suggest that she is at risk of respiratory failure and should be admitted to the hospital for further evaluation and treatment. However, the patient has expressed her reluctance to undergo further testing and treatment, and it is important to determine whether she has the capacity to make decisions about her own care. If she does not have the capacity to make decisions about her own care, involuntary hospital admission may be necessary to protect her from harm.
45
A 62-year-old woman with myasthenia gravis comes to the office because of a 2-week history of falling. She says
her legs "give out" once daily; she falls to the ground but does not hit her head. She took pyridostigmine until
2 weeks ago when she decided that it no longer worked. Respirations are 30/min. Examination shows a hypophonic
nasal voice and ptosis bilaterally. Muscle strength is 2/5 in all extremities. The patient is told that she is in imminent
danger of respiratory compromise and that she should be admitted to the hospital. She says she knows her breathing
could pose a problem, but she hates hospitals and does not want to undergo more tests or therapy. She asks if she
can have a prescription for her former medication and if she can go home. Which of the following is the most
appropriate next step in management? Why are the other choices not the best step?

(A) Assessment of medical decision-making capacity
(B) Electromyography and nerve conduction studies
(C) Involuntary hospital admission
(D) Oxygen therapy by nasal canula
(E) Pulmonary function testing
(A) Assessment of medical decision-making capacity
Correct
The correct choice is (A) Assessment of medical decision-making capacity.

Explanation:
In this scenario, the patient with myasthenia gravis is at risk of respiratory compromise and needs to be admitted to the hospital. However, the patient refuses to be admitted and requests a prescription for her former medication to go home. Before making any decisions regarding the patient's treatment, it is important to assess her medical decision-making capacity.

Assessment of medical decision-making capacity involves evaluating the patient's ability to understand the relevant information, appreciate the potential consequences of her decision, reason through the decision-making process, and communicate her decision. This assessment is crucial in determining whether the patient has the capacity to make informed decisions about her own healthcare.

The other choices are not the best step in this situation:

(B) Electromyography and nerve conduction studies: While these tests can help confirm the diagnosis of myasthenia gravis, they are not the most appropriate next step in management when the patient is at risk of respiratory compromise. Immediate intervention is needed to address the patient's respiratory distress.

(C) Involuntary hospital admission: Involuntary hospital admission should be considered as a last resort when a patient's decision-making capacity is impaired and they are at risk of harm to themselves or others. However, before resorting to involuntary admission, it is important to assess the patient's decision-making capacity and explore other options for ensuring their safety.

(D) Oxygen therapy by nasal cannula: While oxygen therapy may be necessary for respiratory support in patients with myasthenia gravis, it is not the most appropriate next step in management when the patient is at risk of respiratory compromise. The patient's respiratory distress requires more immediate intervention.

(E) Pulmonary function testing: Pulmonary function testing may be useful in evaluating respiratory function in patients with myasthenia gravis, but it is not the most appropriate next step in management when the patient is in imminent danger of respiratory compromise. Immediate intervention is needed to address the patient's respiratory distress.

In summary, the most appropriate next step in management for this patient is to assess her medical decision-making capacity before making any decisions regarding her treatment.
In this situation, the most appropriate next step in management is involuntary hospital admission. The patient is in imminent danger of respiratory compromise and should be admitted to the hospital for further evaluation and treatment. The other options, including assessment of medical decision-making capacity, electromyography and nerve conduction studies, oxygen therapy by nasal canula, and pulmonary function testing, may be appropriate at some point, but they are not the most urgent or important steps in this situation.
46
A 55-year-old man is brought to the emergency department by paramedics 45 minutes after his daughter found him
unresponsive in his apartment. Paramedics initiated intravenous fluid warming and resuscitation at the scene. The
patient has major depressive disorder, hypertension, and alcoholism. His medications are chlordiazepoxide,
fluoxetine, and lisinopril. On arrival, he is arousable but confused. Temperature is 34.0°C (93.2°F), pulse is 64/min,
respirations are 14/min, and blood pressure is 94/62 mm Hg. Physical examination shows no sign of trauma or
prolonged immobilization. On mental status examination, he is oriented to person but not to place or time. His
speech is verbose and nonsensical. Serum glucose concentration is 44 mg/dL, and blood alcohol concentration is
312 mg/dL. Urine toxicology screening is positive for alcohol and benzodiazepines. The most appropriate next step
in management is administration of which of the following?


(A) 50% Dextrose
(B) Flumazenil
(C) Lorazepam
(D) Naloxone
(E) Vitamin B1 (thiamine)
(E) Vitamin B1 (thiamine)Correct
The correct choice is (E) Vitamin B1 (thiamine).

Explanation:
The patient's presentation is consistent with Wernicke encephalopathy, which is caused by thiamine deficiency. Thiamine is an essential cofactor for glucose metabolism, and alcoholism is a common cause of thiamine deficiency. The patient's low body temperature, confusion, nonsensical speech, and low serum glucose concentration are all consistent with Wernicke encephalopathy. Administration of thiamine is the most appropriate next step in management to prevent further neurologic deterioration. Thiamine should be administered before glucose to prevent precipitation of Wernicke encephalopathy. Therefore, the correct choice is (E) Vitamin B1 (thiamine).
The most appropriate next step in management would be to administer 50% Dextrose, which is a concentrated form of glucose. This is because the patient's serum glucose concentration is 44 mg/dL, which is significantly lower than the normal range (typically 70-100 mg/dL). Low blood sugar, or hypoglycemia, can cause symptoms such as confusion and loss of consciousness, which the patient is exhibiting. Administering 50% Dextrose can help to quickly raise the patient's blood sugar levels and improve their symptoms. Additionally, the patient's high blood alcohol concentration and positive urine toxicology for alcohol and benzodiazepines suggest that alcohol and benzodiazepine poisoning may be contributing to their condition, and Dextrose can also help to treat these types of poisoning.


47
A 55-year-old man is brought to the emergency department by paramedics 45 minutes after his daughter found him
unresponsive in his apartment. Paramedics initiated intravenous fluid warming and resuscitation at the scene. The
patient has major depressive disorder, hypertension, and alcoholism. His medications are chlordiazepoxide,
fluoxetine, and lisinopril. On arrival, he is arousable but confused. Temperature is 34.0°C (93.2°F), pulse is 64/min,
respirations are 14/min, and blood pressure is 94/62 mm Hg. Physical examination shows no sign of trauma or
prolonged immobilization. On mental status examination, he is oriented to person but not to place or time. His
speech is verbose and nonsensical. Serum glucose concentration is 44 mg/dL, and blood alcohol concentration is
312 mg/dL. Urine toxicology screening is positive for alcohol and benzodiazepines. The most appropriate next step
in management is administration of which of the following? Why are the other choices not the best fit?


(A) 50% Dextrose
(B) Flumazenil
(C) Lorazepam
(D) Naloxone
(E) Vitamin B1 (thiamine)
(E) Vitamin B1 (thiamine)Correct
The correct choice is (E) Vitamin B1 (thiamine).

Explanation:
The patient's presentation is consistent with Wernicke encephalopathy, which is caused by thiamine deficiency. Thiamine is an essential cofactor for glucose metabolism, and alcoholism is a common cause of thiamine deficiency. The patient's low body temperature, confusion, nonsensical speech, and low serum glucose concentration are all consistent with Wernicke encephalopathy. Administration of thiamine is the most appropriate next step in management to prevent further neurologic deterioration.

The other choices are not the best fit for this patient's presentation:

(A) 50% Dextrose: While the patient has a low serum glucose concentration, administration of dextrose alone without thiamine can worsen Wernicke encephalopathy. Thiamine should be administered before or concurrently with dextrose to prevent precipitation of Wernicke encephalopathy.

(B) Flumazenil: Flumazenil is a benzodiazepine receptor antagonist and is used to reverse the effects of benzodiazepine overdose. However, in this patient, the benzodiazepine detected in the urine toxicology screening is likely due to chronic use rather than acute overdose. Flumazenil is not indicated in the management of Wernicke encephalopathy.

(C) Lorazepam: Lorazepam is a benzodiazepine and is not indicated in the management of Wernicke encephalopathy. In fact, benzodiazepines can worsen the symptoms of Wernicke encephalopathy and should be avoided.

(D) Naloxone: Naloxone is a opioid receptor antagonist and is used to reverse the effects of opioid overdose. There is no evidence to suggest opioid overdose in this patient, and naloxone is not indicated in the management of Wernicke encephalopathy.
The most appropriate next step in management would be administration of 50% dextrose, as the patient's serum glucose concentration is 44 mg/dL, which is significantly low. This can cause confusion and other symptoms, such as those the patient is experiencing. Administering dextrose can help increase the patient's blood sugar levels and improve their symptoms.

The other options are not the best fit because:

Flumazenil is a benzodiazepine antagonist and would not be useful in this situation, as the patient's symptoms are not related to benzodiazepine use.
Lorazepam is a benzodiazepine that is used to treat anxiety and other conditions, but it would not be the best choice in this situation, as the patient's symptoms are not related to anxiety.
Naloxone is used to reverse the effects of opioid drugs, but the patient's symptoms are not related to opioid use.
Vitamin B1 (thiamine) is important for maintaining healthy nerve function, but it would not be the most appropriate choice in this situation, as the patient's symptoms are due to low blood sugar rather than a deficiency of thiamine.
48
A 16-year-old boy is brought to the emergency department because of a 6-hour history of severe right-sided scrotal
pain. He has not had penile discharge, fever, abdominal pain, vomiting, or diarrhea. He has no history of similar pain
or any trauma to the region. He is sexually active with one female partner and uses condoms inconsistently. Vital
signs are within normal limits. Abdominal examination shows no abnormalities. Genitourinary examination shows
moderate edema and diffuse tenderness to palpation of the right scrotum; there is mild erythema. Lifting the right
testicle does not relieve the pain. The left testicle is lower than the right testicle. Cremasteric reflex is present on the
left and absent on the right. Which of the following is the most likely diagnosis?


(A) Epididymitis
(B) Testicular torsion
(C) Testicular tumor
(D) Torsion of the appendix testis
(E) Urinary tract infection
(F) Varicocele
(B) Testicular torsionCorrect
The correct choice is (B) Testicular torsion.

Explanation:
Based on the given information, the most likely diagnosis is testicular torsion. Testicular torsion is a urologic emergency that occurs when the spermatic cord twists, leading to compromised blood flow to the testicle. It commonly presents with sudden onset severe scrotal pain, often with no apparent cause or history of trauma. The pain is usually unilateral and can be associated with scrotal swelling, tenderness, and erythema. In testicular torsion, lifting the affected testicle does not relieve the pain. The absence of the cremasteric reflex (elevation of the testis in response to stroking of the upper inner thigh) on the affected side is a characteristic finding in testicular torsion. In this case, the absence of the cremasteric reflex on the right side supports the diagnosis of testicular torsion. It is important to note that testicular torsion can occur even without the classic presentation of abdominal pain. Prompt surgical intervention is necessary to salvage the affected testicle and prevent complications such as testicular infarction.
Based on the given symptoms, the most likely diagnosis is testicular torsion. This condition is characterized by sudden, severe pain in the scrotum, often accompanied by swelling, tenderness, and a feeling of heaviness. The pain may be severe enough to cause nausea and vomiting. In testicular torsion, the testicle rotates on its axis, twisting the spermatic cord and cutting off the blood supply to the testicle. This can cause permanent damage to the testicle if not treated promptly. The key symptoms to look for in this case are the sudden onset of severe pain, tenderness to palpation, and the absence of the cremasteric reflex on the affected side.


49
A 16-year-old boy is brought to the emergency department because of a 6-hour history of severe right-sided scrotal
pain. He has not had penile discharge, fever, abdominal pain, vomiting, or diarrhea. He has no history of similar pain
or any trauma to the region. He is sexually active with one female partner and uses condoms inconsistently. Vital
signs are within normal limits. Abdominal examination shows no abnormalities. Genitourinary examination shows
moderate edema and diffuse tenderness to palpation of the right scrotum; there is mild erythema. Lifting the right
testicle does not relieve the pain. The left testicle is lower than the right testicle. Cremasteric reflex is present on the
left and absent on the right. Which of the following is the most likely diagnosis? Why are the other choices not the diagnosis?


(A) Epididymitis
(B) Testicular torsion
(C) Testicular tumor
(D) Torsion of the appendix testis
(E) Urinary tract infection
(F) Varicocele
(B) Testicular torsionCorrect
The correct choice is (B) Testicular torsion.

Explanation:
Based on the given information, the most likely diagnosis is testicular torsion. Testicular torsion is a urologic emergency that occurs when the spermatic cord twists, leading to compromised blood flow to the testicle. It commonly presents with sudden onset severe scrotal pain, often with no apparent cause or history of trauma. The pain is usually unilateral and can be associated with scrotal swelling, tenderness, and erythema. In testicular torsion, lifting the affected testicle does not relieve the pain. The absence of the cremasteric reflex (elevation of the testis in response to stroking of the upper inner thigh) on the affected side is a characteristic finding in testicular torsion. In this case, the cremasteric reflex is absent on the right side. The left testicle being lower than the right testicle is also a finding consistent with testicular torsion.

The other choices are not the most likely diagnosis based on the given information:

(A) Epididymitis: Epididymitis is inflammation of the epididymis, which is a structure located behind the testicle. It is commonly caused by a bacterial infection and presents with gradual onset scrotal pain, swelling, and tenderness. It is often associated with urinary symptoms such as dysuria and frequency. In epididymitis, the cremasteric reflex is usually present.

(C) Testicular tumor: Testicular tumors are rare in adolescents and typically present as painless testicular masses. They are not typically associated with sudden onset severe scrotal pain.

(D) Torsion of the appendix testis: Torsion of the appendix testis is a benign condition that occurs when the appendix testis, a small structure attached to the upper pole of the testicle, twists. It can cause localized tenderness and swelling, but it is less painful than testicular torsion. The cremasteric reflex is usually intact in torsion of the appendix testis.

(E) Urinary tract infection: Urinary tract infections typically present with urinary symptoms such as dysuria, frequency, and urgency. They are not typically associated with sudden onset severe scrotal pain.

(F) Varicocele: Varicoceles are dilated veins in the scrotum and are typically painless. They are not typically associated with sudden onset severe scrotal pain.
Based on the given information, the most likely diagnosis is testicular torsion. This condition is characterized by sudden onset of severe scrotal pain, often accompanied by swelling and tenderness. The absence of the cremasteric reflex on the affected side and the lower position of the left testicle are additional signs consistent with this diagnosis.

The other options are not the diagnosis because:

(A) Epididymitis is an infection of the epididymis, a tube that carries sperm from the testicle. It typically presents with fever, penile discharge, and pain that is relieved by lifting the testicle.

(C) Testicular tumors are relatively rare in adolescents and typically present with a painless mass in the testicle, rather than acute pain.

(D) Torsion of the appendix testis is a rare condition that typically presents with acute pain and swelling in the upper scrotum.

(E) Urinary tract infections are typically accompanied by symptoms such as frequent or painful urination, abdominal pain, and fever.

(F) Varicoceles are dilated veins in the scrotum that can cause swelling and discomfort, but they typically do not cause acute pain.
50
A 64-year-old man comes to the clinic because of a 2-week history of severe pain and generalized muscle weakness.
He has hypercholesteremia and hypertension. Three months ago, he had a myocardial infarction. At that time, he
began atorvastatin therapy. Other medications are clopidogrel, lisinopril, and amiodarone. He is 178 cm (5 ft 10 in)
tall and weighs 82 kg (180 lb); BMI is 26 kg/m²

Pulse is 80/min and blood pressure is 125/67 mm Hg. Muscle
strength is 4/5 on abduction of the shoulders and flexion of the hips. Deep tendon reflexes are 2+ throughout. No
other abnormalities are noted. Measurement of which of the following is the most appropriate initial step in
diagnosis?

(A) Erythrocyte sedimentation rate
(B) Serum ALT activity
(C) Serum calcium concentration
(D) Serum C-reactive protein concentration
(E) Serum creatine kinase activity
(E) Serum creatine kinase activityCorrect
The correct choice is (E) Serum creatine kinase activity.

Explanation:
The patient's symptoms of severe pain and generalized muscle weakness, along with the physical examination findings of decreased muscle strength and normal deep tendon reflexes, are suggestive of a myopathy. The most appropriate initial step in diagnosing a myopathy is to measure serum creatine kinase (CK) activity. CK is an enzyme found predominantly in skeletal muscle, and elevated levels of CK in the blood indicate muscle damage or injury. In this patient, the use of atorvastatin, a statin medication, is a potential cause of the myopathy. Statins can cause muscle-related side effects, including myalgias and myopathy, which can be associated with elevated CK levels. Therefore, measuring serum CK activity is the most appropriate initial step in diagnosing the cause of the patient's symptoms. The other options listed (A) Erythrocyte sedimentation rate, (B) Serum ALT activity, (C) Serum calcium concentration, and (D) Serum C-reactive protein concentration, are not specific tests for diagnosing a myopathy and would not provide useful information in this case.
The most appropriate initial step in diagnosis for this patient would be to measure their serum creatine kinase activity. This is because the patient has a history of muscle weakness and the symptoms they are experiencing may be related to a muscle injury or condition. Elevated levels of creatine kinase in the blood can indicate muscle damage, which can help guide further diagnostic testing and treatment.


51
A 64-year-old man comes to the clinic because of a 2-week history of severe pain and generalized muscle weakness.
He has hypercholesteremia and hypertension. Three months ago, he had a myocardial infarction. At that time, he
began atorvastatin therapy. Other medications are clopidogrel, lisinopril, and amiodarone. He is 178 cm (5 ft 10 in)
tall and weighs 82 kg (180 lb); BMI is 26 kg/m²

Pulse is 80/min and blood pressure is 125/67 mm Hg. Muscle
strength is 4/5 on abduction of the shoulders and flexion of the hips. Deep tendon reflexes are 2+ throughout. No
other abnormalities are noted. Measurement of which of the following is the most appropriate initial step in
diagnosis? Why is the answer the best choice and not the others?

(A) Erythrocyte sedimentation rate
(B) Serum ALT activity
(C) Serum calcium concentration
(D) Serum C-reactive protein concentration
(E) Serum creatine kinase activity
(E) Serum creatine kinase activityCorrect
The correct choice is (E) Serum creatine kinase activity.

Explanation:
The patient's presentation of severe pain and generalized muscle weakness, along with the history of atorvastatin therapy, suggests the possibility of statin-induced myopathy. Statins, such as atorvastatin, are known to cause muscle-related side effects, including myalgia (muscle pain) and myopathy (muscle weakness). Measurement of serum creatine kinase (CK) activity is the most appropriate initial step in diagnosing statin-induced myopathy.

CK is an enzyme found in high concentrations in muscle tissue. When muscle tissue is damaged, CK is released into the bloodstream. Elevated serum CK levels indicate muscle damage or injury. In the context of this patient's symptoms and history of atorvastatin therapy, an elevated CK level would support the diagnosis of statin-induced myopathy.

The other options are not as relevant in this case:

(A) Erythrocyte sedimentation rate (ESR) is a non-specific marker of inflammation and is not typically elevated in statin-induced myopathy.

(B) Serum ALT activity is a marker of liver function and is not directly related to muscle damage or myopathy.

(C) Serum calcium concentration is not typically affected in statin-induced myopathy.

(D) Serum C-reactive protein (CRP) concentration is another marker of inflammation, but it is not specific to muscle damage or myopathy.

Therefore, the most appropriate initial step in diagnosis in this case is to measure serum creatine kinase activity.
The most appropriate initial step in diagnosis would be to measure the serum creatine kinase activity. Creatine kinase (CK) is an enzyme that is released into the blood when there is muscle damage or breakdown. Elevated CK levels can indicate muscle damage or breakdown, which may be the cause of the patient's symptoms. In this case, the patient's history of a myocardial infarction and the presence of muscle weakness and pain suggest that muscle damage may be present. Measurement of CK activity would therefore be the most appropriate initial step in diagnosing the cause of the patient's symptoms.
52
A 23-year-old man who is on active duty in the US Army comes to the clinic 1 hour after sustaining a bee sting to
his right cheek. The patient reports continued pain at the site of the sting, but he has not had shortness of breath,
throat tightness, loss of vision, or other neurologic symptoms. Medical history is unremarkable. He takes no
medications. The patient has no known allergies. He is 178 cm (5 ft 10 in) tall and weighs 64 kg (140 lb); BMI is 20
kg/m². Temperature is 37.0°C (98.6°F), pulse is 86/min, respirations are 10/min, and blood pressure is
120/70 mm Hg. Examination of the right cheek shows an erythematous, swollen area of induration surrounding a
central clear area with the stinger in situ. The swelling encroaches on the lower aspect of the patient's right orbit,
which appears puffy and mildly dusky. After the stinger is removed, which of the following is the most appropriate
next step in management?


(A) Administer an injection of epinephrine
(B) Admit the patient to the hospital for intravenous vancomycin therapy
(C) Cleanse the site and apply ice
(D) Obtain an ophthalmologist consultation
(E) Prescribe oral clindamycin
(C) Cleanse the site and apply iceCorrect
The correct choice is (C) Cleanse the site and apply ice.

Explanation:
Based on the information provided, the patient has a local reaction to the bee sting, characterized by erythema, swelling, and pain at the site of the sting. There are no signs of anaphylaxis or systemic involvement, such as shortness of breath, throat tightness, or neurologic symptoms. Therefore, the most appropriate next step in management is to cleanse the site and apply ice.

Cleansing the site helps to remove any remaining venom and reduce the risk of infection. Applying ice can help reduce pain and swelling by constricting blood vessels and decreasing inflammation. These measures are sufficient for managing local reactions to bee stings in most cases.

Administering an injection of epinephrine (choice A) is indicated for the treatment of anaphylaxis, which is not present in this case. Admitting the patient to the hospital for intravenous vancomycin therapy (choice B) is not necessary as there are no signs of infection. Obtaining an ophthalmologist consultation (choice D) is not indicated at this time as there are no signs of ocular involvement. Prescribing oral clindamycin (choice E) is not necessary as there are no signs of infection.
The most appropriate next step in management is to cleanse the site and apply ice. Epinephrine is used to treat severe allergic reactions (anaphylaxis), but the patient does not have any symptoms of anaphylaxis, such as shortness of breath or throat tightness. Intravenous vancomycin is an antibiotic that is not indicated in this situation. Obtaining an ophthalmologist consultation is not necessary at this time, as the patient's symptoms are localized to the site of the bee sting and do not involve any loss of vision. Oral clindamycin may be prescribed if there is evidence of infection at the site of the sting, but this should be determined after the site is cleansed and any necessary measures, such as applying ice, are taken.
53
A 23-year-old man who is on active duty in the US Army comes to the clinic 1 hour after sustaining a bee sting to
his right cheek. The patient reports continued pain at the site of the sting, but he has not had shortness of breath,
throat tightness, loss of vision, or other neurologic symptoms. Medical history is unremarkable. He takes no
medications. The patient has no known allergies. He is 178 cm (5 ft 10 in) tall and weighs 64 kg (140 lb); BMI is 20
kg/m². Temperature is 37.0°C (98.6°F), pulse is 86/min, respirations are 10/min, and blood pressure is
120/70 mm Hg. Examination of the right cheek shows an erythematous, swollen area of induration surrounding a
central clear area with the stinger in situ. The swelling encroaches on the lower aspect of the patient's right orbit,
which appears puffy and mildly dusky. After the stinger is removed, which of the following is the most appropriate
next step in management? Why is the answer the best choice and not the others?


(A) Administer an injection of epinephrine
(B) Admit the patient to the hospital for intravenous vancomycin therapy
(C) Cleanse the site and apply ice
(D) Obtain an ophthalmologist consultation
(E) Prescribe oral clindamycin
(C) Cleanse the site and apply iceCorrect
The correct choice is (C) Cleanse the site and apply ice.

Explanation:
In this scenario, the patient presents with a local reaction to a bee sting on his right cheek. The area is erythematous, swollen, and indurated, with the stinger still in situ. The swelling encroaches on the lower aspect of the right orbit, which appears puffy and mildly dusky. The patient does not have any systemic symptoms of an allergic reaction, such as shortness of breath, throat tightness, or neurologic symptoms.

The most appropriate next step in management is to cleanse the site and apply ice. This is because the patient is experiencing a local reaction, which is a common response to a bee sting. Cleansing the site helps to prevent infection, and applying ice can help reduce pain and swelling.

Administering an injection of epinephrine (choice A) is not indicated in this case because the patient does not have any signs or symptoms of anaphylaxis, which is a severe allergic reaction. Epinephrine is reserved for the treatment of anaphylaxis.

Admitting the patient to the hospital for intravenous vancomycin therapy (choice B) is not necessary as there is no evidence of infection at the site of the sting. The patient does not have any systemic signs of infection, such as fever or chills.

Obtaining an ophthalmologist consultation (choice D) is not necessary at this time. Although the swelling encroaches on the lower aspect of the right orbit, the patient does not have any visual changes or significant eye symptoms.

Prescribing oral clindamycin (choice E) is not indicated as there is no evidence of infection at the site of the sting. The patient does not have any systemic signs of infection, and the local reaction can be managed with cleansing and ice application.
The most appropriate next step in management would be to cleanse the site and apply ice. This is the best choice because it will help to reduce swelling and pain at the site of the sting. Administering an injection of epinephrine, admitting the patient to the hospital for intravenous vancomycin therapy, and prescribing oral clindamycin are not appropriate in this case, as the patient does not have any symptoms that would warrant those interventions. Obtaining an ophthalmologist consultation may be necessary if the patient's symptoms worsen, but it is not the most appropriate next step in this situation.
54
A 32-year-old man is brought to the emergency department after being found by his wife in their bedroom with a
gun in his hands. His wife reports that the patient “has not been himself” since their son was stillborn 6 months ago.
She says that during the past month, he hardly has slept and has not showered or shaved; he sits in the dark and
refuses to eat. He began fluoxetine therapy 3 months ago, but his symptoms have continued to worsen. He appears
disheveled. He is 175 cm (5 ft 9 in) tall and weighs 52 kg (115 lb); BMI is 17 kg/m². His pulse is 90/min, and blood
pressure is 100/60 mm Hg. Physical examination shows decreased skin turgor and dry mucous membranes. When asked about his mood, he says, “I don’t know.” He has a tearful affect. He responds to most questions with one-word answers in a soft voice. He says he hears a voice telling him that he is a failure and says he wants to join his son in heaven. He refuses to answer questions about suicide. After removing all firearms from the patient’s house and admitting him to a psychiatric facility, which of the following is the most appropriate next step in management?


(A) Electroconvulsive therapy
(B) Imipramine therapy
(C) Mirtazapine therapy
(D) Sertraline therapy
(E) Transcranial magnetic stimulation
(A) Electroconvulsive therapyCorrect
The correct choice is (A) Electroconvulsive therapy.

Explanation:
This patient is presenting with severe depressive symptoms, including significant weight loss, insomnia, anhedonia, psychomotor retardation, and suicidal ideation. He has also failed to respond to fluoxetine therapy. Given the severity of his symptoms and the presence of suicidal ideation, electroconvulsive therapy (ECT) is the most appropriate next step in management.

ECT is a highly effective treatment for severe depression, especially when there is a risk of suicide or when other treatments have failed. It involves the induction of a generalized seizure under anesthesia, which leads to the release of neurotransmitters and the modulation of neuronal circuits involved in mood regulation. ECT has a rapid onset of action and can provide relief from depressive symptoms within a few sessions.

Imipramine (B), mirtazapine (C), and sertraline (D) are all antidepressant medications that can be used in the treatment of depression. However, given the severity of this patient's symptoms and the lack of response to fluoxetine, ECT is a more appropriate choice at this time.

Transcranial magnetic stimulation (E) is a non-invasive procedure that uses magnetic fields to stimulate specific areas of the brain. It is typically used as a second-line treatment for depression when medications have failed. However, given the severity of this patient's symptoms and the presence of suicidal ideation, ECT would be a more appropriate choice.
The most appropriate next step in management would be to begin antidepressant therapy, such as with a selective serotonin reuptake inhibitor (SSRI) like sertraline. The patient's symptoms, including his tearful affect, disheveled appearance, and reports of hearing a voice telling him that he is a failure, suggest that he may be experiencing depression. Antidepressant therapy, combined with counseling and support, can help improve his mood and symptoms. Electroconvulsive therapy, transcranial magnetic stimulation, and imipramine therapy are not typically first-line treatments for depression and should only be considered if other treatments are not effective. Mirtazapine therapy may be an option if the patient does not respond to an SSRI, but it is not the most appropriate next step in management.
55
A 32-year-old man is brought to the emergency department after being found by his wife in their bedroom with a
gun in his hands. His wife reports that the patient “has not been himself” since their son was stillborn 6 months ago.
She says that during the past month, he hardly has slept and has not showered or shaved; he sits in the dark and
refuses to eat. He began fluoxetine therapy 3 months ago, but his symptoms have continued to worsen. He appears
disheveled. He is 175 cm (5 ft 9 in) tall and weighs 52 kg (115 lb); BMI is 17 kg/m². His pulse is 90/min, and blood
pressure is 100/60 mm Hg. Physical examination shows decreased skin turgor and dry mucous membranes. When asked about his mood, he says, “I don’t know.” He has a tearful affect. He responds to most questions with one-word answers in a soft voice. He says he hears a voice telling him that he is a failure and says he wants to join his son in heaven. He refuses to answer questions about suicide. After removing all firearms from the patient’s house and admitting him to a psychiatric facility, which of the following is the most appropriate next step in management? Why is the answer the best choice and not the others?


(A) Electroconvulsive therapy
(B) Imipramine therapy
(C) Mirtazapine therapy
(D) Sertraline therapy
(E) Transcranial magnetic stimulation
(A) Electroconvulsive therapyCorrect
The correct choice is (A) Electroconvulsive therapy.

Explanation:
The patient in this scenario is presenting with severe depressive symptoms, including significant weight loss, insomnia, anhedonia, psychomotor retardation, and suicidal ideation. These symptoms have persisted despite treatment with fluoxetine, indicating treatment resistance. Additionally, the patient's physical examination findings of decreased skin turgor and dry mucous membranes suggest severe dehydration, which may be due to his refusal to eat and drink.

Electroconvulsive therapy (ECT) is the most appropriate next step in management for this patient. ECT is a highly effective treatment for severe depression, especially in cases of treatment resistance or when rapid response is needed. It is particularly indicated in patients with severe depressive symptoms, psychotic features, or high suicide risk, as in this case. ECT works by inducing a generalized seizure through the administration of an electrical current, leading to changes in brain chemistry and improved mood.

The other options are not the best choices for this patient:

(B) Imipramine therapy: Imipramine is a tricyclic antidepressant that may be effective in treating depression. However, it is not the best choice in this case because the patient has not responded to fluoxetine, which is also an antidepressant. Additionally, tricyclic antidepressants have a higher risk of side effects and toxicity compared to newer antidepressants.

(C) Mirtazapine therapy: Mirtazapine is an antidepressant that can be effective in treating depression, especially when associated with insomnia and poor appetite. However, it is not the best choice in this case because the patient's symptoms are severe and he has not responded to fluoxetine. ECT would be a more appropriate and effective treatment option.

(D) Sertraline therapy: Sertraline is a selective serotonin reuptake inhibitor (SSRI) that is commonly used to treat depression. However, it is not the best choice in this case because the patient has not responded to fluoxetine, which is also an SSRI. ECT would be a more appropriate and effective treatment option.

(E) Transcranial magnetic stimulation (TMS): TMS is a non-invasive procedure that uses magnetic fields to stimulate nerve cells in the brain. It is an effective treatment option for depression, especially in cases of treatment resistance. However, it is not the best choice in this case because the patient's symptoms are severe and he has not responded to fluoxetine. ECT would be a more appropriate and effective treatment option.
Based on the information provided, the most appropriate next step in management is to initiate antidepressant therapy. Given the patient's history of fluoxetine therapy, it would be appropriate to try a different antidepressant, such as sertraline or mirtazapine. Electroconvulsive therapy (ECT), transcranial magnetic stimulation (TMS), and imipramine therapy are not typically used as first-line treatments for depression and may not be appropriate for this patient. It is important to carefully monitor the patient's symptoms and response to treatment, and to involve the patient's family in the treatment plan to provide support and ensure the patient's safety.
56
A 60-year-old woman comes to the office as a new patient. She recently moved to the area. She feels well. She has
hypertension treated with amlodipine and asthma well controlled with inhaled fluticasone and inhaled albuterol as
needed. According to the medical records she brought with her, a mammography 6 months ago showed no
abnormalities, results of laboratory studies 1 year ago were within the reference ranges, and colonoscopy 5 years ago
showed no abnormalities. Her past three Pap smears have shown no abnormalities; the most recent Pap smear was
1 year ago. Her mother has type 2 diabetes mellitus, and her maternal aunt had breast cancer at the age of 55 years.
The patient does not smoke cigarettes. She drinks two glasses of wine weekly. She works as a nurse. She has been
monogamous with her husband for 30 years. Pulse is 76/min, respirations are 16/min, and blood pressure is
132/86 mm Hg. Examination shows no abnormalities. Screening for which of the following is the most appropriate
next step in management?


(A) Abdominal aortic aneurysm
(B) Breast cancer
(C) Chlamydia trachomatis infection
(D) Colon cancer
(E) Hepatitis C
(E) Hepatitis CCorrect
The correct answer is (E) Hepatitis C.

Explanation:
Based on the patient's history and risk factors, screening for Hepatitis C is the most appropriate next step in management. The patient has a history of hypertension and asthma, which are not risk factors for abdominal aortic aneurysm, breast cancer, or colon cancer. Additionally, there is no indication for screening for Chlamydia trachomatis infection in this patient.

However, the patient's history of being a nurse and potential exposure to bloodborne pathogens, such as Hepatitis C, makes screening for Hepatitis C important. Hepatitis C is a viral infection that primarily affects the liver and is transmitted through contact with infected blood. Healthcare workers, such as nurses, are at an increased risk of exposure to Hepatitis C due to potential needlestick injuries or contact with contaminated blood.

Screening for Hepatitis C involves a blood test to detect the presence of antibodies against the virus. If the test is positive, further testing may be done to determine the viral load and assess liver function. Early detection of Hepatitis C is important as it can lead to chronic liver disease, cirrhosis, and hepatocellular carcinoma if left untreated.

Therefore, given the patient's risk factors and potential occupational exposure, screening for Hepatitis C is the most appropriate next step in management.
Based on the information provided, the most appropriate next step in management would be to schedule a mammography to screen for breast cancer. The patient's age, family history, and recent mammography results all suggest that she may be at increased risk for breast cancer. In addition, it is recommended that women in their 60s undergo regular mammography screenings to detect breast cancer early.
57
A 60-year-old woman comes to the office as a new patient. She recently moved to the area. She feels well. She has
hypertension treated with amlodipine and asthma well controlled with inhaled fluticasone and inhaled albuterol as
needed. According to the medical records she brought with her, a mammography 6 months ago showed no
abnormalities, results of laboratory studies 1 year ago were within the reference ranges, and colonoscopy 5 years ago
showed no abnormalities. Her past three Pap smears have shown no abnormalities; the most recent Pap smear was
1 year ago. Her mother has type 2 diabetes mellitus, and her maternal aunt had breast cancer at the age of 55 years.
The patient does not smoke cigarettes. She drinks two glasses of wine weekly. She works as a nurse. She has been
monogamous with her husband for 30 years. Pulse is 76/min, respirations are 16/min, and blood pressure is
132/86 mm Hg. Examination shows no abnormalities. Screening for which of the following is the most appropriate
next step in management and why are the others not? Provide rationale for each choice below.


(A) Abdominal aortic aneurysm
(B) Breast cancer
(C) Chlamydia trachomatis infection
(D) Colon cancer
(E) Hepatitis C
(E) Hepatitis CCorrect
The most appropriate next step in management for this patient would be screening for Hepatitis C (Choice E).

Rationale:
- Abdominal aortic aneurysm (Choice A): This patient does not have any risk factors for abdominal aortic aneurysm, such as smoking or a family history of the condition. Additionally, there are no symptoms or findings on examination to suggest an abdominal aortic aneurysm. Therefore, screening for this condition is not indicated at this time.
- Breast cancer (Choice B): Although the patient's maternal aunt had breast cancer, the patient's recent mammography showed no abnormalities. Additionally, the patient does not have any personal risk factors for breast cancer, such as a previous abnormal mammogram or a family history of breast cancer at a young age. Therefore, routine screening for breast cancer is not necessary at this time.
- Chlamydia trachomatis infection (Choice C): This patient is a 60-year-old woman who has been monogamous with her husband for 30 years. Chlamydia trachomatis infection is primarily a sexually transmitted infection and is more common in younger individuals who are sexually active. Given the patient's age, monogamous relationship, and lack of symptoms, routine screening for chlamydia is not indicated.
- Colon cancer (Choice D): The patient had a colonoscopy 5 years ago that showed no abnormalities. The current guidelines recommend colon cancer screening every 10 years starting at age 50 for individuals at average risk. Since the patient's previous colonoscopy was within the recommended time frame and showed no abnormalities, routine screening for colon cancer is not necessary at this time.

Hepatitis C screening is the most appropriate next step in management for this patient because it is a bloodborne infection that can be asymptomatic for many years. The patient's history of working as a nurse puts her at risk for exposure to blood and body fluids, which increases her risk for Hepatitis C. Additionally, the patient's age group (born between 1945 and 1965) is known to have a higher prevalence of Hepatitis C. Therefore, screening for Hepatitis C is recommended to identify any potential infection and initiate appropriate management if necessary.
The most appropriate next step in management would be screening for colon cancer, as the patient's most recent colonoscopy was performed 5 years ago. While abdominal aortic aneurysm, breast cancer, and chlamydia trachomatis infection are also important to screen for, they are not the most pressing concern based on the information provided. The patient's most recent mammography showed no abnormalities, and she has no symptoms of a chlamydia infection. Additionally, the patient's risk for hepatitis C is likely low, as she does not have any known risk factors for the disease.
58
A 12-year-old girl is brought to the physician for a follow-up examination 8 days after starting a course of oral
penicillin for streptococcal pharyngitis. Temperature is 38.6°C (101.5°F). Examination shows an erythematous
morbilliform rash, urticaria, mild swelling of the hands and feet, arthralgias, and lymphadenopathy. Leukocyte count
is 13,000/mm³ (62% segmented neutrophils, 10% eosinophils, and 28% lymphocytes). Urinalysis shows 2+ protein.
Which of the following is most appropriate to prevent recurrence of these symptoms if the patient is diagnosed with
streptococcal pharyngitis again?


(A) Pretreatment with an antihistamine
(B) Pretreatment with a corticosteroid
(C) Skin testing with benzylpenicilloyl polylysine prior to treatment
(D) Use of a nonpenicillin antibiotic to treat the pharyngitis
(E) Use of a single injection of benzathine penicillin to treat the pharyngitis
(D) Use of a nonpenicillin antibiotic to treat the pharyngitis
Correct
The correct answer is (D) Use of a nonpenicillin antibiotic to treat the pharyngitis.

Explanation:
The patient in this scenario is experiencing symptoms of an allergic reaction to penicillin, known as a delayed hypersensitivity reaction. The symptoms include an erythematous morbilliform rash, urticaria, mild swelling of the hands and feet, arthralgias, and lymphadenopathy. This type of reaction is mediated by T cells and occurs several days after exposure to the allergen.

To prevent recurrence of these symptoms, it is important to avoid further exposure to penicillin. Therefore, the most appropriate option is to use a nonpenicillin antibiotic to treat the pharyngitis if the patient is diagnosed with streptococcal pharyngitis again. This will help prevent another allergic reaction and its associated symptoms.

Pretreatment with an antihistamine or a corticosteroid may help alleviate the symptoms of the allergic reaction but will not prevent a recurrence if the patient is exposed to penicillin again. Skin testing with benzylpenicilloyl polylysine prior to treatment is used to confirm a penicillin allergy but is not necessary in this case since the patient's symptoms are already indicative of an allergic reaction. The use of a single injection of benzathine penicillin to treat the pharyngitis would not be appropriate in this case as the patient has already demonstrated an allergic reaction to penicillin.
If the patient has been diagnosed with streptococcal pharyngitis and has experienced symptoms such as a rash, urticaria, and swelling of the hands and feet after taking penicillin, it is most appropriate to use a nonpenicillin antibiotic to treat the pharyngitis in order to prevent recurrence of these symptoms. Options (A) and (B) involve pretreatment with an antihistamine and a corticosteroid, respectively, which may help to reduce symptoms but would not prevent the recurrence of streptococcal pharyngitis. Skin testing with benzylpenicilloyl polylysine, as described in option (C), is typically used to determine if a patient is allergic to penicillin, but would not prevent the recurrence of symptoms in this case. Finally, option (E) involves using a single injection of benzathine penicillin to treat the pharyngitis, but given that the patient has already experienced symptoms after taking penicillin, using a nonpenicillin antibiotic would be a better option.
59
A 12-year-old girl is brought to the physician for a follow-up examination 8 days after starting a course of oral
penicillin for streptococcal pharyngitis. Temperature is 38.6°C (101.5°F). Examination shows an erythematous
morbilliform rash, urticaria, mild swelling of the hands and feet, arthralgias, and lymphadenopathy. Leukocyte count
is 13,000/mm³ (62% segmented neutrophils, 10% eosinophils, and 28% lymphocytes). Urinalysis shows 2+ protein.
Which of the following is most appropriate to prevent recurrence of these symptoms if the patient is diagnosed with
streptococcal pharyngitis again and why? Provide rationale for each choice on why it is appropriate or not appopriate as a course of action.


(A) Pretreatment with an antihistamine
(B) Pretreatment with a corticosteroid
(C) Skin testing with benzylpenicilloyl polylysine prior to treatment
(D) Use of a nonpenicillin antibiotic to treat the pharyngitis
(E) Use of a single injection of benzathine penicillin to treat the pharyngitis
(D) Use of a nonpenicillin antibiotic to treat the pharyngitis
Correct
The correct choice is (D) Use of a nonpenicillin antibiotic to treat the pharyngitis.

Explanation:
The patient in this scenario is experiencing symptoms of an allergic reaction to penicillin, which include an erythematous morbilliform rash, urticaria, mild swelling of the hands and feet, arthralgias, and lymphadenopathy. These symptoms are consistent with a type IV hypersensitivity reaction, which is mediated by T cells and can occur 7-10 days after exposure to the allergen.

To prevent recurrence of these symptoms if the patient is diagnosed with streptococcal pharyngitis again, it is most appropriate to use a nonpenicillin antibiotic to treat the infection. This is because the patient has developed an allergic reaction to penicillin, and using penicillin again would likely result in a similar allergic reaction. By using a nonpenicillin antibiotic, the risk of an allergic reaction is minimized.

Pretreatment with an antihistamine (choice A) or a corticosteroid (choice B) may help alleviate the symptoms of the allergic reaction, but they would not prevent a recurrence of the symptoms if the patient is exposed to penicillin again. These medications are more commonly used for immediate allergic reactions, such as anaphylaxis.

Skin testing with benzylpenicilloyl polylysine prior to treatment (choice C) is a diagnostic test used to determine if a patient is allergic to penicillin. However, in this scenario, the patient has already developed an allergic reaction to penicillin, so skin testing is not necessary.

Using a single injection of benzathine penicillin to treat the pharyngitis (choice E) would not be appropriate because the patient has already demonstrated an allergic reaction to penicillin. This would likely result in a recurrence of the allergic symptoms.

Therefore, the most appropriate course of action to prevent recurrence of these symptoms is to use a nonpenicillin antibiotic to treat the pharyngitis.
Based on the symptoms described in the question, it is likely that the 12-year-old girl is experiencing an allergic reaction to the penicillin. In order to prevent recurrence of these symptoms if the patient is diagnosed with streptococcal pharyngitis again, the most appropriate course of action would be to use a nonpenicillin antibiotic to treat the pharyngitis. This will avoid the allergic reaction that the patient experienced with the penicillin.

Pretreatment with an antihistamine or a corticosteroid could be helpful in reducing the severity of allergic symptoms, but they would not prevent the allergic reaction from occurring in the first place. Skin testing with benzylpenicilloyl polylysine prior to treatment may be useful in identifying patients who are allergic to penicillin, but it would not prevent a recurrence of allergic symptoms in this particular patient who has already had an allergic reaction to penicillin.

Finally, using a single injection of benzathine penicillin to treat the pharyngitis would not be appropriate in this case, because the patient has already had an allergic reaction to penicillin and is likely to experience the same reaction again if given another dose of the medication. It is important to avoid administering the same medication that caused the allergic reaction in order to prevent further harm to the patient.
60
A 45-year-old woman is hospitalized for management of Staphylococcus aureus endocarditis with persistent
bacteremia. The patient is discussed during interdisciplinary rounds, which includes physicians, nurses, pharmacists,
and social workers. During rounds, a pharmacy student notices that the patient missed two doses of her scheduled
antibiotic last week but is unsure why. The physician and nurse are unaware of these missed doses, and the student
does not mention her observation. Which of the following measures is most likely to improve communication within
this interdisciplinary health care team?


(A) Conduct interdisciplinary rounds in a quieter location
(B) Encourage questions from all team members
(C) Implement a checklist for standardizing patient rounds
(D) Use computers during rounds to review medications
(B) Encourage questions from all team members
Correct
The correct choice is (B) Encourage questions from all team members.

Explanation:
Encouraging questions from all team members is the most likely measure to improve communication within this interdisciplinary healthcare team. In this scenario, the pharmacy student noticed that the patient missed two doses of her scheduled antibiotic but did not mention it to the physician and nurse. This lack of communication can lead to potential harm to the patient if the missed doses are not addressed. By encouraging questions from all team members, it creates an environment where everyone feels comfortable speaking up and sharing important information or concerns. This can help prevent errors, improve patient safety, and enhance collaboration among the team members.
(B) Encourage questions from all team members is the most likely measure to improve communication within the interdisciplinary health care team. In this scenario, a pharmacy student noticed that the patient missed two doses of her scheduled antibiotic, but did not mention this observation during rounds. By encouraging questions from all team members, the pharmacy student would feel more comfortable speaking up and sharing her observation, which could improve the team's understanding of the patient's medication regimen and help ensure that she receives the proper treatment.
61
A 45-year-old woman is hospitalized for management of Staphylococcus aureus endocarditis with persistent
bacteremia. The patient is discussed during interdisciplinary rounds, which includes physicians, nurses, pharmacists,
and social workers. During rounds, a pharmacy student notices that the patient missed two doses of her scheduled
antibiotic last week but is unsure why. The physician and nurse are unaware of these missed doses, and the student
does not mention her observation. Which of the following measures is most likely to improve communication within
this interdisciplinary health care team? Provide rationale on why your answer might help the situation best compared to the other choices.


(A) Conduct interdisciplinary rounds in a quieter location
(B) Encourage questions from all team members
(C) Implement a checklist for standardizing patient rounds
(D) Use computers during rounds to review medications
(B) Encourage questions from all team members
Correct
The correct choice is (B) Encourage questions from all team members.

Explanation:
Encouraging questions from all team members is the most likely measure to improve communication within this interdisciplinary healthcare team. In this scenario, the pharmacy student noticed that the patient missed two doses of her scheduled antibiotic, but did not mention it to the physician and nurse. This lack of communication can lead to potential harm to the patient, as missed doses of antibiotics can impact the effectiveness of treatment for Staphylococcus aureus endocarditis.

By encouraging questions from all team members, it creates an environment where everyone feels comfortable speaking up and sharing important information. This can help prevent crucial details from being overlooked or missed, leading to better patient care and outcomes. In this case, if the pharmacy student had felt encouraged to ask questions or share their observation, the missed doses could have been addressed and appropriate actions taken to ensure the patient receives the necessary medication.

The other options may also have some benefits, but they are not as directly related to improving communication within the team in this specific scenario. Conducting interdisciplinary rounds in a quieter location (option A) may help reduce distractions, but it does not address the issue of communication and information sharing. Implementing a checklist for standardizing patient rounds (option C) can improve consistency and efficiency, but it does not specifically address the issue of communication or encourage questions. Using computers during rounds to review medications (option D) can be helpful for accessing and reviewing patient information, but it does not directly address the issue of communication or encourage team members to share important observations or ask questions.
(B) Encouraging questions from all team members is most likely to improve communication within this interdisciplinary health care team. This is because it encourages team members to speak up and share any observations or concerns they may have, which can help prevent misunderstandings and improve the overall care of the patient. For example, if the pharmacy student in the scenario had felt comfortable speaking up and mentioning the missed doses of medication, the physician and nurse would have been able to address the issue and prevent any negative effects on the patient's health. By contrast, the other options would not directly address the communication issue and may not have the same impact on improving communication within the team.
62
A 37-year-old man comes to the physician for a routine health maintenance examination. He has no history of
serious illness and takes no medications. He does not smoke cigarettes, and he exercises four times weekly. His
mother died of colon cancer at the age of 76 years. He is 183 cm (6 ft 0 in) tall and weighs 100 kg (220 lb); BMI is
30 kg/m². Pulse is 68/min and blood pressure is 138/74 mm Hg. The remainder of the examination shows no
abnormalities. Which of the following is the most appropriate next step?


(A) Test of the stool for occult blood
(B) Complete blood count
(C) Measurement of serum thyroid-stimulating hormone concentration
(D) Serum lipid studies
(E) No further evaluation is necessary
(D) Serum lipid studiesCorrect
The correct choice is (D) Serum lipid studies.

Explanation:
The patient in this scenario is a 37-year-old man who is overweight (BMI of 30 kg/m²) and has a family history of colon cancer. These factors put him at risk for cardiovascular disease. Therefore, the most appropriate next step would be to perform serum lipid studies to assess his lipid profile and determine his risk for cardiovascular disease.

Testing the stool for occult blood (choice A) is not indicated in this patient because he does not have any symptoms or risk factors for gastrointestinal bleeding.

A complete blood count (choice B) is not necessary in this patient because he does not have any symptoms or risk factors for hematologic disorders.

Measurement of serum thyroid-stimulating hormone concentration (choice C) is not indicated in this patient because he does not have any symptoms or risk factors for thyroid dysfunction.

No further evaluation is necessary (choice E) is not the most appropriate choice because the patient has risk factors for cardiovascular disease and should have his lipid profile assessed.
The most appropriate next step would be to test the stool for occult blood, as this can help detect the presence of colon cancer, which the patient's mother had. This is represented by option (A), which is to test the stool for occult blood. Other tests, such as a complete blood count (option B), measurement of serum thyroid-stimulating hormone concentration (option C), or serum lipid studies (option D), are not necessary at this time and may not be related to the patient's risk of colon cancer. Therefore, option (E) is not correct.
63
A 37-year-old man comes to the physician for a routine health maintenance examination. He has no history of
serious illness and takes no medications. He does not smoke cigarettes, and he exercises four times weekly. His
mother died of colon cancer at the age of 76 years. He is 183 cm (6 ft 0 in) tall and weighs 100 kg (220 lb); BMI is
30 kg/m². Pulse is 68/min and blood pressure is 138/74 mm Hg. The remainder of the examination shows no
abnormalities. Which of the following is the most appropriate next step and provide rationale for each of the other choices why you did not pick them.


(A) Test of the stool for occult blood
(B) Complete blood count
(C) Measurement of serum thyroid-stimulating hormone concentration
(D) Serum lipid studies
(E) No further evaluation is necessary
(D) Serum lipid studiesCorrect
The correct choice is (D) Serum lipid studies.

Explanation:
The most appropriate next step in this case is to perform serum lipid studies. This patient has multiple risk factors for cardiovascular disease, including obesity (BMI of 30 kg/m²), a family history of colon cancer, and a sedentary lifestyle. Serum lipid studies can help assess his cardiovascular risk by measuring his cholesterol levels, including total cholesterol, LDL cholesterol, HDL cholesterol, and triglycerides. Abnormal lipid levels can indicate an increased risk of developing cardiovascular disease and may warrant further intervention, such as lifestyle modifications or medication.

Rationale for each of the other choices:

(A) Test of the stool for occult blood: This option is not the most appropriate next step because the patient does not have any symptoms or risk factors for gastrointestinal bleeding. Routine screening for occult blood in the stool is typically recommended starting at age 50 for individuals at average risk for colorectal cancer.

(B) Complete blood count: This option is not the most appropriate next step because the patient does not have any symptoms or risk factors that would suggest the need for a complete blood count. A complete blood count is typically ordered to evaluate for anemia, infection, or other blood disorders, but there is no indication for it in this case.

(C) Measurement of serum thyroid-stimulating hormone concentration: This option is not the most appropriate next step because the patient does not have any symptoms or risk factors for thyroid dysfunction. Measurement of serum thyroid-stimulating hormone concentration is typically indicated in patients with symptoms or risk factors for thyroid disease, such as fatigue, weight changes, or a family history of thyroid disorders.

(E) No further evaluation is necessary: This option is not the most appropriate next step because the patient has multiple risk factors for cardiovascular disease, including obesity and a sedentary lifestyle. It is important to assess his cardiovascular risk through serum lipid studies and potentially intervene with lifestyle modifications or medication if indicated.
The most appropriate next step would be to test the stool for occult blood. This is because the patient's mother had a history of colon cancer, and testing for occult blood can help screen for colon cancer. A complete blood count, measurement of serum thyroid-stimulating hormone concentration, and serum lipid studies are not necessarily indicated in this situation and would not be the most appropriate next steps. It is not necessarily the case that no further evaluation is necessary, as screening for colon cancer is important in this situation given the patient's family history.
64
A 35-year-old woman is evaluated for a persistent fever 4 days after admission to the hospital for treatment of
pyelonephritis. She has type 2 diabetes mellitus poorly controlled with insulin. She has received ciprofloxacin since
admission. Results of initial blood cultures are pending. Temperature is 40.0°C (104.0°F), pulse is 110/min,
respirations are 20/min, and blood pressure is 100/60 mm Hg. Examination shows marked right costovertebral angle
tenderness. Which of the following is the most appropriate next step in management?


(A) Addition of clindamycin to the medication regimen
(B) Addition of vancomycin to the medication regimen
(C) CT scan of the abdomen
(D) Doppler ultrasonography of the lower extremities
(E) Repeat blood cultures and fungal cultures
(C) CT scan of the abdomenCorrect
The correct choice is (C) CT scan of the abdomen.

Explanation:
The patient's presentation with persistent fever, right costovertebral angle tenderness, and poorly controlled diabetes raises concern for a complicated urinary tract infection (UTI) or pyelonephritis. The addition of clindamycin or vancomycin to the medication regimen would be appropriate if there was concern for a resistant organism, such as methicillin-resistant Staphylococcus aureus (MRSA). However, the presence of marked right costovertebral angle tenderness suggests the possibility of an abscess or other complication, which would require further evaluation. A CT scan of the abdomen is the most appropriate next step in management to assess for the presence of an abscess or other complications. Doppler ultrasonography of the lower extremities would not be indicated in this case, as there are no findings to suggest a lower extremity thrombosis. Repeat blood cultures and fungal cultures may be considered if the initial cultures are negative or if there is concern for a fungal infection, but they would not provide immediate information about the source of the patient's symptoms.
The most appropriate next step in management would be to repeat blood cultures and fungal cultures to determine the cause of the persistent fever. If the initial results of the blood cultures are positive, the healthcare provider may need to adjust the patient's medication regimen, potentially by adding vancomycin or clindamycin, to address the infection. CT scan of the abdomen and Doppler ultrasonography of the lower extremities may be useful in certain circumstances, but they are not the most appropriate next step in this case.
65
A 35-year-old woman is evaluated for a persistent fever 4 days after admission to the hospital for treatment of
pyelonephritis. She has type 2 diabetes mellitus poorly controlled with insulin. She has received ciprofloxacin since
admission. Results of initial blood cultures are pending. Temperature is 40.0°C (104.0°F), pulse is 110/min,
respirations are 20/min, and blood pressure is 100/60 mm Hg. Examination shows marked right costovertebral angle
tenderness. Select an answer and why is it the most appropriate next step in management among the others?


(A) Addition of clindamycin to the medication regimen
(B) Addition of vancomycin to the medication regimen
(C) CT scan of the abdomen
(D) Doppler ultrasonography of the lower extremities
(E) Repeat blood cultures and fungal cultures
(C) CT scan of the abdomenCorrect
The most appropriate next step in management for this patient is (C) CT scan of the abdomen.

The patient's persistent fever, marked right costovertebral angle tenderness, and poorly controlled diabetes mellitus raise concern for a possible complication of pyelonephritis, such as an abscess or renal obstruction. A CT scan of the abdomen is the most appropriate next step to evaluate for these complications.

Option (A) Addition of clindamycin to the medication regimen is not the most appropriate next step because it does not address the need for further evaluation of possible complications.

Option (B) Addition of vancomycin to the medication regimen is not the most appropriate next step because it does not address the need for further evaluation of possible complications. Additionally, vancomycin is not typically used as first-line therapy for pyelonephritis.

Option (D) Doppler ultrasonography of the lower extremities is not the most appropriate next step because the patient's symptoms and examination findings are suggestive of a renal complication, not a lower extremity issue.

Option (E) Repeat blood cultures and fungal cultures may be considered, but they do not address the need for further evaluation of possible complications. A CT scan of the abdomen would provide more information about the source of the patient's persistent fever and tenderness.
The most appropriate next step in management would be to repeat blood cultures and fungal cultures. Since the initial blood cultures are pending and the patient has a persistent fever, it is important to identify the specific cause of the infection in order to provide appropriate treatment. In addition, given the patient's history of diabetes and the presence of costovertebral angle tenderness, it is possible that the infection has spread to the kidneys, making it important to rule out any potential complications. Other options, such as adding clindamycin or vancomycin to the medication regimen or performing imaging studies, may be necessary depending on the results of the repeat blood cultures and fungal cultures.
66
A 23-year-old man is being evaluated in the intensive care unit because of steadily decreasing blood pressure since
he was admitted 2 hours ago for treatment of septic shock caused by pneumonia. Medical history is remarkable for
cystic fibrosis and severe obstructive lung disease. In the emergency department, the patient was intubated, had three
peripheral intravenous catheters placed, and was treated empirically with broad-spectrum antibiotics,
bronchodilators, a continuous infusion of propofol and fentanyl, and 4 L of 0.9% saline. Immediately after
administration of fluids, the patient's pulse was 105/min and blood pressure was 100/50 mm Hg. The patient is 173
cm (5 ft 8 in) tall and weighs 59 kg (130 lb); BMI is 20 kg/m². Pulse is 135/min and blood pressure is 79/43 mm Hg.
Auscultation of the lungs discloses diffuse rhonchi and inspiratory crackles with low-pitched bilateral expiratory
wheezes. Cardiac examination discloses no murmurs, rubs, or gallops. Bedside ultrasonography of the abdomen
shows the inferior vena cava is 1.8 cm in diameter with minimum respirophasic collapsibility, and cardiac
ultrasonography shows that the left ventricular ejection fraction is approximately 0.65. Results of laboratory studies
obtained on admission and now are shown:

On Admission, the White Blood Cell count is 21,000/mm³ and Blood Lactic acid is 5.2 mg/dL. Now the patient's White Blood Cell count is 19,000/mm³ and Blood Lactic acid is 4.0 mg/dL

Which of the following is the most appropriate next step in management?

(A) Administer an additional 1-L bolus of 0.9% saline
(B) Administer intravenous hydrocortisone
(C) Administer intravenous voriconazole
(D) Begin a continuous infusion of norepinephrine
(E) Order CT scan of the head
(D) Begin a continuous infusion of norepinephrine
Correct
The correct answer is (D) Begin a continuous infusion of norepinephrine.

Explanation:
The patient in this scenario is presenting with septic shock, as evidenced by the steadily decreasing blood pressure, tachycardia, and elevated lactate levels. Septic shock is a life-threatening condition that requires immediate intervention to restore adequate tissue perfusion and prevent organ dysfunction.

The initial management of septic shock involves the administration of intravenous fluids to restore intravascular volume. In this case, the patient has already received 4 L of 0.9% saline, but his blood pressure and perfusion remain inadequate. Therefore, administering an additional 1-L bolus of saline (choice A) is unlikely to be effective and may even worsen the patient's condition by causing fluid overload.

Intravenous hydrocortisone (choice B) is indicated in patients with septic shock who do not respond to fluid resuscitation and vasopressor therapy. However, it is not the most appropriate next step in this case, as the patient has not yet received vasopressor therapy.

Intravenous voriconazole (choice C) is an antifungal medication and is not indicated in the management of septic shock.

The most appropriate next step in management for this patient is to begin a continuous infusion of norepinephrine (choice D). Norepinephrine is a vasopressor that increases systemic vascular resistance and improves blood pressure and tissue perfusion. It is the first-line vasopressor in the management of septic shock and should be initiated when fluid resuscitation alone is insufficient to restore blood pressure.

Ordering a CT scan of the head (choice E) is not indicated in the management of septic shock. The patient's symptoms and clinical presentation are consistent with septic shock, and immediate intervention to restore tissue perfusion is necessary. A CT scan of the head would not provide any relevant information or impact the management of the patient's septic shock.
The most appropriate next step in management would be to administer a continuous infusion of norepinephrine. This is because the patient's blood pressure is decreasing and norepinephrine can help to increase blood pressure. Additionally, the patient's pulse is elevated and norepinephrine can help to reduce the heart rate. Administering an additional bolus of 0.9% saline may not be necessary, as the patient has already received 4 L of fluid. Voriconazole is not typically used to treat septic shock, and ordering a CT scan of the head would not be immediately beneficial for the management of the patient's current condition.
67
A 23-year-old man is being evaluated in the intensive care unit because of steadily decreasing blood pressure since
he was admitted 2 hours ago for treatment of septic shock caused by pneumonia. Medical history is remarkable for
cystic fibrosis and severe obstructive lung disease. In the emergency department, the patient was intubated, had three
peripheral intravenous catheters placed, and was treated empirically with broad-spectrum antibiotics,
bronchodilators, a continuous infusion of propofol and fentanyl, and 4 L of 0.9% saline. Immediately after
administration of fluids, the patient's pulse was 105/min and blood pressure was 100/50 mm Hg. The patient is 173
cm (5 ft 8 in) tall and weighs 59 kg (130 lb); BMI is 20 kg/m². Pulse is 135/min and blood pressure is 79/43 mm Hg.
Auscultation of the lungs discloses diffuse rhonchi and inspiratory crackles with low-pitched bilateral expiratory
wheezes. Cardiac examination discloses no murmurs, rubs, or gallops. Bedside ultrasonography of the abdomen
shows the inferior vena cava is 1.8 cm in diameter with minimum respirophasic collapsibility, and cardiac
ultrasonography shows that the left ventricular ejection fraction is approximately 0.65. Results of laboratory studies
obtained on admission and now are shown:

On Admission, the White Blood Cell count is 21,000/mm³ and Blood Lactic acid is 5.2 mg/dL. Now the patient's White Blood Cell count is 19,000/mm³ and Blood Lactic acid is 4.0 mg/dL

Which of the following would be the most appropriate next step in management compared to others? Provide rationale for each.

(A) Administer an additional 1-L bolus of 0.9% saline
(B) Administer intravenous hydrocortisone
(C) Administer intravenous voriconazole
(D) Begin a continuous infusion of norepinephrine
(E) Order CT scan of the head
(D) Begin a continuous infusion of norepinephrine
Correct
The correct answer is (D) Begin a continuous infusion of norepinephrine.

Rationale:
The patient in this scenario is presenting with septic shock, as evidenced by the steadily decreasing blood pressure, tachycardia, and elevated lactate levels. Septic shock is a life-threatening condition that requires immediate intervention to restore adequate tissue perfusion and oxygenation.

The initial management of septic shock involves fluid resuscitation with intravenous fluids, which has already been done in this case with the administration of 4 L of 0.9% saline. However, despite the fluid administration, the patient's blood pressure remains low, indicating inadequate response to fluid therapy.

In this situation, the next step in management would be to initiate vasopressor therapy, specifically a continuous infusion of norepinephrine. Norepinephrine is a potent vasoconstrictor that increases systemic vascular resistance and improves blood pressure. It is the first-line vasopressor recommended in the management of septic shock.

Administering an additional 1-L bolus of 0.9% saline (choice A) may further increase fluid overload and worsen the patient's condition. Intravenous hydrocortisone (choice B) may be considered in patients with refractory shock, but it is not the initial treatment of choice. Intravenous voriconazole (choice C) is an antifungal medication and is not indicated in the management of septic shock. Ordering a CT scan of the head (choice E) is not necessary in this case as the patient's symptoms and clinical presentation are consistent with septic shock, and the focus should be on immediate resuscitation and stabilization.
Based on the information provided, it appears that the patient is experiencing septic shock, which is characterized by low blood pressure and a rapid heart rate. The most appropriate next step in management would be to administer intravenous norepinephrine (option D). Norepinephrine is a vasopressor that can help to increase blood pressure and improve perfusion to vital organs. Administering additional saline (option A) may not be effective in increasing blood pressure in this situation. Administering hydrocortisone (option B) and voriconazole (option C) would not be appropriate given the patient's current condition and may not address the underlying cause of the patient's symptoms. A CT scan of the head (option E) is not indicated based on the information provided and would not address the patient's current symptoms.
68
A 32-year-old nulligravid woman comes to the office for counseling prior to conception. Menses occur at irregular
40- and 60-day intervals. She has impaired glucose tolerance controlled by diet. She takes no medications. She is
165 cm (5 ft 5 cm) tall and weighs 84 kg (185 lb); BMI is 31 kg/m². Vital signs are within normal limits.
Examination shows no abnormalities. Her fasting serum glucose concentration is 120 mg/dL. This patient is at
greatest risk for developing which of the following?


(A) Gestational diabetes
(B) Hypothyroidism
(C) Intrauterine growth restriction
(D) Placenta previa
(E) Preterm labor
(A) Gestational diabetesCorrect
The correct answer is (A) Gestational diabetes.

Explanation:
This patient is at greatest risk for developing gestational diabetes. Gestational diabetes is a condition characterized by high blood sugar levels that develop during pregnancy. Risk factors for gestational diabetes include impaired glucose tolerance, obesity (BMI > 30 kg/m²), and a history of polycystic ovary syndrome with insulin resistance. In this case, the patient has impaired glucose tolerance controlled by diet and a BMI of 31 kg/m², which puts her at increased risk for developing gestational diabetes. It is important for her to be screened for gestational diabetes during pregnancy to ensure proper management and prevent complications for both the mother and the baby.
Based on the information provided, this patient is at greatest risk for developing gestational diabetes. This is because she has impaired glucose tolerance and is overweight, as indicated by her BMI of 31 kg/m². Gestational diabetes is a condition that occurs during pregnancy and is characterized by high blood sugar levels. It can increase the risk of complications for the mother and baby, and is typically managed through diet and exercise.
69
A 32-year-old nulligravid woman comes to the office for counseling prior to conception. Menses occur at irregular
40- and 60-day intervals. She has impaired glucose tolerance controlled by diet. She takes no medications. She is
165 cm (5 ft 5 cm) tall and weighs 84 kg (185 lb); BMI is 31 kg/m². Vital signs are within normal limits.
Examination shows no abnormalities. Her fasting serum glucose concentration is 120 mg/dL. This patient is at
greatest risk for developing which of the following? What is the rationale for the answer picked?


(A) Gestational diabetes
(B) Hypothyroidism
(C) Intrauterine growth restriction
(D) Placenta previa
(E) Preterm labor
(A) Gestational diabetesCorrect
The correct answer is (A) Gestational diabetes.

The patient in the scenario has impaired glucose tolerance controlled by diet, which indicates that she already has some degree of glucose metabolism dysfunction. Additionally, her fasting serum glucose concentration is elevated at 120 mg/dL, which further suggests impaired glucose control.

These factors, along with her BMI of 31 kg/m², put her at a higher risk for developing gestational diabetes. Gestational diabetes is a condition characterized by high blood sugar levels that develop during pregnancy. Women who are overweight or obese, have a family history of diabetes, or have a history of impaired glucose tolerance are at an increased risk for developing gestational diabetes.

Therefore, given the patient's impaired glucose tolerance, elevated fasting serum glucose concentration, and BMI, she is at the greatest risk for developing gestational diabetes during pregnancy.
Based on the information provided, this patient is most at risk for developing gestational diabetes. This is because the patient has impaired glucose tolerance and is overweight, with a body mass index (BMI) of 31 kg/m². Gestational diabetes is a type of diabetes that occurs during pregnancy and is characterized by high blood sugar levels. It is more likely to develop in women who are overweight or obese, and who have a history of impaired glucose tolerance.
70
A 17-year-old girl is brought to the physician by her parents because they are concerned she is using illicit drugs.
They report that, “she just isn’t herself anymore.” She has no history of serious illness and takes no medications. She
is upset about being “tricked” to come to the doctor. Physical examination shows no abnormalities. On mental status
examination, she has an angry mood and anxious affect. The patient’s parents privately request a toxicology
screening for their daughter but ask that she be told that her blood will be taken to “check her thyroid gland.” After
discussing the parents’ wish for secrecy, which of the following is the most appropriate next step in management?


(A) Comply with the parents’ wishes
(B) Inform the parents that everything the patient tells the physician is confidential
(C) Inform the parents that if the patient’s toxicology screening is positive, it must be reported to the police
(D) Inform the parents that the physician cannot deceive the patient
(D) Inform the parents that the physician cannot deceive the patient
Correct
The correct choice is (D) Inform the parents that the physician cannot deceive the patient.

Explanation:
In this scenario, the parents are requesting the physician to deceive their daughter by telling her that her blood will be taken to check her thyroid gland when in reality it will be used for a toxicology screening. Deceiving the patient in this manner goes against the principles of honesty and trust in the physician-patient relationship. It is important to maintain open and honest communication with the patient, especially when it comes to sensitive issues such as substance abuse.

By informing the parents that the physician cannot deceive the patient, the physician upholds the ethical principle of honesty and ensures that the patient's trust is not compromised. The physician can explain to the parents the importance of open communication with the patient and the potential negative consequences of deceiving her. It is also important to discuss the benefits of obtaining the patient's consent for the toxicology screening and involving her in the decision-making process.
The most appropriate next step in management would be to inform the parents that the physician cannot deceive the patient. It is important to maintain trust and honesty in the physician-patient relationship, and deceiving the patient in this way would violate that trust. Additionally, it is the patient's right to know what is being done to her body and to provide informed consent for any medical procedures. As a result, option (D) is the correct answer.
71
A 17-year-old girl is brought to the physician by her parents because they are concerned she is using illicit drugs.
They report that, “she just isn’t herself anymore.” She has no history of serious illness and takes no medications. She
is upset about being “tricked” to come to the doctor. Physical examination shows no abnormalities. On mental status
examination, she has an angry mood and anxious affect. The patient’s parents privately request a toxicology
screening for their daughter but ask that she be told that her blood will be taken to “check her thyroid gland.” After
discussing the parents’ wish for secrecy, which of the following is the most appropriate next step in management? Provide rationale for each choice provided and why you picked the answer.


(A) Comply with the parents’ wishes
(B) Inform the parents that everything the patient tells the physician is confidential
(C) Inform the parents that if the patient’s toxicology screening is positive, it must be reported to the police
(D) Inform the parents that the physician cannot deceive the patient
(D) Inform the parents that the physician cannot deceive the patient
Correct
The correct answer is (D) Inform the parents that the physician cannot deceive the patient.

Explanation:
In this scenario, the parents are requesting the physician to deceive their daughter by telling her that her blood will be taken to check her thyroid gland, when in reality it will be used for a toxicology screening. Deceiving the patient in this manner goes against the principles of honesty and trust in the physician-patient relationship.

It is important for the physician to prioritize the well-being and trust of the patient. By informing the parents that the physician cannot deceive the patient, the physician upholds the ethical principles of honesty and transparency. This allows for open communication and trust between the physician and the patient, which is crucial for effective medical care.

Additionally, it is important to note that the decision to perform a toxicology screening should be made in collaboration with the patient. The patient's autonomy and right to make decisions about their own healthcare should be respected. If there are concerns about substance abuse, the physician should have a private conversation with the patient to discuss these concerns and explore appropriate interventions or referrals.
The most appropriate next step in management would be to inform the parents that everything the patient tells the physician is confidential. This is because the physician has a duty to maintain confidentiality with their patient, and cannot disclose any information without the patient's consent, unless there is a compelling reason to do so (such as if the patient is a threat to themselves or others). By informing the parents of this, the physician can help them understand the importance of maintaining confidentiality and the need for the patient to give their consent before any testing or disclosure of information can occur. Additionally, the physician can reassure the parents that they will do everything they can to help the patient, while respecting their rights and confidentiality.
72
A 52-year-old woman with type 2 diabetes mellitus comes to the office for a follow-up examination. Two weeks
ago, she was treated with amoxicillin for a urinary tract infection. At that time, her blood pressure was 165/98 mm
Hg, and serum creatinine concentration was 1.2 mg/dL; lisinopril therapy was also begun. During the past week, she
has had increased swelling of the lower extremities and shortness of breath. Her dysuria has resolved. She appears
anxious. Temperature is 36.8°C (98.2°F), pulse is 110/min, respirations are 20/min, and blood pressure is 185/105
mm Hg. Diffuse bilateral crackles are heard on pulmonary examination. An S3 gallop is heard. There is 3+ pitting
bilateral pretibial edema.

The laboratory studies showed that the levels for Serum Urea nitrogen is 45 mg/dL and Serum Creatinine is 4 mg/dL. As for the urine, its specific gravity is 1.010, no blood, and has moderate numbers of Hyaline casts.


Which of the following is the most likely diagnosis?

(A) Acute glomerulonephritis
(B) Acute interstitial nephritis
(C) Acute kidney injury
(D) Acute obstructive uropathy
(E) Intravascular volume depletion
(C) Acute kidney injuryCorrect
The correct answer is (C) Acute kidney injury.

Explanation:
The patient's presentation is consistent with acute kidney injury (AKI). AKI is characterized by a sudden decrease in kidney function, resulting in the accumulation of waste products and fluid imbalances.

In this case, the patient has a history of type 2 diabetes mellitus, hypertension, and recent treatment with amoxicillin for a urinary tract infection. These factors, along with the elevated blood pressure and presence of bilateral pretibial edema, suggest that the patient may have developed diabetic nephropathy, which is a common cause of AKI in patients with diabetes.

The laboratory findings of elevated serum urea nitrogen (BUN) and serum creatinine levels further support the diagnosis of AKI. The BUN and creatinine levels are markers of kidney function, and an increase in these levels indicates impaired kidney function.

The urine findings of a specific gravity of 1.010, no blood, and moderate numbers of hyaline casts are also consistent with AKI. A low specific gravity suggests the inability of the kidneys to concentrate urine, which is a characteristic feature of AKI. The absence of blood in the urine rules out acute glomerulonephritis as the cause of the patient's symptoms. The presence of hyaline casts suggests tubular injury, which can occur in AKI.

Therefore, based on the patient's clinical presentation and laboratory findings, the most likely diagnosis is acute kidney injury.
Based on the information provided, the most likely diagnosis is acute kidney injury. The patient's serum creatinine concentration has increased from 1.2 mg/dL to 4 mg/dL within a two-week period, which suggests that the kidneys are not functioning properly. The presence of 3+ pitting bilateral pretibial edema and diffuse bilateral crackles on pulmonary examination also support a diagnosis of acute kidney injury. This condition can be caused by a variety of factors, including the use of certain medications, such as amoxicillin, which the patient was recently treated with.
73
A 52-year-old woman with type 2 diabetes mellitus comes to the office for a follow-up examination. Two weeks
ago, she was treated with amoxicillin for a urinary tract infection. At that time, her blood pressure was 165/98 mm
Hg, and serum creatinine concentration was 1.2 mg/dL; lisinopril therapy was also begun. During the past week, she
has had increased swelling of the lower extremities and shortness of breath. Her dysuria has resolved. She appears
anxious. Temperature is 36.8°C (98.2°F), pulse is 110/min, respirations are 20/min, and blood pressure is 185/105
mm Hg. Diffuse bilateral crackles are heard on pulmonary examination. An S3 gallop is heard. There is 3+ pitting
bilateral pretibial edema.

The laboratory studies showed that the levels for Serum Urea nitrogen is 45 mg/dL and Serum Creatinine is 4 mg/dL. As for the urine, its specific gravity is 1.010, no blood, and has moderate numbers of Hyaline casts.


Which of the following is the most likely diagnosis? Can you provide your rationale on each choice provided?

(A) Acute glomerulonephritis
(B) Acute interstitial nephritis
(C) Acute kidney injury
(D) Acute obstructive uropathy
(E) Intravascular volume depletion
(C) Acute kidney injuryCorrect
The most likely diagnosis in this case is (C) Acute kidney injury.

Rationale for each choice:

(A) Acute glomerulonephritis: Acute glomerulonephritis is characterized by inflammation of the glomeruli in the kidneys. It typically presents with hematuria, proteinuria, and hypertension. In this case, the patient does not have hematuria or significant proteinuria. Additionally, the presence of bilateral pretibial edema and crackles on pulmonary examination suggests fluid overload, which is not typically seen in acute glomerulonephritis.

(B) Acute interstitial nephritis: Acute interstitial nephritis is characterized by inflammation of the interstitial tissue in the kidneys. It is often caused by medications, such as antibiotics. While the patient was recently treated with amoxicillin, the presentation of increased swelling of the lower extremities, shortness of breath, and crackles on pulmonary examination suggests fluid overload, which is not typically seen in acute interstitial nephritis.

(D) Acute obstructive uropathy: Acute obstructive uropathy occurs when there is a blockage in the urinary tract, preventing the normal flow of urine. This can lead to increased pressure in the kidneys and subsequent kidney injury. However, in this case, there is no evidence of obstruction on the urine dipstick test, and the presence of bilateral pretibial edema and crackles on pulmonary examination suggests fluid overload, which is not typically seen in acute obstructive uropathy.

(E) Intravascular volume depletion: Intravascular volume depletion refers to a decrease in the volume of blood circulating within the blood vessels. This can lead to decreased blood pressure and decreased perfusion to the kidneys, resulting in acute kidney injury. However, in this case, the patient's blood pressure is elevated, and there is evidence of fluid overload with bilateral pretibial edema and crackles on pulmonary examination. These findings are not consistent with intravascular volume depletion.

In summary, the most likely diagnosis in this case is acute kidney injury, as evidenced by the elevated serum creatinine and blood urea nitrogen levels, along with the presence of fluid overload and signs of cardiac dysfunction. The patient's history of diabetes and recent initiation of lisinopril therapy also increase the likelihood of acute kidney injury.
Based on the given information, the most likely diagnosis is acute kidney injury. This is because the patient's serum creatinine levels have increased significantly from 1.2 mg/dL to 4 mg/dL in a short period of time, indicating impaired kidney function. Additionally, the presence of bilateral crackles on pulmonary examination, an S3 gallop, and pretibial edema suggest fluid overload, which is a common complication of acute kidney injury.

Other possible diagnoses, such as acute glomerulonephritis and acute interstitial nephritis, could also cause acute kidney injury, but they would typically be accompanied by other specific symptoms that are not mentioned in the scenario. Acute obstructive uropathy and intravascular volume depletion are less likely given the patient's symptoms and laboratory results.
74
A 15-year-old girl is brought to the emergency department by her father because of a 2-hour history of right leg pain
that began after she fell from her bicycle. Medical history is remarkable for bilateral hearing loss, a fracture of the
humerus at age 8 years, and multiple rib fractures. She takes no medications. Family history is remarkable for
kyphoscoliosis, hearing loss, and early onset osteoporosis in her mother; the mother died from complications related
to a hip fracture at age 44 years. The patient is a refugee who has recently immigrated. The patient appears to be in
distress. She is below the 5th percentile for height, at the 70th percentile for weight, and above the 95th percentile
for BMI. Her temperature is 37.0°C (98.6°F), pulse is 118/min, respirations are 12/min, and blood pressure is
130/68 mm Hg. Examination shows kyphosis and hypermobile joints of the hands, wrists, and feet. Examination of
the right lower extremity shows superficial abrasions and ecchymoses; there is tenderness to palpation and
movement of the extremity. Which of the following is the most likely underlying diagnosis?


(A) Idiopathic juvenile osteoporosis
(B) Juvenile Paget disease
(C) Osteogenesis imperfecta
(D) Osteomalacia
(E) Rickets
(C) Osteogenesis imperfectaCorrect
The correct answer is (C) Osteogenesis imperfecta.

Osteogenesis imperfecta (OI) is a genetic disorder characterized by brittle bones and increased susceptibility to fractures. It is caused by mutations in the genes that produce type I collagen, the main protein in bone. Patients with OI often have a history of multiple fractures, as seen in this patient who has a history of humerus fracture and multiple rib fractures. The patient's family history of early onset osteoporosis in the mother is also suggestive of OI.

In addition to fractures, patients with OI may also have other skeletal abnormalities such as kyphoscoliosis, which is seen in this patient. Hypermobile joints of the hands, wrists, and feet are also common in OI.

The patient's presentation of right leg pain after a fall from a bicycle, along with tenderness to palpation and movement of the extremity, suggests a fracture in the right lower extremity. This is consistent with the increased susceptibility to fractures seen in OI.

The other answer choices are less likely based on the patient's presentation and history. Idiopathic juvenile osteoporosis typically presents with fractures in adolescence, but it is not associated with other skeletal abnormalities or a family history of osteoporosis. Juvenile Paget disease is a rare genetic disorder characterized by increased bone turnover, but it typically presents with bone pain and deformities, not fractures. Osteomalacia is a condition characterized by softening of the bones due to vitamin D deficiency, but it typically presents with diffuse bone pain and muscle weakness. Rickets is a condition characterized by impaired mineralization of the growth plate, leading to skeletal deformities, but it typically presents in childhood and is associated with vitamin D deficiency.
Based on the information provided, the most likely underlying diagnosis is (C) osteogenesis imperfecta. This condition is characterized by brittle bones, hypermobile joints, and a tendency to develop fractures easily. It is also associated with hearing loss, which the patient has a history of. The patient's mother had a history of early-onset osteoporosis, which is also a common complication of osteogenesis imperfecta. The patient's high BMI may be a result of the kyphosis (abnormal curvature of the spine) associated with this condition.
75
A 15-year-old girl is brought to the emergency department by her father because of a 2-hour history of right leg pain
that began after she fell from her bicycle. Medical history is remarkable for bilateral hearing loss, a fracture of the
humerus at age 8 years, and multiple rib fractures. She takes no medications. Family history is remarkable for
kyphoscoliosis, hearing loss, and early onset osteoporosis in her mother; the mother died from complications related
to a hip fracture at age 44 years. The patient is a refugee who has recently immigrated. The patient appears to be in
distress. She is below the 5th percentile for height, at the 70th percentile for weight, and above the 95th percentile
for BMI. Her temperature is 37.0°C (98.6°F), pulse is 118/min, respirations are 12/min, and blood pressure is
130/68 mm Hg. Examination shows kyphosis and hypermobile joints of the hands, wrists, and feet. Examination of
the right lower extremity shows superficial abrasions and ecchymoses; there is tenderness to palpation and
movement of the extremity. Which of the following is the most likely underlying diagnosis and how did you arrive at your answer? Why are the other choices not likely the answer?


(A) Idiopathic juvenile osteoporosis
(B) Juvenile Paget disease
(C) Osteogenesis imperfecta
(D) Osteomalacia
(E) Rickets
(C) Osteogenesis imperfectaCorrect
The most likely underlying diagnosis in this case is (C) Osteogenesis imperfecta.

Osteogenesis imperfecta (OI) is a genetic disorder characterized by brittle bones and increased susceptibility to fractures. The patient in this case has a history of multiple fractures, including a fracture of the humerus at age 8 years and multiple rib fractures. This history, along with the current presentation of right leg pain after a fall from a bicycle, is consistent with the diagnosis of OI.

Additionally, the patient's family history is remarkable for early onset osteoporosis in her mother, who died from complications related to a hip fracture at age 44 years. This family history further supports the diagnosis of OI, as it is a genetic disorder that can be inherited.

The patient's physical examination findings are also consistent with OI. Examination shows kyphosis, which is a forward curvature of the spine, and hypermobile joints of the hands, wrists, and feet. These findings are commonly seen in patients with OI.

The other answer choices are less likely to be the underlying diagnosis based on the given information:

(A) Idiopathic juvenile osteoporosis: This condition typically presents with fractures in adolescence, but it does not typically have the associated physical examination findings of kyphosis and hypermobile joints seen in this patient.

(B) Juvenile Paget disease: This is a rare genetic disorder characterized by increased bone turnover and abnormal bone remodeling. It typically presents with bone pain, deformities, and fractures. However, the patient's history and physical examination findings are more consistent with OI.

(D) Osteomalacia: This is a condition characterized by softening of the bones due to vitamin D deficiency or impaired vitamin D metabolism. It typically presents with bone pain, muscle weakness, and fractures. The patient's history and physical examination findings are not consistent with osteomalacia.

(E) Rickets: This is a condition characterized by impaired mineralization of the growing bones in children, usually due to vitamin D deficiency or calcium or phosphate abnormalities. It typically presents with bone pain, deformities, and growth retardation. The patient's history and physical examination findings are not consistent with rickets.
Based on the patient's history of hearing loss, kyphoscoliosis, hypermobile joints, and family history of osteoporosis and early onset death from complications related to a hip fracture, the most likely underlying diagnosis is osteogenesis imperfecta, or OI. OI is a genetic disorder characterized by brittle bones and other connective tissue abnormalities. The other choices are not as likely because idiopathic juvenile osteoporosis typically does not present with hearing loss, kyphoscoliosis, and hypermobile joints, and juvenile Paget disease, osteomalacia, and rickets typically present with different symptoms and signs.
76
A 22-year-old primigravid woman at 10 weeks’ gestation comes to the emergency department (ED) because of a 3-
day history of persistent vomiting. She is treated with intravenous hydration, pyridoxine, and doxylamine. As the
resident physician prepares the patient for discharge, he asks who will be picking her up from the ED. She says her
husband will be arriving soon and asks if the hospital staff can refer to her symptoms as a “stomach flu.” She does
not want anyone to mention to her husband that she is pregnant because the pregnancy resulted from an oral
contraceptive failure and “he is not expecting this.” Which of the following is the most appropriate next question to
ask this patient?


(A) “Are finances tight at home?”
(B) “How do you think he will react?”
(C) “Is there somebody else who can pick you up?”
(D) “Was there a problem with your birth control?”
(E) “Will he try to force you to do something you aren’t comfortable with?”
(B) “How do you think he will react?”Correct
The correct choice is (B) "How do you think he will react?"

Explanation:
This question is the most appropriate next question to ask the patient because it allows the healthcare provider to assess the patient's concerns and potential risks in disclosing her pregnancy to her husband. By asking how the patient thinks her husband will react, the healthcare provider can gain insight into the patient's relationship dynamics and potential risks of harm or coercion. This information is crucial in ensuring the patient's safety and well-being. It also allows the healthcare provider to provide appropriate support and resources if needed. The other options, such as asking about finances, someone else picking her up, or problems with birth control, do not directly address the patient's concerns about disclosing her pregnancy and potential risks in her relationship. The question about whether her husband will try to force her to do something she isn't comfortable with is important, but it is more appropriate to ask this question after assessing her initial concerns and potential risks.
The most appropriate next question to ask this patient is: "Is there somebody else who can pick you up?" (Option C). The resident physician should ensure that the patient has a safe way to get home after being discharged from the ED. This is particularly important in this case because the patient may not want her husband to know about her pregnancy. Asking about the patient's finances or her husband's potential reaction to the news of her pregnancy is not directly relevant to her care at this time. Additionally, it is not appropriate for the physician to ask about any potential problems with the patient's birth control or whether her husband may try to force her to do something she is not comfortable with. These questions could be intrusive and could make the patient uncomfortable.
77
A 22-year-old primigravid woman at 10 weeks’ gestation comes to the emergency department (ED) because of a 3-
day history of persistent vomiting. She is treated with intravenous hydration, pyridoxine, and doxylamine. As the
resident physician prepares the patient for discharge, he asks who will be picking her up from the ED. She says her
husband will be arriving soon and asks if the hospital staff can refer to her symptoms as a “stomach flu.” She does
not want anyone to mention to her husband that she is pregnant because the pregnancy resulted from an oral
contraceptive failure and “he is not expecting this.” Which of the following is the most appropriate next question to
ask this patient? What did you consider to pick your answer and not the other choices?


(A) “Are finances tight at home?”
(B) “How do you think he will react?”
(C) “Is there somebody else who can pick you up?”
(D) “Was there a problem with your birth control?”
(E) “Will he try to force you to do something you aren’t comfortable with?”
(B) “How do you think he will react?”Correct
The correct choice is (B) "How do you think he will react?"

Explanation:
This question is the most appropriate next question to ask the patient because it allows the healthcare provider to assess the patient's concerns and potential risks in disclosing her pregnancy to her husband. By asking how she thinks he will react, the healthcare provider can gain insight into the patient's relationship dynamics and potential risks of harm or coercion. This information is crucial in ensuring the patient's safety and well-being.

Option (A) "Are finances tight at home?" is not the most appropriate next question because it does not directly address the patient's concerns about disclosing her pregnancy to her husband. While financial concerns may be important, they are not the primary focus in this situation.

Option (C) "Is there somebody else who can pick you up?" is not the most appropriate next question because it does not address the patient's concerns about disclosing her pregnancy to her husband. While it is important to ensure the patient has a safe and reliable means of transportation, this question does not address the underlying issue.

Option (D) "Was there a problem with your birth control?" is not the most appropriate next question because it does not address the patient's concerns about disclosing her pregnancy to her husband. While it may be important to discuss the failure of her birth control method, it is not the immediate concern in this situation.

Option (E) "Will he try to force you to do something you aren't comfortable with?" is not the most appropriate next question because it assumes a potential risk of harm or coercion without first assessing the patient's concerns and perspective. It is important to gather more information before making assumptions about the patient's situation.
The most appropriate next question to ask this patient is "Is there somebody else who can pick you up?" (C). This question addresses the immediate concern of ensuring that the patient has a safe way to leave the hospital and can provide the patient with an alternative option if her husband is not able to pick her up. It also allows the physician to assess whether the patient has a supportive network of friends or family members who can provide support and assistance.

The other options are not appropriate in this situation because they are not relevant to the patient's immediate needs and could potentially cause the patient further distress. Asking about finances (A) or the patient's birth control (D) is not necessary for the physician to provide appropriate care and could be invasive. Asking about the patient's husband's reaction (B) or whether he will try to force the patient to do something she is uncomfortable with (E) could be distressing for the patient and could escalate the situation if the patient's concerns are valid.
78
Five days after admission to the hospital for management of an ST-elevation myocardial infarction, a 59-year-old
man has the sudden onset of severe abdominal and bilateral lower extremity pain, cyanosis of the lower body, and
paresthesias. On hospital days 1 and 2, he received vasopressor and inotropic therapy. On hospital day 2,
echocardiography showed extensive anterolateral akinesis of the left ventricle. He has no history of serious illness.
His medications are metoprolol, lisinopril, simvastatin, and dalteparin. He appears to be in acute distress. Pulse is
105/min and regular, respirations are 24/min, and blood pressure is 80/60 mm Hg. Examination shows lower body
cyanosis to the level of the costal margins. Crackles are heard over the lower lung fields bilaterally. Distant heart
sounds but no murmurs are heard. The abdomen is nondistended and nontender. Pulses are 1+ in the upper
extremities and absent in the lower extremities. Muscle strength is 3/5 in the lower extremities. Sensation to touch
and temperature is decreased over the lower extremities. ECG shows sinus tachycardia. Which of the following is
the most likely explanation for this patient’s clinical findings?

(A) Aortic dissection
(B) Aortic embolism
(C) Pericardial tamponade
(D) Right ventricular failure
(E) Ventricular rupture
(B) Aortic embolismCorrect
The correct choice is (B) Aortic embolism.

Explanation:
The patient's clinical findings are consistent with an aortic embolism. Aortic embolism occurs when a blood clot or other material (such as cholesterol plaque) becomes lodged in the aorta, blocking blood flow to the lower extremities. This can result in severe abdominal and bilateral lower extremity pain, cyanosis of the lower body, and paresthesias. The absence of pulses in the lower extremities and decreased sensation to touch and temperature further support the diagnosis of aortic embolism. The crackles heard over the lower lung fields bilaterally may indicate pulmonary edema, which can occur as a result of decreased cardiac output and increased pulmonary venous pressure. The patient's history of ST-elevation myocardial infarction and the presence of extensive anterolateral akinesis of the left ventricle on echocardiography suggest underlying cardiovascular disease, which increases the risk for embolic events. The use of vasopressor and inotropic therapy may have further contributed to the development of aortic embolism. Aortic dissection typically presents with sudden-onset severe tearing or ripping chest pain that radiates to the back. Pericardial tamponade is characterized by muffled heart sounds, distended neck veins, and hypotension. Right ventricular failure would not explain the absence of pulses in the lower extremities. Ventricular rupture would typically present with acute hemodynamic collapse and signs of cardiac tamponade.
Based on the given information, the most likely explanation for the patient's clinical findings is aortic dissection. This condition is characterized by the sudden onset of severe abdominal and bilateral lower extremity pain, cyanosis of the lower body, and paresthesias. It can also cause extensive anterolateral akinesis of the left ventricle, as well as sinus tachycardia on ECG. Treatment typically involves emergency surgery to repair the damaged aorta.
79
Five days after admission to the hospital for management of an ST-elevation myocardial infarction, a 59-year-old
man has the sudden onset of severe abdominal and bilateral lower extremity pain, cyanosis of the lower body, and
paresthesias. On hospital days 1 and 2, he received vasopressor and inotropic therapy. On hospital day 2,
echocardiography showed extensive anterolateral akinesis of the left ventricle. He has no history of serious illness.
His medications are metoprolol, lisinopril, simvastatin, and dalteparin. He appears to be in acute distress. Pulse is
105/min and regular, respirations are 24/min, and blood pressure is 80/60 mm Hg. Examination shows lower body
cyanosis to the level of the costal margins. Crackles are heard over the lower lung fields bilaterally. Distant heart
sounds but no murmurs are heard. The abdomen is nondistended and nontender. Pulses are 1+ in the upper
extremities and absent in the lower extremities. Muscle strength is 3/5 in the lower extremities. Sensation to touch
and temperature is decreased over the lower extremities. ECG shows sinus tachycardia. Which of the following is
the most likely explanation for this patient’s clinical findings and how did you arrive at your answer? Why are the other choices less likely?

(A) Aortic dissection
(B) Aortic embolism
(C) Pericardial tamponade
(D) Right ventricular failure
(E) Ventricular rupture
(B) Aortic embolismCorrect
The correct choice is (B) Aortic embolism.

Explanation:
The patient's clinical findings are consistent with aortic embolism. Aortic embolism occurs when a blood clot or other material (such as cholesterol plaque) becomes lodged in the aorta, blocking blood flow to the lower extremities. This can result in severe abdominal and bilateral lower extremity pain, cyanosis of the lower body, and paresthesias. The absence of pulses in the lower extremities, decreased muscle strength, and decreased sensation further support the diagnosis of aortic embolism.

The patient's history of ST-elevation myocardial infarction and the use of vasopressor and inotropic therapy suggest a hypercoagulable state, increasing the risk of embolism. The presence of crackles over the lower lung fields suggests pulmonary edema, which can occur as a result of decreased cardiac output and increased pulmonary venous pressure due to the embolism.

The other choices are less likely based on the clinical findings:
(A) Aortic dissection typically presents with severe tearing or ripping chest pain that radiates to the back. The absence of chest pain and the presence of abdominal and lower extremity symptoms make aortic dissection less likely.
(C) Pericardial tamponade would typically present with muffled heart sounds, jugular venous distention, and pulsus paradoxus. The absence of these findings makes pericardial tamponade less likely.
(D) Right ventricular failure would typically present with jugular venous distention, hepatomegaly, and peripheral edema. The absence of these findings makes right ventricular failure less likely.
(E) Ventricular rupture would typically present with acute onset of severe chest pain, hypotension, and signs of cardiac tamponade. The absence of chest pain and the presence of abdominal and lower extremity symptoms make ventricular rupture less likely.
Based on the information provided, the most likely explanation for the patient's symptoms is aortic dissection. This is because the patient has sudden onset of severe abdominal and bilateral lower extremity pain, cyanosis of the lower body, and paresthesias, which are all symptoms of aortic dissection. Additionally, the patient has a pulse of 105/min and regular, and crackles are heard over the lower lung fields bilaterally, which are also indicative of aortic dissection.

The other options, aortic embolism, pericardial tamponade, right ventricular failure, and ventricular rupture, are less likely because they do not typically cause the symptoms described in the scenario. For example, pericardial tamponade is characterized by fluid accumulation in the pericardium, which can cause chest pain and shortness of breath, but typically does not cause the severe abdominal and bilateral lower extremity pain and cyanosis of the lower body seen in this patient.
80
A 75-year-old man comes to the clinic because of a 2-year history of progressive shortness of breath. He also has
had an occasional dry cough for the past 6 months. He has not had chest pain, palpitations, or swelling of his legs.
Medical history is unremarkable, and the patient takes no medications. He has smoked one pack of cigarettes daily
for 57 years. He does not drink alcoholic beverages. Vital signs are temperature 37.0°C (98.6°F), pulse 92/min,
respirations 18/min, and blood pressure 122/76 mm Hg. Pulse oximetry on room air shows an oxygen saturation of
89%. The patient has mild dyspnea when he speaks. Auscultation of the lungs discloses fine and dry crackles in the
bases of both lungs. Cardiac examination discloses a normal S1 and S2. No extra heart sounds are heard. Jugular
venous pulsation is measured at 2 cm above the sternal angle with the head of the bed elevated at 30 degrees.
Examination of the extremities shows clubbing of the digits of both hands. Results of complete blood count, serum
chemistry profile, and urinalysis are within the reference ranges. Chest x-ray shows scattered reticular opacities in
both lung bases. There is no hilar lymphadenopathy. Which of the following is the most appropriate next step in
evaluation?


(A) Echocardiography
(B) High-resolution CT scan of the chest
(C) Lung biopsy
(D) Serum antiglomerular basement membrane antibody assay
(E) Thoracic PET scan
(B) High-resolution CT scan of the chest
Correct
The correct choice is (B) High-resolution CT scan of the chest.

Explanation:
Based on the patient's history and physical examination findings, the most likely diagnosis is interstitial lung disease (ILD). ILD refers to a group of lung disorders characterized by inflammation and scarring of the lung tissue. The patient's symptoms of progressive shortness of breath, dry cough, and crackles on lung auscultation are consistent with ILD.

The presence of clubbing of the digits suggests chronic hypoxia, which is commonly seen in ILD. The low oxygen saturation of 89% on room air further supports this diagnosis.

A high-resolution CT scan of the chest is the most appropriate next step in evaluation. This imaging modality provides detailed images of the lung parenchyma and can help identify the pattern and extent of lung involvement in ILD. It can also help differentiate between different types of ILD, such as idiopathic pulmonary fibrosis, sarcoidosis, or connective tissue disease-associated ILD.

Echocardiography (choice A) is not indicated in this patient as there are no signs or symptoms of heart failure or cardiac abnormalities on examination.

Lung biopsy (choice C) may be considered in certain cases of ILD, particularly when the diagnosis is unclear or if specific treatment options depend on the histopathological findings. However, it is an invasive procedure and is not the initial step in evaluation.

Serum antiglomerular basement membrane antibody assay (choice D) is used to diagnose anti-glomerular basement membrane (anti-GBM) disease, which typically presents with rapidly progressive glomerulonephritis and pulmonary hemorrhage. The patient's clinical presentation and imaging findings are not consistent with this diagnosis.

Thoracic PET scan (choice E) is not indicated in the evaluation of ILD. PET scans are primarily used in the evaluation of malignancies and are not the initial step in the evaluation of ILD.
Based on the information provided, the most appropriate next step in evaluation would be a high-resolution CT scan of the chest. This type of imaging test can provide detailed pictures of the inside of the chest and can help identify the cause of the patient's symptoms, such as lung cancer or chronic obstructive pulmonary disease (COPD). Other tests, such as echocardiography and lung biopsy, may also be helpful in certain cases but are not specifically mentioned as being indicated based on the information provided.
81
A 75-year-old man comes to the clinic because of a 2-year history of progressive shortness of breath. He also has
had an occasional dry cough for the past 6 months. He has not had chest pain, palpitations, or swelling of his legs.
Medical history is unremarkable, and the patient takes no medications. He has smoked one pack of cigarettes daily
for 57 years. He does not drink alcoholic beverages. Vital signs are temperature 37.0°C (98.6°F), pulse 92/min,
respirations 18/min, and blood pressure 122/76 mm Hg. Pulse oximetry on room air shows an oxygen saturation of
89%. The patient has mild dyspnea when he speaks. Auscultation of the lungs discloses fine and dry crackles in the
bases of both lungs. Cardiac examination discloses a normal S1 and S2. No extra heart sounds are heard. Jugular
venous pulsation is measured at 2 cm above the sternal angle with the head of the bed elevated at 30 degrees.
Examination of the extremities shows clubbing of the digits of both hands. Results of complete blood count, serum
chemistry profile, and urinalysis are within the reference ranges. Chest x-ray shows scattered reticular opacities in
both lung bases. There is no hilar lymphadenopathy. Which of the following is the most appropriate next step in
evaluation? Why are the other options not considered as the next step?


(A) Echocardiography
(B) High-resolution CT scan of the chest
(C) Lung biopsy
(D) Serum antiglomerular basement membrane antibody assay
(E) Thoracic PET scan
(B) High-resolution CT scan of the chest
Correct
The most appropriate next step in evaluation for this patient is (B) High-resolution CT scan of the chest.

The patient's history and physical examination findings are suggestive of a chronic lung disease, most likely related to his long history of smoking. The presence of progressive shortness of breath, dry cough, crackles on lung auscultation, and clubbing of the digits are all consistent with a chronic lung disease such as interstitial lung disease or chronic obstructive pulmonary disease (COPD).

A high-resolution CT scan of the chest is the most appropriate next step in evaluation because it can provide detailed imaging of the lung parenchyma and help identify the underlying cause of the patient's symptoms. It can help differentiate between different types of interstitial lung disease, assess the extent and severity of lung involvement, and guide further management decisions.

The other options are not considered as the next step for the following reasons:

(A) Echocardiography: While heart failure can cause shortness of breath, the absence of other cardiac findings on examination (normal S1 and S2, no extra heart sounds) makes heart failure less likely in this patient. Additionally, the presence of crackles on lung auscultation and the findings on chest x-ray suggest a primary lung pathology rather than a cardiac cause.

(C) Lung biopsy: Lung biopsy is an invasive procedure and is typically reserved for cases where the diagnosis remains uncertain after non-invasive imaging and other tests. In this case, a high-resolution CT scan can provide valuable information and may obviate the need for a lung biopsy.

(D) Serum antiglomerular basement membrane antibody assay: This test is used to diagnose anti-glomerular basement membrane (anti-GBM) disease, a rare autoimmune disorder that primarily affects the kidneys and lungs. However, the patient's clinical presentation and imaging findings are not consistent with anti-GBM disease, making this test unnecessary at this point.

(E) Thoracic PET scan: A positron emission tomography (PET) scan is a nuclear medicine imaging technique that is primarily used for cancer staging and evaluation of treatment response. It is not the most appropriate next step in this patient's evaluation because there is no indication of malignancy based on the history, physical examination, or chest x-ray findings. A high-resolution CT scan would be a more appropriate initial imaging modality.
The most appropriate next step in the evaluation of this patient would be a high-resolution CT scan of the chest. This imaging test can provide detailed information about the structure and function of the lungs and can help to identify the cause of the patient's symptoms. Echocardiography, lung biopsy, and thoracic PET scan are not typically indicated in this situation, as they are not typically used to diagnose the type of lung disorder that is most likely causing the patient's symptoms. Additionally, a serum antiglomerular basement membrane antibody assay is not typically performed in this situation, as it is used to diagnose a specific autoimmune disorder that is not suspected based on the patient's presentation.
82
A previously healthy 20-year-old woman, who is a sophomore in college, is brought to the emergency department
by her roommates because of an 8-hour history of weakness and vomiting blood and a 2-day history of dizziness.
She has no history of similar symptoms. She says she overeats in response to the stress of her classes. She induces
vomiting after consuming a significant amount of food because she feels "too full" and "out of control." She now
vomits almost daily. She feels she is slightly overweight. She is physically active and exercises moderately two to
three times weekly. She does not use laxatives or diuretics. She has many friends and enjoys going out socially. She
takes no medications. She drinks three to four beers on weekends and does not use illicit drugs. She appears
uncomfortable and is tearful and mildly diaphoretic. She is 157 cm (5 ft 2 in) tall and weighs 59 kg (130 lb); BMI is
24 kg/m². Temperature is 37.5°C (99.5°F), pulse is 115/min and regular, and blood pressure is 100/68 mm Hg.
Examination shows tender parotid glands bilaterally and poor dentition. Results of laboratory studies are most likely
to show which of the following in this patient?

(A) K+ is Decreased, Cl⁻ is decreased, HCO³⁻ is decreased
(B) K+ is Decreased, Cl⁻ is decreased, HCO³⁻ is increased
(C) K+ is Decreased, Cl⁻ is increased, HCO³⁻ is decreased
(D) K+ is Increased, Cl⁻ is decreased, HCO³⁻ is increased
(E) K+ is Increased, Cl⁻ is increased, HCO³⁻ is decreased
(F) K+ is Increased, Cl⁻ is increased, HCO³⁻ is increased
(B) K⁺ Decreased Cl⁻ decreased HCO³⁻ increased
Correct
The correct answer is (B) K+ is Decreased, Cl⁻ is decreased, HCO³⁻ is increased.

Explanation:
This patient's clinical presentation is consistent with bulimia nervosa, which is characterized by recurrent episodes of binge eating followed by inappropriate compensatory behaviors to prevent weight gain, such as self-induced vomiting. The patient's symptoms of weakness, vomiting blood, and dizziness are likely due to electrolyte abnormalities and dehydration resulting from repeated vomiting.

In bulimia nervosa, vomiting leads to loss of gastric acid, which can result in metabolic alkalosis. This is reflected in the laboratory findings of decreased chloride (Cl⁻) and increased bicarbonate (HCO³⁻) levels. The loss of gastric acid also leads to hypokalemia (decreased potassium, K⁺) due to the loss of hydrogen ions and subsequent exchange with potassium ions in the renal tubules.

The tender parotid glands and poor dentition seen on examination are consistent with the chronic vomiting associated with bulimia nervosa. The tearfulness and diaphoresis may be related to the emotional distress associated with the disorder. The patient's slightly overweight perception and the use of vomiting as a means of weight control are also characteristic of bulimia nervosa.
It is difficult to answer this question without more information about the patient's condition and the results of laboratory studies. However, some potential laboratory findings in a patient with a history of vomiting blood and weakness could include low levels of potassium (K+), decreased levels of chloride (Cl-), and decreased levels of bicarbonate (HCO3-). This would be consistent with answer choice (A). It is important for the patient to receive prompt medical care in order to diagnose and treat the underlying cause of her symptoms.
83
A previously healthy 20-year-old woman, who is a sophomore in college, is brought to the emergency department
by her roommates because of an 8-hour history of weakness and vomiting blood and a 2-day history of dizziness.
She has no history of similar symptoms. She says she overeats in response to the stress of her classes. She induces
vomiting after consuming a significant amount of food because she feels "too full" and "out of control." She now
vomits almost daily. She feels she is slightly overweight. She is physically active and exercises moderately two to
three times weekly. She does not use laxatives or diuretics. She has many friends and enjoys going out socially. She
takes no medications. She drinks three to four beers on weekends and does not use illicit drugs. She appears
uncomfortable and is tearful and mildly diaphoretic. She is 157 cm (5 ft 2 in) tall and weighs 59 kg (130 lb); BMI is
24 kg/m². Temperature is 37.5°C (99.5°F), pulse is 115/min and regular, and blood pressure is 100/68 mm Hg.
Examination shows tender parotid glands bilaterally and poor dentition. Results of laboratory studies are most likely
to show which of the following in this patient? How did you arrive at your answer and why are the other choices not likely?

(A) K+ is Decreased, Cl⁻ is decreased, HCO³⁻ is decreased
(B) K+ is Decreased, Cl⁻ is decreased, HCO³⁻ is increased
(C) K+ is Decreased, Cl⁻ is increased, HCO³⁻ is decreased
(D) K+ is Increased, Cl⁻ is decreased, HCO³⁻ is increased
(E) K+ is Increased, Cl⁻ is increased, HCO³⁻ is decreased
(F) K+ is Increased, Cl⁻ is increased, HCO³⁻ is increased
(B) K⁺ Decreased Cl⁻ decreased HCO³⁻ increased
Correct
The correct answer is (B) K+ is Decreased, Cl⁻ is decreased, HCO³⁻ is increased.

The patient's history and physical examination findings are consistent with bulimia nervosa, which is characterized by recurrent episodes of binge eating followed by inappropriate compensatory behaviors to prevent weight gain, such as self-induced vomiting. The patient's symptoms of weakness, vomiting blood, and dizziness are likely due to electrolyte abnormalities and dehydration resulting from repeated vomiting.

In bulimia nervosa, vomiting leads to loss of gastric acid, which can result in metabolic alkalosis. This is reflected by an increased bicarbonate (HCO³⁻) level. Additionally, vomiting can lead to loss of chloride (Cl⁻) and potassium (K⁺) ions, resulting in decreased levels of both electrolytes.

The other answer choices are not likely because they do not match the expected electrolyte abnormalities seen in bulimia nervosa.

Choice (A) is incorrect because it suggests decreased levels of all three electrolytes (K⁺, Cl⁻, and HCO³⁻), which is not consistent with the expected findings in bulimia nervosa.

Choice (C) is incorrect because it suggests increased chloride (Cl⁻) levels, which is not consistent with the expected findings in bulimia nervosa.

Choice (D) is incorrect because it suggests increased potassium (K⁺) levels, which is not consistent with the expected findings in bulimia nervosa.

Choice (E) is incorrect because it suggests increased levels of both chloride (Cl⁻) and bicarbonate (HCO³⁻), which is not consistent with the expected findings in bulimia nervosa.

Choice (F) is incorrect because it suggests increased levels of all three electrolytes (K⁺, Cl⁻, and HCO³⁻), which is not consistent with the expected findings in bulimia nervosa.
Based on the information provided in the scenario, the most likely laboratory results for this patient would be K+ is Decreased, Cl⁻ is decreased, HCO³⁻ is decreased (Option A). Vomiting blood, or hematemesis, can result in the loss of electrolytes, including potassium, chlorine, and bicarbonate. Additionally, the patient's history of inducing vomiting after eating can lead to electrolyte imbalances. The other options are not as likely because they do not reflect the effects of vomiting blood and inducing vomiting on electrolyte levels.
84
A 58-year-old man comes to the clinic for a follow-up examination 3 months after beginning warfarin therapy for
deep venous thrombosis. He noted blood in his urine 1 week ago but has not seen any since that time. He has not had
pain, fever, or increased urinary frequency. He has not sustained any recent trauma. His prothrombin time and INR
have been within the therapeutic ranges during his previous measurements. He has hypertension and benign
prostatic hyperplasia. Medications are warfarin and hydrochlorothiazide. He smoked one pack of cigarettes daily for
40 years but quit when diagnosed with deep venous thrombosis. Temperature is 35.9°C (96.6°F), pulse is 88/min,
respirations are 18/min, and blood pressure is 135/87 mm Hg. The prostate is moderately enlarged; no nodules are
noted.

Laboratory studies showed that the Prothrombin time is 19 sec (INR=2.5), serum Urea nitrogen is 18 mg/dL, serum Creatinine is 1 mg/dL, and Prostate-specific antigen is 3.8 ng/mL (N<4).

The laboratory result on urine showed the following: blood is 3+, protein is 1+, RBC is too numerous to count, WBC is 2/hpf, no casts, and negative for Nitrites and Leukocyte esterase.

In addition to a CT scan of the abdomen, which of the following is the most appropriate next step in management?

(A) Adjustment of the warfarin dosage to maintain an INR between 1.5 and 2.0
(B) Ciprofloxacin therapy
(C) Cystoscopy
(D) Discontinuation of hydrochlorothiazide therapy
(E) Finasteride therapy
(C) CystoscopyCorrect
The correct answer is (C) Cystoscopy.

Explanation:
The patient's presentation of painless hematuria, along with the presence of blood in the urine and numerous red blood cells on microscopic examination, raises concern for a genitourinary source of bleeding. Given the patient's age, history of smoking, and the finding of an enlarged prostate, the most appropriate next step in management is cystoscopy. Cystoscopy allows for direct visualization of the bladder and urethra, which can help identify the source of bleeding, such as bladder cancer or benign prostatic hyperplasia. This is especially important in patients on anticoagulant therapy, as the risk of genitourinary bleeding may be increased. A CT scan of the abdomen may be useful to evaluate the kidneys and upper urinary tract, but it would not provide direct visualization of the bladder and urethra. Adjustment of the warfarin dosage is not indicated, as the patient's prothrombin time and INR have been within the therapeutic range. Ciprofloxacin therapy would be appropriate if there were signs of a urinary tract infection, such as fever or increased urinary frequency, but the patient does not have these symptoms. Discontinuation of hydrochlorothiazide therapy or initiation of finasteride therapy would not address the underlying cause of the hematuria.
Based on the information provided, the most appropriate next step in management would be to perform a cystoscopy. This is because the patient has blood in his urine, which could be a sign of a urinary tract infection or other urinary tract issue. A cystoscopy would allow the healthcare provider to directly visualize the inside of the patient's bladder and urethra to determine the cause of the bleeding and take appropriate action.
85
A 58-year-old man comes to the clinic for a follow-up examination 3 months after beginning warfarin therapy for
deep venous thrombosis. He noted blood in his urine 1 week ago but has not seen any since that time. He has not had
pain, fever, or increased urinary frequency. He has not sustained any recent trauma. His prothrombin time and INR
have been within the therapeutic ranges during his previous measurements. He has hypertension and benign
prostatic hyperplasia. Medications are warfarin and hydrochlorothiazide. He smoked one pack of cigarettes daily for
40 years but quit when diagnosed with deep venous thrombosis. Temperature is 35.9°C (96.6°F), pulse is 88/min,
respirations are 18/min, and blood pressure is 135/87 mm Hg. The prostate is moderately enlarged; no nodules are
noted.

Laboratory studies showed that the Prothrombin time is 19 sec (INR=2.5), serum Urea nitrogen is 18 mg/dL, serum Creatinine is 1 mg/dL, and Prostate-specific antigen is 3.8 ng/mL (N<4).

The laboratory result on urine showed the following: blood is 3+, protein is 1+, RBC is too numerous to count, WBC is 2/hpf, no casts, and negative for Nitrites and Leukocyte esterase.

In addition to a CT scan of the abdomen, which of the following is the most appropriate next step in management? What is the rationale for your answer and the other choices?

(A) Adjustment of the warfarin dosage to maintain an INR between 1.5 and 2.0
(B) Ciprofloxacin therapy
(C) Cystoscopy
(D) Discontinuation of hydrochlorothiazide therapy
(E) Finasteride therapy
(C) CystoscopyCorrect
The correct answer is (C) Cystoscopy.

The patient's presentation of painless hematuria, along with the presence of blood in the urine and numerous red blood cells on microscopic examination, raises concern for a genitourinary source of bleeding. Given the patient's age, history of smoking, and the finding of an enlarged prostate, the most appropriate next step in management is cystoscopy.

Cystoscopy is a procedure that allows direct visualization of the urethra and bladder using a flexible or rigid scope. It is the gold standard for evaluating the lower urinary tract and can help identify the source of bleeding, such as bladder tumors or other abnormalities. In this case, cystoscopy is necessary to evaluate the patient's hematuria and rule out any underlying genitourinary pathology.

The other answer choices are not appropriate in this scenario:

(A) Adjustment of the warfarin dosage to maintain an INR between 1.5 and 2.0: The patient's INR is already within the therapeutic range, and adjusting the warfarin dosage is unlikely to be the cause of the hematuria. The focus should be on evaluating the source of bleeding rather than changing the anticoagulation regimen.

(B) Ciprofloxacin therapy: Ciprofloxacin is an antibiotic commonly used to treat urinary tract infections. However, there is no evidence of an active urinary tract infection in this patient, as the urine culture was negative for nitrites and leukocyte esterase. Therefore, initiating antibiotic therapy would not be appropriate.

(D) Discontinuation of hydrochlorothiazide therapy: Hydrochlorothiazide is a diuretic commonly used to treat hypertension and fluid retention. While it can cause electrolyte imbalances and kidney dysfunction, there is no evidence to suggest that it is the cause of the patient's hematuria. Discontinuing hydrochlorothiazide would not address the underlying issue.

(E) Finasteride therapy: Finasteride is a medication used to treat benign prostatic hyperplasia (BPH). While BPH can cause urinary symptoms, such as increased urinary frequency and difficulty urinating, it is not typically associated with painless hematuria. Initiating finasteride therapy would not be appropriate in this case.

In summary, given the patient's presentation of painless hematuria, cystoscopy is the most appropriate next step in management to evaluate the genitourinary source of bleeding.
The most appropriate next step in management would be cystoscopy, which is a procedure that allows the doctor to directly visualize the inside of the bladder and urethra using a small camera. This will help determine the cause of the blood in the urine, which could be related to the patient's benign prostatic hyperplasia or potentially to a more serious condition such as bladder cancer. The other options are not as directly relevant to the patient's current symptoms and may not address the underlying cause of the blood in the urine.
86
A 32-year-old woman is brought to the emergency department immediately after being involved in a motor vehicle
collision. She was the unrestrained driver. On arrival, she says she has severe pain in her right groin. On
examination, any movement of the right hip produces pain. An x-ray of the pelvis shows a completely displaced
fracture of the femoral neck. In addition to intravenous morphine, which of the following is the most appropriate
management?

(A) Bed rest
(B) Closed reduction followed by traction
(C) Hip spica hinge brace
(D) Limited weight bearing with use of crutches
(E) Operative reduction and internal fixation
(E) Operative reduction and internal fixation
Correct
The correct answer is (E) Operative reduction and internal fixation.

Explanation:
In the case of a completely displaced fracture of the femoral neck, operative reduction and internal fixation is the most appropriate management. This involves surgical intervention to realign the fractured bone fragments and fix them in place with internal fixation devices such as screws or plates.

Operative reduction and internal fixation is preferred over non-operative management options such as bed rest, closed reduction followed by traction, hip spica hinge brace, or limited weight bearing with crutches. These non-operative options may not provide adequate stability and alignment of the fractured bone, leading to complications such as non-union or avascular necrosis.

Surgical intervention allows for better anatomical alignment, which promotes proper healing and reduces the risk of complications. It also allows for early mobilization and rehabilitation, which is important for restoring function and preventing muscle wasting and loss of mobility, especially in younger patients like the 32-year-old woman in this case.
The most appropriate management for a completely displaced fracture of the femoral neck in a 32-year-old woman would be operative reduction and internal fixation. This procedure involves surgically realigning the bones in the hip joint and using screws or pins to hold them in place. This will provide the best chance of a successful recovery and allow the patient to regain full function of their hip. Bed rest, closed reduction and traction, and the use of a hip spica hinge brace are all less effective treatments for this type of fracture. Limited weight bearing with crutches may be recommended after the surgery, but it would not be sufficient as the sole treatment.


87
A 32-year-old woman is brought to the emergency department immediately after being involved in a motor vehicle
collision. She was the unrestrained driver. On arrival, she says she has severe pain in her right groin. On
examination, any movement of the right hip produces pain. An x-ray of the pelvis shows a completely displaced
fracture of the femoral neck. In addition to intravenous morphine, which of the following is the most appropriate
management? Why is your answer the better choice compared to the others?

(A) Bed rest
(B) Closed reduction followed by traction
(C) Hip spica hinge brace
(D) Limited weight bearing with use of crutches
(E) Operative reduction and internal fixation
(E) Operative reduction and internal fixation
Correct
The correct answer is (E) Operative reduction and internal fixation.

Explanation:
In the case of a completely displaced fracture of the femoral neck, operative reduction and internal fixation is the most appropriate management. This involves surgical intervention to realign the fractured bone fragments and fix them in place using screws, plates, or other fixation devices.

The rationale behind choosing operative reduction and internal fixation over the other options is as follows:

(A) Bed rest: Bed rest alone is not sufficient for the management of a completely displaced fracture of the femoral neck. It does not address the need for realignment and fixation of the fractured bone fragments.

(B) Closed reduction followed by traction: Closed reduction and traction may be appropriate for certain types of fractures, but in the case of a completely displaced fracture of the femoral neck, operative reduction and internal fixation is the preferred treatment option.

(C) Hip spica hinge brace: A hip spica hinge brace is not appropriate for the management of a completely displaced fracture of the femoral neck. It is more commonly used for the management of hip dislocations or certain types of pelvic fractures.

(D) Limited weight bearing with use of crutches: Limited weight bearing with the use of crutches may be recommended as part of the postoperative rehabilitation process, but it is not sufficient as the primary management for a completely displaced fracture of the femoral neck.

Operative reduction and internal fixation is the preferred management for completely displaced fractures of the femoral neck because it allows for proper realignment and stabilization of the fractured bone fragments. This approach helps to promote healing, restore normal anatomy, and minimize the risk of complications such as non-union or avascular necrosis.
The most appropriate management in this case would be operative reduction and internal fixation. A displaced fracture of the femoral neck, as seen on the x-ray, requires surgical intervention to realign the bone and stabilize it with internal fixation. This is the best choice because it will provide the most effective and lasting relief from pain and allow for the best possible recovery and healing of the bone.
88
A 57-year-old man comes to the physician 1 week after he had a 45-minute episode of slurred speech and
clumsiness of his right hand. The episode occurred during his daily walk; the symptoms resolved during the next 24
hours. He has no history of similar symptoms. He has a 20-year history of hypertension and a 15-year history of
hypercholesterolemia controlled with a low-cholesterol diet and daily exercise. He takes no medications. He has
smoked one pack of cigarettes daily for 40 years and drinks one to two glasses of wine nightly with dinner. He is
alert and fully oriented. He is 178 cm (5 ft 10 in) tall and weighs 86 kg (190 lb); BMI is 27 kg/m². His temperature is
37.0°C (98.6°F), pulse is 88/min, respirations are 10/min, and blood pressure is 170/90 mm Hg. Examination,
including neurologic examination, shows no other abnormalities. Daily enteric-coated aspirin therapy is begun.
Which of the following is the most appropriate next step to decrease this patient’s risk for subsequent cerebral
infarction?


(A) Alcohol cessation
(B) Smoking cessation
(C) Weight loss program
(D) Antihypertensive therapy
(E) Cholesterol-lowering therapy
(D) Antihypertensive therapyCorrect
The correct choice is (D) Antihypertensive therapy.

Explanation:
This patient has a history of hypertension and a recent episode of transient ischemic attack (TIA), which is a warning sign for an impending stroke. Hypertension is a major risk factor for stroke, and controlling blood pressure is crucial in reducing the risk of subsequent cerebral infarction. Antihypertensive therapy is the most appropriate next step in this patient's management.

Alcohol cessation (choice A) may be beneficial for overall health, but it is not the most appropriate next step in this patient's case. The patient's alcohol consumption is within moderate limits, and the main focus should be on controlling his hypertension.

Smoking cessation (choice B) is important for reducing the risk of cardiovascular disease, including stroke. However, in this patient's case, the most immediate concern is his hypertension, which should be addressed first.

Weight loss program (choice C) may be beneficial for overall health and reducing the risk of cardiovascular disease. However, in this patient's case, the most immediate concern is his hypertension, which should be addressed first.

Cholesterol-lowering therapy (choice E) is important for managing hypercholesterolemia and reducing the risk of cardiovascular disease. However, in this patient's case, the most immediate concern is his hypertension, which should be addressed first. Cholesterol-lowering therapy can be considered as part of long-term management.
The most appropriate next step to decrease this patient's risk for subsequent cerebral infarction would be to begin antihypertensive therapy. High blood pressure is a major risk factor for stroke, and reducing it can help prevent future episodes of slurred speech and clumsiness. While alcohol cessation, smoking cessation, and weight loss may also be beneficial for this patient, they are not as directly related to reducing the risk of stroke as controlling blood pressure through medication. Cholesterol-lowering therapy may be beneficial for reducing the patient's risk of heart disease, but it is not as directly related to reducing the risk of stroke as controlling blood pressure.
89
A 57-year-old man comes to the physician 1 week after he had a 45-minute episode of slurred speech and
clumsiness of his right hand. The episode occurred during his daily walk; the symptoms resolved during the next 24
hours. He has no history of similar symptoms. He has a 20-year history of hypertension and a 15-year history of
hypercholesterolemia controlled with a low-cholesterol diet and daily exercise. He takes no medications. He has
smoked one pack of cigarettes daily for 40 years and drinks one to two glasses of wine nightly with dinner. He is
alert and fully oriented. He is 178 cm (5 ft 10 in) tall and weighs 86 kg (190 lb); BMI is 27 kg/m². His temperature is
37.0°C (98.6°F), pulse is 88/min, respirations are 10/min, and blood pressure is 170/90 mm Hg. Examination,
including neurologic examination, shows no other abnormalities. Daily enteric-coated aspirin therapy is begun.
Which of the following is the most appropriate next step to decrease this patient’s risk for subsequent cerebral
infarction and why is it the best choice? What is your reason on not choosing the other options?


(A) Alcohol cessation
(B) Smoking cessation
(C) Weight loss program
(D) Antihypertensive therapy
(E) Cholesterol-lowering therapy
(D) Antihypertensive therapyCorrect
The correct choice is (D) Antihypertensive therapy.

Explanation:
The patient in the scenario has a history of hypertension, which is a major risk factor for cerebral infarction (stroke). Hypertension can lead to the development of atherosclerosis and narrowing of the blood vessels, increasing the risk of blood clots and subsequent stroke. Therefore, the most appropriate next step to decrease this patient's risk for subsequent cerebral infarction is to initiate antihypertensive therapy.

Option (A) Alcohol cessation may be beneficial for overall health, but it is not the most appropriate next step in this case. The patient's alcohol consumption is within moderate limits, and the primary risk factor for stroke in this patient is hypertension.

Option (B) Smoking cessation is also important for overall health, but it is not the most appropriate next step in this case. While smoking is a risk factor for stroke, the patient's hypertension is a more significant risk factor that needs to be addressed first.

Option (C) Weight loss program may be beneficial for the patient's overall health and may help in reducing the risk of hypertension. However, given the patient's history of hypertension and the immediate need to decrease the risk of subsequent cerebral infarction, initiating antihypertensive therapy is a more appropriate next step.

Option (E) Cholesterol-lowering therapy is important for managing hypercholesterolemia, but it is not the most appropriate next step in this case. The patient's primary risk factor for stroke is hypertension, and addressing this risk factor should take precedence over cholesterol management.
The most appropriate next step to decrease this patient's risk for subsequent cerebral infarction would be to begin antihypertensive therapy. Hypertension is a major risk factor for cerebral infarction, and reducing blood pressure can help to prevent future episodes of slurred speech and clumsiness of the hand.

Alcohol cessation and smoking cessation may also help to reduce the risk of cerebral infarction, but they are not as important as controlling blood pressure in this case. The patient's cholesterol levels are already controlled through diet and exercise, so starting cholesterol-lowering therapy would not likely have a significant impact on their risk of cerebral infarction. Similarly, losing weight may help to improve the patient's overall health, but it is not as immediately necessary as controlling blood pressure in this case.
90
A 27-year-old primigravid woman at 21 weeks’ gestation comes to the emergency department because of a 2-day
history of moderate headache, shortness of breath, nausea, vomiting, muscle aches, and malaise. She also has had
cough occasionally productive of sputum. Pregnancy had been uncomplicated. She has no history of serious illness,
and her only medication is a prenatal vitamin. She immigrated to the USA from India 6 months ago. Temperature is
38.3°C (100.9°F), pulse is 100/min, respirations are 18/min, and blood pressure is 100/60 mm Hg. On examination,
breath sounds are decreased at the right lung base. Fundal height is 22 cm. Fetal heart rate is 160/min. Chest x-ray
shows right-sided interstitial infiltrates. Which of the following is the most likely infectious agent?


(A) Haemophilus influenzae
(B) Influenza A virus
(C) Legionella pneumophila
(D) Mycobacterium tuberculosis
(E) Pseudomonas aeruginosa
(F) Streptococcus pneumoniae
(B) Influenza A virusCorrect
The correct choice is (B) Influenza A virus.

Explanation:
Based on the patient's symptoms of headache, shortness of breath, nausea, vomiting, muscle aches, and malaise, along with a cough occasionally productive of sputum, and the presence of right-sided interstitial infiltrates on chest x-ray, the most likely infectious agent is the Influenza A virus.

Influenza A virus is a common cause of respiratory tract infections and can lead to symptoms such as headache, muscle aches, malaise, and cough. It can also cause pneumonia, which can present with decreased breath sounds and infiltrates on chest x-ray. The patient's recent immigration from India may have exposed her to different strains of the virus, increasing her risk of infection.

Haemophilus influenzae is a bacteria that can cause respiratory tract infections, but it is less likely in this case as the symptoms and chest x-ray findings are more consistent with a viral infection.

Legionella pneumophila is a bacteria that can cause Legionnaires' disease, a severe form of pneumonia. However, the symptoms and chest x-ray findings in this case are not typical of Legionnaires' disease.

Mycobacterium tuberculosis is the bacteria that causes tuberculosis, but the symptoms and chest x-ray findings in this case are not consistent with tuberculosis.

Pseudomonas aeruginosa is a bacteria that can cause respiratory tract infections, but it is less likely in this case as the symptoms and chest x-ray findings are more consistent with a viral infection.

Streptococcus pneumoniae is a bacteria that can cause pneumonia, but the symptoms and chest x-ray findings in this case are more consistent with a viral infection.
Based on the symptoms and examination findings described in the question, the most likely infectious agent is influenza A virus (option B). Influenza, also known as the flu, is a common viral infection that affects the respiratory system and can cause symptoms such as fever, headache, muscle aches, and shortness of breath. Pregnant women are at increased risk of complications from the flu, including pneumonia, so it is important for them to receive the flu vaccine.
91
A 27-year-old primigravid woman at 21 weeks’ gestation comes to the emergency department because of a 2-day
history of moderate headache, shortness of breath, nausea, vomiting, muscle aches, and malaise. She also has had
cough occasionally productive of sputum. Pregnancy had been uncomplicated. She has no history of serious illness,
and her only medication is a prenatal vitamin. She immigrated to the USA from India 6 months ago. Temperature is
38.3°C (100.9°F), pulse is 100/min, respirations are 18/min, and blood pressure is 100/60 mm Hg. On examination,
breath sounds are decreased at the right lung base. Fundal height is 22 cm. Fetal heart rate is 160/min. Chest x-ray
shows right-sided interstitial infiltrates. Which of the following is the most likely infectious agent? Why are the other choices most likely not the cause?


(A) Haemophilus influenzae
(B) Influenza A virus
(C) Legionella pneumophila
(D) Mycobacterium tuberculosis
(E) Pseudomonas aeruginosa
(F) Streptococcus pneumoniae
(B) Influenza A virusCorrect
The correct choice is (B) Influenza A virus.

Explanation:
Based on the patient's symptoms of headache, shortness of breath, nausea, vomiting, muscle aches, and malaise, along with the presence of cough and productive sputum, the most likely infectious agent is the Influenza A virus.

The patient's symptoms are consistent with influenza, which commonly presents with fever, headache, muscle aches, malaise, and respiratory symptoms such as cough and shortness of breath. The decreased breath sounds at the right lung base and the right-sided interstitial infiltrates on chest x-ray suggest a lower respiratory tract infection.

The other choices can be ruled out based on the clinical presentation and context information provided:

(A) Haemophilus influenzae: This bacterium is a common cause of respiratory tract infections, but it is less likely in this case because the patient's symptoms are more consistent with a viral infection.

(C) Legionella pneumophila: Legionella is a bacterium that can cause a severe form of pneumonia known as Legionnaires' disease. However, the patient's symptoms and chest x-ray findings are not typical of Legionella pneumonia.

(D) Mycobacterium tuberculosis: Tuberculosis is a chronic bacterial infection that typically presents with a chronic cough, weight loss, and night sweats. The acute onset of symptoms and the presence of interstitial infiltrates on chest x-ray make tuberculosis less likely in this case.

(E) Pseudomonas aeruginosa: Pseudomonas is a common cause of hospital-acquired pneumonia, but it is less likely in this case because the patient has no history of hospitalization or healthcare exposure.

(F) Streptococcus pneumoniae: Streptococcus pneumoniae is a common cause of community-acquired pneumonia, but it is less likely in this case because the patient's symptoms and chest x-ray findings are not typical of pneumococcal pneumonia.
Based on the symptoms and examination findings described in the question, the most likely cause of the infection is influenza A virus (option B). The other choices are unlikely because they are not typically associated with the symptoms and findings described. Haemophilus influenzae (option A) is a bacterium that can cause respiratory infections, but it is not typically associated with the symptoms described in this case. Legionella pneumophila (option C) is a bacterium that can cause Legionnaire's disease, which is a type of pneumonia, but it typically presents with different symptoms than those described in this case. Mycobacterium tuberculosis (option D) is a bacterium that causes tuberculosis, which is a chronic infection that typically presents with different symptoms than those described in this case. Pseudomonas aeruginosa (option E) is a bacterium that can cause respiratory infections, but it is not typically associated with the symptoms described in this case. Streptococcus pneumoniae (option F) is a bacterium that can cause pneumonia, but it typically presents with different symptoms than those described in this case.
92
A 7-year-old boy is brought to the physician because of a 1-year history of poor performance in school. His parents
say that he is bright, has many friends, and seems to want to do well in school. His teachers report that he seems
frustrated with his own progress and his inability to meet the expectations of his parents. Speech fluency and
articulation and motor skills are appropriate for age. Physical examination shows no abnormalities. When asked to
read during the examination, he has significant difficulty sounding out words he is unfamiliar with. Visual acuity
test and audiometry show no abnormalities. Which of the following is the most likely diagnosis?


(A) Attention-deficit/hyperactivity disorder
(B) Expressive language disorder
(C) Intellectual developmental disorder
(D) Learning disorder
(E) Social anxiety disorder (social phobia)
(F) Normal behavior
(D) Learning disorderCorrect
The correct choice is (D) Learning disorder.

Explanation:
Based on the information provided, the 7-year-old boy is experiencing poor performance in school, frustration with his own progress, and difficulty sounding out unfamiliar words while reading. These symptoms are indicative of a learning disorder.

Attention-deficit/hyperactivity disorder (ADHD) is characterized by symptoms such as inattention, hyperactivity, and impulsivity. While poor performance in school can be a symptom of ADHD, the boy's frustration with his own progress and difficulty with reading suggest a different diagnosis.

Expressive language disorder is characterized by difficulties with verbal expression, such as forming sentences and using appropriate vocabulary. However, the boy's speech fluency and articulation are reported to be appropriate for his age, making this diagnosis less likely.

Intellectual developmental disorder (IDD) is characterized by significant limitations in intellectual functioning and adaptive behavior. The boy's parents report that he is bright, and there are no abnormalities found during the physical examination, making IDD an unlikely diagnosis.

Social anxiety disorder (social phobia) is characterized by excessive fear or anxiety in social situations. While the boy's frustration with his own progress and inability to meet expectations may cause some anxiety, there is no mention of significant social anxiety symptoms in the scenario.

Therefore, the most likely diagnosis based on the provided information is a learning disorder. This diagnosis would explain the boy's difficulties with reading and poor performance in school despite his apparent intelligence and desire to do well. Further evaluation and assessment would be necessary to determine the specific type of learning disorder and develop appropriate interventions.
Based on the information provided, the most likely diagnosis is a learning disorder. Children with learning disorders often have difficulty with school work, even though they are bright and have good social skills. In this case, the boy's teachers report that he is struggling to meet expectations, and he has difficulty reading unfamiliar words. Physical examination and testing for vision and hearing show no abnormalities, so other potential causes of the boy's
93
A 7-year-old boy is brought to the physician because of a 1-year history of poor performance in school. His parents
say that he is bright, has many friends, and seems to want to do well in school. His teachers report that he seems
frustrated with his own progress and his inability to meet the expectations of his parents. Speech fluency and
articulation and motor skills are appropriate for age. Physical examination shows no abnormalities. When asked to
read during the examination, he has significant difficulty sounding out words he is unfamiliar with. Visual acuity
test and audiometry show no abnormalities. Which of the following is the most likely diagnosis and why are the other choices not?


(A) Attention-deficit/hyperactivity disorder
(B) Expressive language disorder
(C) Intellectual developmental disorder
(D) Learning disorder
(E) Social anxiety disorder (social phobia)
(F) Normal behavior
(D) Learning disorderCorrect
The correct choice is (D) Learning disorder.

Explanation:
The patient's history of poor performance in school, frustration with his own progress, and difficulty sounding out unfamiliar words during the examination are all indicative of a learning disorder. Learning disorders are characterized by difficulties in acquiring and using academic skills, such as reading, writing, and math, despite having normal intelligence and adequate educational opportunities.

Choice (A) Attention-deficit/hyperactivity disorder (ADHD) is not the most likely diagnosis in this case because there is no mention of symptoms such as inattention, hyperactivity, or impulsivity, which are characteristic of ADHD.

Choice (B) Expressive language disorder is not the most likely diagnosis because the patient's speech fluency and articulation are appropriate for his age. Expressive language disorder is characterized by difficulties in expressing oneself through speech, such as limited vocabulary, grammatical errors, and difficulty organizing thoughts.

Choice (C) Intellectual developmental disorder is not the most likely diagnosis because there is no mention of intellectual impairment or developmental delays in the patient's history or examination findings. Intellectual developmental disorder is characterized by significant limitations in intellectual functioning and adaptive behavior.

Choice (E) Social anxiety disorder (social phobia) is not the most likely diagnosis because there is no mention of social anxiety symptoms, such as fear or avoidance of social situations, in the patient's history. Social anxiety disorder is characterized by excessive fear of being embarrassed, humiliated, or judged by others.

Choice (F) Normal behavior is not the most likely diagnosis because the patient's poor performance in school and frustration with his own progress indicate that there is an underlying issue affecting his academic abilities.
The most likely diagnosis for the 7-year-old boy described in the scenario is a learning disorder, specifically a reading disorder (dyslexia). This is because the boy's difficulty with sounding out unfamiliar words suggests a difficulty with phonemic awareness, a common characteristic of dyslexia. The other conditions listed in the question are not as likely because they do not specifically address the boy's difficulty with reading. For example, attention-deficit/hyperactivity disorder (ADHD) is characterized by problems with attention, impulse control, and hyperactivity, but it is not specifically associated with difficulty with reading. Expressive language disorder is characterized by difficulty with language production, but it does not typically cause difficulty with reading. Intellectual developmental disorder is a broad term that refers to any intellectual disability, but it does not necessarily cause specific difficulty with reading. Social anxiety disorder (social phobia) is characterized by excessive fear of social situations, but it does not cause difficulty with reading. Finally, it is unlikely that the boy is experiencing "normal behavior" because his poor performance in school is a concern for his parents and teachers.
94
A 23-year-old woman who is an active-duty enlisted airman in the US Air Force comes to the base clinic because of
a 4-week history of an itchy sensation in her right ear. She has not had ringing in her ears or hearing loss. She works
as a fuel specialist and at first attributed her symptom to the earplugs she uses when she is on the flight line. The
symptom has not improved despite the patient's wearing headphones instead of the earplugs during the past 2 weeks.
She has no history of serious illness and takes no medications. Vital signs are within normal limits. During
examination, pulling of the right pinna inferiorly to examine the ear canal produces pain. The right ear canal appears
erythematous and edematous. The tympanic membrane cannot be fully visualized because of the presence of
cerumen. Examination of the left ear shows no abnormalities. Which of the following is the most likely diagnosis?


(A) Cerumen impaction
(B) Ear canal trauma
(C) Otitis externa
(D) Polychondritis
(E) Seborrheic dermatitis
(C) Otitis externaCorrect
The correct answer is (C) Otitis externa.

Explanation:
Based on the information provided, the patient presents with a 4-week history of an itchy sensation in her right ear, which has not improved despite changing from earplugs to headphones. During examination, pulling of the right pinna inferiorly produces pain, and the right ear canal appears erythematous and edematous. The tympanic membrane cannot be fully visualized due to the presence of cerumen. These findings are consistent with otitis externa.

Otitis externa, also known as swimmer's ear, is an infection or inflammation of the external ear canal. It is commonly caused by bacterial infection, most often by Pseudomonas aeruginosa or Staphylococcus aureus. Risk factors for otitis externa include local trauma, prolonged moisture exposure (such as swimming), and high humidity and temperature climates.

The symptoms of otitis externa include itching, pain, and a feeling of fullness in the ear. The ear canal may appear erythematous and edematous, and there may be a purulent discharge. Pulling on the pinna or tragus can reproduce pain. In severe cases, the tympanic membrane may be difficult to visualize due to swelling and debris.

Management of otitis externa involves pain relief and eradication of infection. Topical antibiotics and topical steroids are often prescribed. Patients are advised to avoid swimming until the infection has resolved. Follow-up with a primary care physician is recommended. In severe cases where the tympanic membrane is not visible, referral to an ENT surgeon may be necessary for aural toilet to remove debris from the auditory canal.

Cerumen impaction (choice A) can cause ear discomfort, but intense pain is uncommon. It is more commonly associated with symptoms of ear fullness or hearing impairment.

Ear canal trauma (choice B) may cause symptoms similar to otitis externa, but there is no mention of any traumatic event in the patient's history.

Polychondritis (choice D) is a rare autoimmune condition characterized by inflammation of cartilage throughout the body. It typically presents with symptoms such as joint pain, redness, and swelling. There is no mention of these symptoms in the patient's history.

Seborrheic dermatitis (choice E) is a chronic inflammatory skin condition that can affect the external ear canal. However, the patient's symptoms and examination findings are more consistent with otitis externa, which is specifically characterized by infection or inflammation of the external ear canal.
Based on the information provided, the most likely diagnosis is otitis externa (Option C). Otitis externa, commonly known as swimmer's ear, is an inflammation of the ear canal that is characterized by the symptoms described in the case, including itching, pain, and redness of the ear canal. The condition can be caused by exposure to water, which may explain why the patient initially thought it was related to her earplugs.
95
A 23-year-old woman who is an active-duty enlisted airman in the US Air Force comes to the base clinic because of
a 4-week history of an itchy sensation in her right ear. She has not had ringing in her ears or hearing loss. She works
as a fuel specialist and at first attributed her symptom to the earplugs she uses when she is on the flight line. The
symptom has not improved despite the patient's wearing headphones instead of the earplugs during the past 2 weeks.
She has no history of serious illness and takes no medications. Vital signs are within normal limits. During
examination, pulling of the right pinna inferiorly to examine the ear canal produces pain. The right ear canal appears
erythematous and edematous. The tympanic membrane cannot be fully visualized because of the presence of
cerumen. Examination of the left ear shows no abnormalities. Which of the following is the most likely diagnosis and how did you arrive to that conclusion? Why are the other choices not likely?


(A) Cerumen impaction
(B) Ear canal trauma
(C) Otitis externa
(D) Polychondritis
(E) Seborrheic dermatitis
(C) Otitis externaCorrect
The most likely diagnosis in this case is (C) Otitis externa.

Otitis externa, also known as swimmer's ear, is an infection or inflammation of the external ear canal. It is commonly caused by bacteria, such as Pseudomonas aeruginosa or Staphylococcus aureus. The patient's symptoms of itching and pain in the right ear, along with the erythematous and edematous appearance of the ear canal, are consistent with otitis externa.

Cerumen impaction (choice A) is the buildup of earwax in the ear canal. While cerumen impaction can cause symptoms such as hearing loss and ear fullness, it is unlikely to cause the itching and pain described by the patient. Additionally, the presence of cerumen obstructing the view of the tympanic membrane suggests that the primary issue is inflammation and infection of the ear canal rather than cerumen impaction.

Ear canal trauma (choice B) is a possibility, especially given the patient's occupation as a fuel specialist. However, trauma alone is unlikely to cause the persistent itching and pain described by the patient. The presence of erythema and edema in the ear canal suggests an inflammatory process, which is more consistent with otitis externa.

Polychondritis (choice D) is a rare autoimmune condition characterized by inflammation of cartilage throughout the body. While it can affect the ear, it typically presents with symptoms such as pain, redness, and swelling of the pinna rather than symptoms localized to the ear canal. The absence of these findings in the patient's examination makes polychondritis less likely.

Seborrheic dermatitis (choice E) is a chronic inflammatory skin condition that can affect various areas of the body, including the scalp, face, and ears. While seborrheic dermatitis can cause itching and redness, it typically presents with a scaly or greasy appearance of the affected skin. The absence of these findings in the patient's examination makes seborrheic dermatitis less likely.

In summary, based on the patient's symptoms, examination findings, and the most likely causes of ear canal inflammation, otitis externa is the most likely diagnosis in this case.
Based on the symptoms described in the scenario, the most likely diagnosis is otitis externa, or inflammation of the outer ear canal. This is indicated by the patient's history of an itchy sensation in the right ear, the presence of erythema and edema in the right ear canal, and the pain that is produced when the right pinna is pulled inferiorly.

Of the options given, cerumen impaction is not likely to be the diagnosis because the symptoms described do not suggest a blockage of the ear canal by earwax. Ear canal trauma is not likely to be the diagnosis because the patient does not have any history of trauma to the ear, and the presence of erythema and edema in the ear canal is not typically seen with trauma. Polychondritis is not likely to be the diagnosis because this condition typically affects cartilage in the nose, ears, and respiratory tract, and the patient does not have any symptoms that suggest involvement of these structures. Seborrheic dermatitis is not likely to be the diagnosis because this condition typically affects the scalp and face, and the patient does not have any symptoms that suggest involvement of these areas.
96
A 34-year-old woman who is on active duty in the US Army comes to the military urgent care clinic because of a 4-
day history of headache, purulent nasal discharge, and left facial pain. She has not had fever or chills. She has
chronic bilateral knee pain, which she attributes to multiple parachute jumps while she was assigned to an airborne
unit 7 years ago. The knee pain is exacerbated by long-distance running on pavement. Her only medication is
occasional ibuprofen for the knee pain. The patient has no known medication allergies. Temperature is 37.1°C
(98.8°F), pulse is 60/min, respirations are 14/min, and blood pressure is 122/80 mm Hg. Cervical lymph nodes are
not palpable. There is tenderness to palpation over the left maxillary sinus. Nasal examination shows inflamed
turbinates and purulent discharge. The patient needs to blow her nose twice during the examination. Bilateral
otoscopic examination discloses no abnormalities. Examination of the oropharynx shows scant purulent discharge
posteriorly. The remainder of the examination discloses no abnormalities. Which of the following is the most
appropriate next step in management?


(A) CT scan of the sinuses
(B) Gram stain and culture of nasal discharge
(C) Ibuprofen and decongestant therapy
(D) Trimethoprim-sulfamethoxazole therapy
(E) X-ray of the sinuses
(C) Ibuprofen and decongestant therapy
Correct
The correct choice is (C) Ibuprofen and decongestant therapy.

Explanation:
Based on the patient's history and physical examination findings, the most likely diagnosis is acute sinusitis. Acute sinusitis is typically caused by a viral infection, and antibiotics are not necessary unless there are signs of severe infection or complications. In this case, the patient does not have fever or systemic symptoms, and there are no signs of orbital or intracranial complications. Therefore, the most appropriate next step in management is symptomatic treatment with ibuprofen and decongestant therapy.

Ibuprofen can help relieve the headache and facial pain associated with sinusitis. Decongestants, such as pseudoephedrine, can help reduce nasal congestion and improve sinus drainage. These medications can provide symptomatic relief while the viral infection runs its course.

CT scan of the sinuses (choice A) is not necessary in this case because the diagnosis of acute sinusitis can be made based on the patient's history and physical examination findings. CT scans are reserved for cases with severe or complicated sinusitis.

Gram stain and culture of nasal discharge (choice B) are not necessary for the diagnosis of acute sinusitis. These tests are typically reserved for cases of chronic sinusitis or when there is suspicion of a bacterial infection.

Trimethoprim-sulfamethoxazole therapy (choice D) is not necessary in this case because the most likely cause of the patient's symptoms is a viral infection. Antibiotics are not indicated for viral sinusitis.

X-ray of the sinuses (choice E) is not necessary in this case because it does not provide additional information beyond what can be obtained from the physical examination. CT scan is a more sensitive imaging modality if further evaluation is needed.
The most appropriate next step in management would be to perform a gram stain and culture of the nasal discharge to determine the cause of the infection. This will help guide treatment and determine if the infection is caused by bacteria that is susceptible to trimethoprim-sulfamethoxazole therapy. A CT scan of the sinuses may also be useful in certain cases to assess the extent of the infection and to guide treatment, but it is not the most immediate next step in management. Ibuprofen and decongestant therapy may help with the symptoms but will not treat the underlying infection. X-ray of the sinuses is not typically useful in the management of sinus infections.
97
A 34-year-old woman who is on active duty in the US Army comes to the military urgent care clinic because of a 4-
day history of headache, purulent nasal discharge, and left facial pain. She has not had fever or chills. She has
chronic bilateral knee pain, which she attributes to multiple parachute jumps while she was assigned to an airborne
unit 7 years ago. The knee pain is exacerbated by long-distance running on pavement. Her only medication is
occasional ibuprofen for the knee pain. The patient has no known medication allergies. Temperature is 37.1°C
(98.8°F), pulse is 60/min, respirations are 14/min, and blood pressure is 122/80 mm Hg. Cervical lymph nodes are
not palpable. There is tenderness to palpation over the left maxillary sinus. Nasal examination shows inflamed
turbinates and purulent discharge. The patient needs to blow her nose twice during the examination. Bilateral
otoscopic examination discloses no abnormalities. Examination of the oropharynx shows scant purulent discharge
posteriorly. The remainder of the examination discloses no abnormalities. Among the choices, why is your answer the most most
appropriate next step in management and not the others?


(A) CT scan of the sinuses
(B) Gram stain and culture of nasal discharge
(C) Ibuprofen and decongestant therapy
(D) Trimethoprim-sulfamethoxazole therapy
(E) X-ray of the sinuses
(C) Ibuprofen and decongestant therapy
Correct
The most appropriate next step in management for this patient is (C) Ibuprofen and decongestant therapy.

The patient's history and physical examination findings are consistent with acute sinusitis. Sinusitis is typically a clinical diagnosis and imaging studies such as CT scan or X-ray of the sinuses are not necessary in most cases. CT scan of the sinuses (choice A) is only indicated if there are signs of orbital or intracranial complications, chronic sinusitis that has not responded to treatment, or concern about rare nasopharyngeal cancer. X-ray of the sinuses (choice E) is not recommended for the diagnosis of sinusitis.

Gram stain and culture of nasal discharge (choice B) may be considered in severe or complicated cases of sinusitis, but it is not necessary for the initial management of uncomplicated acute sinusitis.

Trimethoprim-sulfamethoxazole therapy (choice D) is an option for the treatment of acute bacterial sinusitis, but it is not the most appropriate next step in management for this patient. The patient does not have severe symptoms or signs of complications, and initial treatment with ibuprofen and decongestant therapy is sufficient.

Ibuprofen and decongestant therapy (choice C) is the most appropriate next step in management for this patient. Ibuprofen can help relieve the headache and facial pain associated with sinusitis, while decongestants can help reduce nasal congestion and improve sinus drainage. This conservative management approach is recommended as the first-line treatment for uncomplicated acute sinusitis. If symptoms persist or worsen despite conservative management, then antibiotic therapy may be considered.
Based on the information provided, the most appropriate next step in management would be to perform a Gram stain and culture of the nasal discharge. This will allow the healthcare provider to identify the specific type of bacteria causing the infection and determine the most effective course of treatment. Options A, C, D, and E would not be as helpful in identifying the cause of the infection and determining the appropriate treatment.
98
A 16-year-old boy with VATER syndrome is brought to the physician because of 2-week history of harsh cough that
began with a common cold. He receives no medications. Temperature is 37°C (98.6°F), pulse is 70/min, respirations
are 20/min, and blood pressure is 112/64 mm Hg. Examination shows a harsh vibratory sound in the mid anterior
chest with forced expiration. A systolic murmur is heard best at the lower left sternal border. There is mild scoliosis.
Spirometry shows an FVC of 2.54 L (78% of predicted) and a slow vital capacity of 2.88 L (85% of predicted). A
flow volume loop shows scooping on the expiratory limb. Which of the following is the most likely cause of the
discrepancy between this patient’s forced and slow vital capacity measurements?


(A) Bronchospasm
(B) Laryngeal braking
(C) Subglottic mass
(D) Tracheal collapse
(E) Upper airway compression
(D) Tracheal collapseCorrect
The correct choice is (D) Tracheal collapse.

Explanation:
The discrepancy between the forced vital capacity (FVC) and slow vital capacity (SVC) measurements in this patient is likely due to tracheal collapse. Tracheal collapse is a condition in which the trachea, or windpipe, becomes weakened and collapses during expiration, leading to airflow limitation. This can result in a scooping appearance on the expiratory limb of the flow volume loop, as seen in this patient.

Bronchospasm (choice A) is a constriction of the airway smooth muscles in the bronchi, which can cause wheezing and difficulty breathing. However, bronchospasm would not explain the discrepancy between the FVC and SVC measurements.

Laryngeal braking (choice B) refers to the protective reflex that occurs during forced expiration to prevent overinflation of the lungs. It involves the closure of the vocal cords and the contraction of the expiratory muscles. Laryngeal braking would not cause a discrepancy between the FVC and SVC measurements.

Subglottic mass (choice C) refers to a growth or obstruction in the area below the vocal cords but above the trachea. While a subglottic mass can cause airway obstruction and respiratory symptoms, it would not explain the discrepancy between the FVC and SVC measurements.

Upper airway compression (choice E) refers to external compression of the upper airway, which can occur due to various causes such as tumors or enlarged lymph nodes. While upper airway compression can cause respiratory symptoms, it would not explain the discrepancy between the FVC and SVC measurements.
It is likely that the discrepancy between this patient's forced and slow vital capacity measurements is due to laryngeal braking, which is characterized by a harsh vibratory sound in the mid-anterior chest and forced expiration. Laryngeal braking can occur in patients with VATER syndrome, which is a rare congenital condition characterized by a combination of vertebral, anal, tracheal, esophageal, and renal anomalies.
99
A 16-year-old boy with VATER syndrome is brought to the physician because of 2-week history of harsh cough that
began with a common cold. He receives no medications. Temperature is 37°C (98.6°F), pulse is 70/min, respirations
are 20/min, and blood pressure is 112/64 mm Hg. Examination shows a harsh vibratory sound in the mid anterior
chest with forced expiration. A systolic murmur is heard best at the lower left sternal border. There is mild scoliosis.
Spirometry shows an FVC of 2.54 L (78% of predicted) and a slow vital capacity of 2.88 L (85% of predicted). A
flow volume loop shows scooping on the expiratory limb. Which of the following is the most likely cause of the
discrepancy between this patient’s forced and slow vital capacity measurements? Why are the other choices less likely the cause?


(A) Bronchospasm
(B) Laryngeal braking
(C) Subglottic mass
(D) Tracheal collapse
(E) Upper airway compression
(D) Tracheal collapseCorrect
The correct choice is (D) Tracheal collapse.

Explanation:
The discrepancy between the forced vital capacity (FVC) and slow vital capacity (SVC) measurements in this patient is likely due to tracheal collapse. Tracheal collapse is a condition characterized by the narrowing or collapse of the trachea, leading to airflow limitation. In this patient, the flow volume loop shows scooping on the expiratory limb, which is consistent with tracheal collapse.

Bronchospasm (choice A) is the constriction of the smooth muscles in the bronchi, leading to narrowing of the airways. While bronchospasm can cause airflow limitation, it would not explain the discrepancy between the FVC and SVC measurements seen in this patient.

Laryngeal braking (choice B) refers to the reflexive closure of the vocal cords during forced expiration, which can limit airflow. However, laryngeal braking would not explain the discrepancy between the FVC and SVC measurements seen in this patient.

Subglottic mass (choice C) refers to the presence of a mass in the area below the vocal cords. While a subglottic mass can cause airflow limitation, it would not explain the discrepancy between the FVC and SVC measurements seen in this patient.

Upper airway compression (choice E) refers to the compression of the upper airway, which can lead to airflow limitation. However, upper airway compression would not explain the discrepancy between the FVC and SVC measurements seen in this patient.

Therefore, the most likely cause of the discrepancy between the FVC and SVC measurements in this patient is tracheal collapse.
It is most likely that the discrepancy between the patient's forced and slow vital capacity measurements is caused by laryngeal braking. Laryngeal braking occurs when the larynx (voice box) narrows during exhalation, impeding the flow of air. This can cause the expiratory limb of the flow volume loop to appear scooped. The other options are less likely because bronchospasm typically causes a reduction in both forced and slow vital capacity, and a subglottic mass or tracheal collapse would likely cause a more severe obstruction to airflow. Upper airway compression may cause a reduction in forced vital capacity, but it would not typically affect slow vital capacity.
100
A chief medical officer at a hospital has received several reports that one of her colleagues, a 64-year-old male
surgeon, has “not been acting like himself” recently. An administrative assistant says that during the past 6 months,
the surgeon’s dictations have become hard to follow and appear to be missing information; previously, they were
comprehensive and easy to transcribe. The assistant adds that after the surgeon missed three meetings in a row
during the past 2 months, he asked her to send him a text message 5 minutes before the next meeting to let him
know its location. Previously, he was always in attendance and on time for meetings. A senior resident also has
noted unusual behavior by the surgeon; the resident says that while on rounds 3 days ago, the surgeon listed
incorrect indications for a specific surgical procedure. A scrub nurse reported that the surgeon used unsterile
practices three times during a 2-hour procedure today. The chief medical officer meets with the surgeon and asks
him about these reported incidents. The surgeon says that “nothing is wrong” and the people who reported him are
angry with him for various reasons and “are trying to stir up trouble.” Which of the following is the most appropriate
action for the chief medical officer to take?


(A) Accept the surgeon’s answer and only pursue further action if there is another complaint
(B) Determine whether other staff have noticed anything different about the surgeon’s behavior
(C) Inform the surgeon that he should consider retirement
(D) Instruct the surgeon to take a temporary leave of absence and undergo a medical evaluation
(D) Instruct the surgeon to take a temporary leave of absence and undergo a medical evaluation
Correct
The correct answer is (D) Instruct the surgeon to take a temporary leave of absence and undergo a medical evaluation.

Explanation:
The reports from multiple sources about the surgeon's unusual behavior and lapses in professional conduct raise concerns about his ability to safely and effectively perform his duties. The chief medical officer has a responsibility to ensure patient safety and the well-being of the surgeon. Accepting the surgeon's answer without further action or waiting for another complaint would not address the potential issues at hand.

Determining whether other staff have noticed anything different about the surgeon's behavior is a reasonable step to gather more information. However, it should not be the only action taken, as the reports from multiple sources already indicate a pattern of concerning behavior.

Informing the surgeon to consider retirement is premature and may not be necessary if the underlying cause of the behavior can be identified and addressed.

Instructing the surgeon to take a temporary leave of absence and undergo a medical evaluation is the most appropriate action in this situation. This allows for a thorough assessment of the surgeon's physical and mental health, which could help identify any underlying medical conditions or personal issues that may be contributing to the reported behavior. It also ensures patient safety and protects the reputation of the hospital.
(D) Instruct the surgeon to take a temporary leave of absence and undergo a medical evaluation.

It is concerning that several people have reported unusual behavior by the surgeon, and that he has made several mistakes in his work. As the chief medical officer, it is important to take these reports seriously and take appropriate action to ensure the safety of the patients and the quality of care at the hospital. In this case, the most appropriate action would be to instruct the surgeon to take a temporary leave of absence and undergo a medical evaluation to determine the cause of his unusual behavior and ensure that he is fit to continue practicing medicine. This will also give the chief medical officer time to investigate the reported incidents and determine whether any further action is needed.